Download as pdf or txt
Download as pdf or txt
You are on page 1of 74

PRELIMS PRACTICE

WORKBOOK
POLITY

CONTENTS
 POLITY PREVIOUS YEAR QUESTIONS ................................................................. 01-28
1. Evolution Of The Constitution ......................................................................................... 1
2. Salient Features Of Indian Constitution ........................................................................ 1
3. Union & Its Territory ......................................................................................................... 4
4. Fundamental Rights.......................................................................................................... 4
5. Directive Principles Of State Policy (Dpsp) .................................................................... 6
6. Fundamental Duties ......................................................................................................... 7
7. System Of Governance ..................................................................................................... 8
8. Union Executive ................................................................................................................. 9
9. Union Legislature ............................................................................................................ 10
10. State Executive ................................................................................................................ 15
11. State Legislature ............................................................................................................. 15
12. Local Government ........................................................................................................... 16
13. Union Territories And Special Areas ............................................................................. 17
14. The Judiciary .................................................................................................................... 17
15. Constitutional And Non-Constitutional Bodies ........................................................... 19
16. Governance & Ir ............................................................................................................... 20
17. Miscellaneous .................................................................................................................. 27

 POLITY ANSWERS ................................................................................................. 29-65


1. Evolution Of The Constitution ....................................................................................... 29
2. Salient Features Of Indian Constitution ...................................................................... 29
3. Union & Its Territory ....................................................................................................... 33
4. Fundamental Rights........................................................................................................ 33
5. Directive Principles Of State Policy (Dpsp) .................................................................. 36
6. Fundamental Duties ....................................................................................................... 37
7. System Of Governance ................................................................................................... 38
8. Union Executive ............................................................................................................... 39
9. Union Legislature ............................................................................................................ 41
10. State Executive ................................................................................................................ 47
11. State Legislature ............................................................................................................. 48
12. Local Government ........................................................................................................... 49
13. Union Territories And Special Areas ............................................................................. 50
14. The Judiciary .................................................................................................................... 51
15. Constitutional And Non-Constitutional Bodies ........................................................... 53
16. Governance & Ir ............................................................................................................... 54
17. Miscellaneous .................................................................................................................. 65
POLITY & GOVERNANCE
PREVIOUS YEAR QUESTIONS

EVOLUTION OF THE CONSTITUTION

1. In India, which one of the following Constitutional 3. What was the exact constitutional status of India
Amendments was widely believed to be enacted on 26th January, 1950?
to overcome the judicial interpretations of the
Fundamental Rights? (a) A Democratic Republic
(a) 1 Amendment
st
(b) A Sovereign Democratic Republic
(b) 42nd Amendment (c) A Sovereign Secular Democratic Republic
(c) 44th Amendment
(d) A Sovereign Socialist Secular Democratic Republic
(d) 86th Amendment
4. Constitutional government means
2. The “Instrument of Instructions” contained in
the Government of India Act 1935 have been (a) a representative government of nation with
incorporated in the Constitution of India in the federal structure
year 1950 as:
(b) a government whose Head enjoys nominal powers
(a) Fundamental Rights
(c) a government whose Head enjoys real powers
(b) Directive Principles of State Policy
(d) a government limited by the terms of the
(c) Extent of executive power of State Constitution
(d) Conduct of business of the Government of India

SALIENT FEATURES OF INDIAN CONSTITUTION

1. Which one of the following statements best reflects 3. The Ninth Schedule was introduced in the Constitution
the Chief purpose of the ‘Constitution’ of a country? of India during the prime ministership of
(a) It determines the objective for the making of (a) Jawaharlal Nehru (c) Indira Gandhi
necessary laws. (b) Lal Bahadur Shastri (d) Morarji Desai
(b) It enables the creation of political offices and a
government. 4. Under which Schedule of the Constitution of India
can the transfer of tribal land to private parties for
(c) It defines and limits the powers of government. mining be declared null and void?
(d) It secures social justice, social equality and social (a) Third Schedule (c) Ninth Schedule
security.
(b) Fifth Schedule (d) Twelfth Schedule
2. In essence, what does ‘Due Process of Law’ means?
5. In the context of polity, which one of the following
(a) The principle of natural Justice would you accept as the most appropriate definition
(b) The procedure established by law of liberty?
(c) Fair application of law (a) Protection against the tyranny of political rulers
(d) Equality before law (b) Absence of restraint

1
POLITY WORKBOOK www.iasscore.in

(c) Opportunity to do whatever one likes 11. The main advantage of the parliamentary form of
government is that
(d) Opportunity to develop oneself fully
(a) The executive and legislature work independently.
6. With reference to the Constitution of India,
(b) It provides continuity of policy and is more efficient.
prohibitions or limitations or provisions contained
in ordinary laws cannot act as prohibitions or (c) The executive remains responsible to the
limitations on the constitutional powers under legislature.
Article 142. It could mean which one of the following?
(d) The head of the government cannot be changed
(a) The decisions taken by the Election Commission without election.
of India while discharging its duties cannot be
challenged in any court of law. 12. The mind of the makers of the Constitution of India
(b) The Supreme Court of India is not constrained in is reflected in which of the following?
the exercise of its powers by laws made by the (a) The Preamble
Parliament.
(b) The Fundamental Rights
(c) In the event of grave financial crisis in the country,
the President of India can declare Financial (c) The Directive Principles of State Policy
Emergency without the counsel from the Cabinet. (d) The Fundamental Duties
(d) State legislatures cannot make law on certain
matters without the concurrence of Union 13. One of the implications of equality in society is the
Legislature. absence of
(a) Privileges (c) Competition
7. Which one of the following reflects the most
appropriate relationship between law and liberty? (b) Restraints (d) Ideology
(a) It there are more laws, there is less liberty.
14. Which one of the following statements is correct?
(b) If there are no laws, there is no liberty.
(a) Right are claims of the State against the citizens.
(c) If there is liberty, laws have to be made by the
(b) Rights are privileges which are incorporated in
people.
the Constitution of a State.
(d) If laws are changed too often, liberty is in danger
(c) Rights are claims of the citizens against the State.
8. Consider the following statements: (d) Rights are privileges of a few citizens against the
1. The Parliament of India can place a particular many.
law in the Ninth Schedule of the Constitutions of
India. 15. Which one of the following is not a feature of
2. The validity of a law placed in the Ninth Schedule Indian Federalism ?
cannot be examined by any court and no (a) There is an independent judiciary in India.
judgement can be made on it.
(b) Powers have been clearly divided between the
Which of the statements given above is/are correct? Centre and the States.
(a) 1 only (c) Both 1 and 2 (c) The federating units have been given unequal
(b) 2 only (d) Neither 1 nor 2 representation in the Rajya Sabha.
(d) It is the result of an agreement among the
9. Which one of the following objectives is not
federating units.
embodied in the Preamble to the Constitution of
India?
16. Right to vote and to be elected in India is a :
(a) Liberty of thought (c) Liberty of expression
(a) Fundamental Right (c) Constitutional Right
(b) Economic liberty (d) Liberty of belief
(b) Natural Right (d) Legal Right
10. Democracy’s superior virtue lies in the fact that it
calls into activity 17. There is a Parliamentary System of Government in
India because the
(a) The intelligence and character of ordinary men
and women. (a) Lok Sabha is elected directly by the people
(b) The methods for strengthening executive (b) Parliament can amend the Constitution
leadership
(c) Rajya Sabha cannot be dissolved
(c) A superior individual with dynamism and vision.
(d) Council of Ministers is responsible to the Lok
(d) A band of dedicated party workers. Sabha

2
www.iasscore.in POLITY WORKBOOK

18. Which one of the following Schedules of the 24. Which one of the following in Indian polity is an
Constitution of India contains provisions regarding essential feature that indicates that it is federal in
anti-defection? character?
(a) Second Schedule (c) Eighth Schedule (a) The independence of judiciary is safeguarded.
(b) Fifth Schedule (d) Tenth Schedule
(b) The Union Legislature has elected representatives
19. ‘Economic Justice’ as one of the objectives of the from constituent units.
Indian Constitution has been provided in (c) The Union Cabinet can have elected
(a) The Preamble and Fundamental Rights representatives from regional parties.
(b) The Preamble and the Directive Principles of (d) The Fundamental Rights are enforceable by
State Policy Courts of Law.
(c) The Fundamental Rights and the Directive
Principles of State Policy 25. With reference to India, consider the following
(d) None of the above statements:
1. There is only ‘one citizenship and one domicile’.
20. A constitutional government by definition is a
2. A citizen by birth only can become the Head of
(a) government by legislature State.
(b) popular government 3. A foreigner once granted the citizenship cannot
(c) multi-party government be deprived of it under any circumstances.
(d) limited government Which of the statements given above is/are correct?

21. Consider the following statements: (a) 1 only (c) 1 and 3 only
1. The Constitution of India defines its ‘basic (b) 2 only (d) 2 and 3 only
structure’ in terms of federalism, secularism,
fundamental rights and democracy. 26. We adopted parliamentary democracy based on
2. The Constitution of India provides for ‘judicial the British model, but how does our model differ
review’ to safeguard the citizens’ liberties and to from that model?
preserve the ideals on which the Constitution is
1. As regards legislation, the British Parliament is
based.
supreme or sovereign but in India, the power of
Which of the statements given above is/are correct? the Parliament to legislate is limited.
(a) 1 only (c) Both 1 and 2 2. In India, matters related to the constitutionality
(b) 2 only (d) Neither 1 nor 2 of the Amendment of an Act of the Parliament
are referred to the Constitution Bench by the
22. The Preamble to the Constitution of India is Supreme Court.
(a) a part of the Constitution but has no legal effect
Select the correct answer using the code given below.
(b) not a part of the Constitution and has no legal
effect either (a) 1 only (c) Both 1 and 2

(c) a part of the Constitution and has the same legal (b) 2 only (d) Neither 1 nor 2
effect an any other part
27. Consider the following statements:
(d) a part of the Constitution but has no legal effect
independently of other parts 1. A bill amending the Constitution requires a prior
recommendation of the President of India.
23. We adopted parliamentary democracy based on
2. When a Constitution Amendment Bill is presented
the British model, but how does our model differ
from that model? to the President of India, it is obligatory for the
President of India to give his/her assent.
1. As regards legislation, the British Parliament is
supreme or sovereign but in India, the power of 3. A Constitution Amendment Bill must be passed
the Parliament to legislate is limited. by both the Lok Sabha and Rajya Sabha by a
2. In India, matters related to the constitutionality special majority and there is no provision for
of the Amendment of an Act of the Parliament joint sitting.
are referred to the Constitution Bench by the
Supreme Court. Which of the statements given above are correct?

Select the correct answer using the code given below. (a) 1 and 2 only (c) 1 and 3 only
(a) 1 only (c) Both 1 and 2 (b) 2 and 3 only (d) 1, 2 and 3
(b) 2 only (d) Neither 1 nor 2

3
POLITY WORKBOOK www.iasscore.in

UNION & ITS TERRITORY

1. With reference to the Delimitation Commission, Assembly, they cannot effect any modifications
consider the following statements: in the orders.
1. The orders of the Delimitation Commission Which of the statements given above is/are correct?
cannot be challenged in a Court of Law.
(a) 1 only (c) Both 1 and 2
2. When the orders of the Delimitation Commission
are laid before the Lok Sabha or State Legislative (b) 2 only (d) Neither 1 nor 2

FUNDAMENTAL RIGHTS

1. Which Article of the Constitution of India safeguards 5. Which of the following are envisaged by the Right
one’s right to marry the person of one’s choice? against Exploitation in the Constitution of India?
(a) Article 19 (c) Article 25 1. Prohibition of traffic in human beings and forced
labour
(b) Article 21 (d) Article 29
2. Abolition of untouchability
2. Which of the following are regarded as the main 3. Protection of the interests of minorities
features of the “Rule of Law”?
4. Prohibition of employment of children in
1. Limitation of powers factories and mines
2. Equality before law Select the correct answer using the code given below :
3. People’s responsibility to the government (a) 1, 2 and 4 only (c) 1 and 4 only
4. Liberty and civil rights (b) 2, 3 and 4 only (d) 1, 2, 3 and 4
Select the correct answer using the code given below:
6. In India, Judicial Review implies :
(a) 1 and 3 only (c) 1,2 and 4 only (a) The power of the Judiciary to pronounce upon the
(b) 1 and 4 only (d) 1, 2, 3, and only constitutionality of laws and executive orders.
(b) The power of the Judiciary to question the wisdom
3. Right to Privacy is protected as an intrinsic part of of the laws enacted by the Legislatures.
Right to Life and Personal Liberty. Which of the
following in the Constitution of India correctly and (c) The power of the Judiciary to review- all the
appropriately imply the above statements? legislative enactments before they – are assented
to by the President.
(a) Article 14 and the provisions under the 42nd
Amendment to the Constitution (d) The power of the Judiciary to review its own
(b) Article 17 and the Directive Principles of State judgements given earlier in similar or different
Policy in Part IV cases?

(c) Article 21 and the freedoms guaranteed in Part III 7. In the Constitution of India, promotion of
(d) Article 24 and the provisions under the 44th international peace and security is included in the
Amendment to the Constitution (a) Preamble to the Constitution

4. In the context of India, which one of the following is (b) Directive Principles of State Policy
the correct relationship between Rights and Duties? (c) Fundamental Duties
(a) Rights are correlative with Duties. (d) Ninth Schedule
(b) Rights are personal and hence independent of
society and Duties. 8. Which of the following provisions of the
Constitution of India have a bearing on Education?
(c) Rights, not Duties, are important for the
advancement of the personality of the citizen. 1. Directive Principles of State Policy
(d) Duties, not Rights, are important for the stability 2. Rural and Urban Local Bodies
of the State. 3. Fifth Schedule

4
www.iasscore.in POLITY WORKBOOK

4. Sixth Schedule 3. ‘Right to the City’ means that the State cannot
5. Seventh Schedule deny any public service or facility to the
unauthorized colonies in the city.
Select the correct answer using the codes given below:
Which of the statements given above is/are correct?
(a) 1 and 2 only (c) 1, 2 and 5 only
(a) 1 only (c) 1 and 2 only
(b) 3, 4 and 5 only (d) 1, 2, 3 4 and 5
(b) 3 only (d) 2 and 3 only
9. Which of the following is/are among the
Fundamental Duties of citizens laid down in the 14. With reference to India, consider the following
Indian Constitution? statements:
1. To preserve the rich heritage of our composite 1. Judicial custody means an accused is in the
culture custody of the concerned magistrate and such
accused is locked up in police station, not in jail.
2. To protect the weaker sections from social
injustice 2. During judicial custody, the police officer in
charge of the case is not allowed to interrogate
3. To develop the scientific temper and spirit of
the suspect without the approval of the court.
inquiry
4. To strive towards excellence in all spheres of Which of the statements given above is/are correct?
individual and collective activity. (a) 1 only (c) Both 1 and 2
Select the correct answer using the codes given below: (b) 2 only (d) Neither 1 nor 2
(a) 1 and 2 only (c) 1, 3 and 4 only
15. With reference to India, consider the following
(b) Only 2 (d) 1, 2, 3 and 4 statements:
1. When a prisoner makes out a sufficient case,
10. In India, if a religious sect/community is given
parole cannot be denied to such prisoner because
the status of a national minority, what special
it becomes a matter of his/her right.
advantages it is entitled to?
2. State Governments have their own Prisoners
1. It can establish and administer exclusive
Release on Parole Rules.
educational institutions.
2. The President of India automatically nominates Which of the statements given above is/are correct?
a representative of the community of Lok Sabha. (a) 1 only (c) Both 1 and 2
3. It can derive benefits from the Prime Minister’s (b) 2 only (d) Neither 1 Nor 2
15-Point Programme.
Select the correct answer using the codes given below: 16. A legislation which confers on the executive
or administrative authority an unguided and
(a) 1, 2 and 3 (c) 1 and 2 only uncontrolled discretionary power in the matter
(b) 1 only (d) 2 and 3 only of application of law violates which one of the
following articles of the Constitution of India?
11. Which one of the following categories of
(a) Article 14 (c) Article 32
Fundamental Rights incorporates protection
against untouchability as a form of discrimination? (b) Article 28 (d) Article 44
(a) Right against Exploitation 17. Which one of the following best defines the term
(b) Right to Freedom ‘State’?
(c) Right to Constitutional Remedies (a) A community of persons permanently occupying
a definite territory independent of external
(d) Right to Equality
control and possessing an organized government
12. ‘Right to Privacy’ is protected under which Article (b) A politically organized people of a definite
of the Constitution of India? territory and possessing an authority to govern
(a) Article 15 (c) Article 21 them, maintain law and order, protect their
natural rights and safeguard their means of
(b) Article 19 (d) Article 29 sustenance
13. Consider the following statements: (c) A number of persons who have been living in a
1. ‘Right to the City’ is an agreed human right and definite territory for a very long time with their
the UN-Habitat monitors the commitments made own culture, tradition and government
by each country in this regard. (d) A society permanently living in a definite territory
2. ‘Right to the City’ gives every occupant of the with a central authority, an executive responsible
city the right to reclaim public spaces and public to the central authority and an independent
participation in the city. judiciary

5
POLITY WORKBOOK www.iasscore.in

18. What is the position of the Right to Property in (b) Legal right available to any person
India?
(c) Fundamental Right available to citizens only
(a) Legal right available to citizens only
(d) Neither Fundamental Right nor legal right

DIRECTIVE PRINCIPLES OF STATE POLICY (DPSP)

1. Consider the following statements: Which of the statements given above is/are correct?
Once the Central Government notifies an area as a (a) 1 only (c) Both 1 and 2
‘Community Reserve’ (b) 2 only (d) Neither 1 nor 2
1. The Chief Wildlife Warden of the State becomes
the governing authority of such forest. 5. The ideal of ‘Welfare State’ in the Indian
2. Hunting is not allowed in such area. Constitution is enshrined in its
3. People of such area are allowed to collect non- (a) Preamble
timber forest produce
(b) Directive Principles of State Policy
4. People of such area are allowed traditional
agricultural practices (c) Fundamental Rights
How many of the above statements are correct? (d) Seventh Schedule
(a) Only one (c) Only three
6. According to the Constitution of India, which of the
(b) Only two (d) All four following are fundamental for the governance of
the country?
2. Which principle among the following was added to
the Directive Principles of State Policy by the 42nd (a) Fundamental Rights
Amendment to the constitution? (b) Fundamental Duties
(a) Equal pay for equal work for both men and (c) Directive Principles of state Policy
women (d) Fundamental Rights and Fundamental Duties
(b) Participation of workers in the management of
industries 7. Consider the following provisions under the
Directive Principles of State Policy as enshrined in
(c) Right to work, education and public assistance
the Constitution of India:
(d) Securing living wage and human conditions of 1. Securing for citizens of India a uniform civil
work to workers code

3. Consider the following statements: 2. Organising village panchayats


3. Promoting cottage industries in rural areas
With reference to the Constitution of India, the
Directive Principles of State Policy constitute 4. Securing for all the workers reasonable leisure
limitations upon and cultural opportunities.
1. legislative function Which of the above are the Gandhian Principles that
2. executive function are reflected in the Directive Principles of State Policy?

Which of the above statements is/are correct? (a) 1, 2 and 4 only (c) 1, 3 and 4 only

(a) 1 only (c) Both 1 and 2 (b) 2 and 3 only (d) 1, 2, 3 and 4

(b) 2 only (d) Neither 1 nor 2


8. Other than the Fundamental Rights, which of
the following parts of the Constitution of India
4. Consider the following statements regarding the
reflect/reflects the principles and provisions of the
Directive Principles of State Policy:
Universal Declaration of Human Rights (1948)?
1. The Principles spell out the socio-economic
democracy in the country. 1. Preamble

2. The provisions contained in these Principles are 2. Directive Principles of State Policy
not enforceable by any court. 3. Fundamental Duties

6
www.iasscore.in POLITY WORKBOOK

Select the correct answer using the code given below: 11. Which one of the following factors constitutes the
(a) 1 and 2 only (c) 1 and 3 only best safeguard of liberty in a liberal democracy?

(b) 2 only (d) 1, 2 and 3 (a) A committed judiciary


(b) Centralization of powers
9. Which part of the Constitution of India declares
(c) Elected government
the ideal of Welfare State?
(d) Separation of powers
(a) Directive Principles of State Policy
(b) Fundamental Rights 12. Under the Indian Constitution, concentration of
(c) Preamble wealth violates

(d) Seventh Schedule (a) the Right to Equality


(b) the Directive Principles of State Policy
10. With reference to the provisions contained in
Part IV of the Constitution of India, which of the (c) the Right to Freedom
following statements is/are correct? (d) the Concept of Welfare
1. They shall be enforceable by courts.
13. Which one of the following factors constitutes the
2. They shall not be enforceable by any court.
best safeguard of liberty in a liberal democracy?
3. The principles laid down in this part are to
(a) A committed judiciary
influence the making of laws by the State.
(b) Centralization of powers
Select the correct answer using the code given below:
(c) Elected government
(a) 1 only (c) 1 and 3 only
(d) Separation of powers
(b) 2 only (d) 2 and 3 only

FUNDAMENTAL DUTIES

1. Which of the following statements is/are true of (a) Preamble of the Constitution
the Fundamental Duties of an Indian citizen? (b) Directive Principles of State Policy
1. A legislative process has been provided to (c) Fundamental Rights
enforce these duties.
(d) Fundamental Duties
2. They are, correlative to legal duties.
3. Under the Constitution of India, which one of the
Select the correct answer using the code given below: following is not a fundamental duty?
(a) To vote in public elections
(a) 1 only (c) Both 1 and 2
(b) To develop the scientific temper
(b) 2 only (d) Neither 1 nor 2
(c) To safeguard public property
2. “To uphold and protect the Sovereignty, Unity and (d) To abide by me Constitution and respect its ideals
Integrity of India” is a provision made in the

********

7
POLITY WORKBOOK www.iasscore.in

SYSTEM OF GOVERNANCE

1. Consider the following statements: 2002, confession of the accused before the police
Statement-I: cannot be used as evidence.
The Supreme Court of India has held in some How many of the above statements are correct?
judgements that the reservation policies made under
Article 16(4) of the Constitution of India would be (a) Only one
limited by Article 335 for maintenance of efficiency of (b) Only two
administration.
(c) All three
Statement-II:
Article 335 of the Constitution of India defines the (d) None
term ‘efficiency of administration’.
4. If the president of India exercise his power as
Which one of the following is correct in respect of the
above statements? provided under article 356 of the constitution in
respect of a particular state, then
(a) Both Statement-I and Statement-II are correct
and Statement-II is the correct explanation for (a) The assembly of the state is automatically dissolved.
Statement-I
(b) The powers of the legislature of that state shall
(b) Both Statement-I and Statement-II are correct
be exercisable by or under the authority of the
and Statement-II is not the correct explanation
for Statement-I parliament.

(c) Statement-I is correct but Statement-II is incorrect (c) Article 19 is suspended in that state.
(d) Statement-I is incorrect but Statement-II is correct (d) The president can make laws relating to that state.

2. With reference to Finance Bill and Money Bill in 5. Which of the following are not necessarily
the Indian Parliament, consider the following
the consequences of the proclamation of the
statements:
President’s rule in a State?
1. When the Lok Sabha transmits Finance Bill to
the Rajya Sabha, it can amend or reject the Bill. 1. Dissolution of the State Legislative Assembly
2. When the Lok Sabha transmits Money Bill to the 2. Removal of the Council of Ministers in the State
Rajya Sabha, it cannot amend or reject the Bill, it
3. Dissolution of the local bodies
can only make recommendations.
3. In the case of disagreement between the Lok Select the correct answer using the code given below:
Sabha and the Rajya Sabha, there is not joint
(a) 1 and 2 only (c) 2 and 3 only
sitting for Money Bill, but a joint sitting becomes
necessary for Finance Bill. (b) 1 and 3 only (d) 1, 2 and 3
How many of the above statements are correct?
6. With reference to anti-defection law in India,
(a) Only one
consider the following statements:
(b) Only two
1. The law specifies that a nominated legislator
(c) All three
cannot join any political party within six months
(d) None of being appointed to the House.
3. Consider the following statements: 2. The law does not provide any time-frame within
1. According to the Constitution of India, the which the presiding officer has to decide a
Central Government has a duty to protect States defection case.
from internal disturbances.
Which of the statements given above is/are correct?
2. The Constitution of India exempts the States
from providing legal counsel to a person being (a) 1 only (c) Both 1 and 2
held for preventive detention.
(b) 2 only (d) Neither 1 nor 2
3. According to the Prevention of Terrorism Act,

8
www.iasscore.in POLITY WORKBOOK

UNION EXECUTIVE

1. Consider the following statements: 4. Consider the following statements:


1. If the election of the President of India is declared 1. The Executive Power of the Union of India is
void by the Supreme Court of India, all acts done vested in the Prime Minister.
by him/her in the performance of duties of his/ 2. The Prime Minister is the ex-officio Chairman of
her office of President before the date of decision the Civil Services Board.
become invalid.
Choose the Exp :
2. Election for the post of the President of India can
be postponed on the ground that some legislative (a) 1 only (c) Both 1 and 2
Assemblies have been dissolved and elections (b) 2 only (d) Neither 1 nor 2
are yet to take place.
3. When a Bill is presented to the President of India, 5. Which of the following is / are the function/
the Constitution prescribes time limits within functions of the Cabinet Secretariat?
which he/she has to declare his/her assent. 1. Preparation of agenda for Cabinet Meetings
How many of the above statements are correct? 2. Secretarial assistance to Cabinet Committees

(a) Only one 3. Allocation of financial resources to the Ministries


Select the correct answer using the code given below:
(b) Only two
(a) 1 only (c) 1 and 2 only
(c) All three
(b) 2 and 3 only (d) 1, 2 and 3
(d) None
6. Consider the following statements:
2. Consider the following statements in respect of
election to the President of India: 1. The Council of Ministers in the Centre shall be
collectively responsible to the Parliament.
1. The members nominated to either House of
the Parliament or the Legislative Assemblies 2. The Union Ministers shall hold the office during
of States are also eligible to be included in the the pleasure of the President of India.
Electoral College. 3. The Prime Minister shall communicate to the
2. Higher the number of elective Assembly seats, President about the proposals for Legislation.
higher is the value of vote of each MLA of that Choose the Exp :
State. (a) 1 only (c) 1 and 3 only
3. The value of vote of each MLA of Puducherry is
(b) 2 and 3 only (d) 1, 2 and 3
higher than that of Arunachal Pradesh because
the ratio of total population to total number
7. The Prime Minister of India, at the time of his/ her
of elective seats in Puducherry is greater as
appointment
compared to Arunachal Pradesh.
(a) need not necessarily be a member of one of the
How many of the above statements are correct? Houses of the Parliament but must become a
(a) Only one member of one of the Houses within six months
(b) Only two (b) need not necessarily be a member of one of the
Houses of the Parliament but must become a
(c) Only three
member of the Lok Sabha within six months
(d) All four
(c) must be a member of one of the Houses of the
parliament
3. With reference to the election of the president of
India, consider the following statements. (d) must be a member of the Lok Sabha
1. The value of the vote of each MLA varies from
8. According to the Constitution of India, it is the duty
state to state.
of the President of India to cause to be laid before
2. The value of the vote of MPs of the LokSabha is the Parliament which of the following?
more than the value of the vote of MPs of the
RajyaSabha. 1. The Recommendations of the Union Finance
Commission
Which of the statements given above is/are correct?
2. The Report of the Public Accounts Committee
(a) 1 only (c) Both 1 and 2
3. The Report of the Comptroller and Auditor
(b) 2 only (d) Neither 1 nor 2 General

9
POLITY WORKBOOK www.iasscore.in

4. The Report of the National Commission for the Which of the statements given above is/are correct?
Scheduled Castes (a) 1 only (c) Both 1 and 2
Select the correct answer the using the codes given (b) 2 only (d) Neither 1 nor 2
below:
(a) 1 only (c) 1, 3 and 4 only 10. Consider the following statements:

(b) 2 and 4 only (d) 1, 2, 3 and 4 1. Attorney General of India and Solicitor General
of India are the only officers of the Government
9. Consider the following statements: who are allowed to participate in the meetings of
the Parliament of India.
1. The Constitution of India classifies the ministers
2. According to the Constitution of India, the
into four ranks viz. Cabinet Minister, Minister of
Attorney General of India submits his resignation
State with Independent Charge, Minister of State
when the Government which appointed him
and Deputy Minister.
resigns.
2. The total number of ministers in the Union
Which of the statements given above is/are correct?
Government, including the Prime Minister, shall
not exceed 15 percent of the total number of (a) 1 only (c) Both 1 and 2
members in the Lok Sabha. (b) 2 only (d) Neither 1 nor 2

UNION LEGISLATURE

1. With reference to Home Guards, consider the 3. In India, which of the following review
following statements: the independent regulators in sectors like
1. Home Guards are raised under the Home Guards telecommunications, insurance, electricity, etc.?
Act and Rules of the Central Government. 1. Ad Hoc Committees set up by the Parliament
2. The role of the Home Guards is tow serve as an 2. Parliamentary Department Related Standing
auxiliary force to the police in maintenance of Committees
the internal security 3. Finance Commission
3. To present infiltration on the international 4. Financial Sector Legislative Reforms Commission
border/coastal areas, the Border Wings Home 5. NITI Aayog
Guards Battalions have been raised in some
States. Select the correct answer using the code given below:

How many of the above statements are correct? (a) 1 and 2 only (c) 3, 4 and 5 only
(b) 1, 3 and 4 only (d) 2 and 5 only
(a) Only one
(b) Only two 4. With reference to the Parliament of India which
(c) All three of the following Parliamentary Committees
scrutinizes and reports to the House whether
(d) None the powers to make regulations, rules, sub rules,
bylaws, etc. conferred by the Constitution or
2. Consider the following statements: delegated by the Parliament are being properly
1. The Parliament (Prevention of Disqualification) exercised by the Executive within the scope of such
Act, 1959 exempts several posts from delegation?
disqualification on the grounds of ‘Office of (a) Committee on Government Assurances
Profit’.
(b) Committee on Subordinate Legislation
2. The above-mentioned Act was amended five
times. (c) Rules Committee
3. The term ‘Office of Profit’ is well- defined in the (d) Business Advisory Committee
Constitution of India.
5. Consider the following statements:
Which of the statements given above is/are correct?
1. The Speakers of the Legislative Assembly shall
(a) 1 and 2 only vacate his/her office if he/she ceases to be a
(b) 3 only member of the Assembly
2. Whenever the legislative assembly is dissolved,
(c) 2 and 3 only
the speaker shall vacate his/her office
(d) 1, 2 and 3 immediately.

10
www.iasscore.in POLITY WORKBOOK

Which of the statements given above is/are correct? 10. For election to the Lok Sabha, a nomination paper
can be filed by
(a) 1 only (c) Both 1 and 2
(a) Anyone residing in India.
(b) 2 only (d) Neither 1 nor 2
(b) A resident of the constituency from which the
6. Regarding Money Bill, which of the following election is to be contested.
statements in not correct?
(c) Any citizen of India whose name appears in the
(a) A bill shall be deemed to be a Money Bill if it electoral roll of a constituency.
contains only provisions relating to imposition, (d) Any citizen of India.
abolition, remission, alteration or regulation of
any tax. 11. Out of the following statements, choose the one
(b) A Money Bill has provisions for the custody of that brings out the principle underlying the
the consolidated fund of India or the contingency “Cabinet form of Government:”
fund of India. (a) An arrangement for minimizing the criticism
(c) A Money Bill is concerned with the appropriation against the Government whose responsibilities
of moneys out of the contingency fund of India. are complex and hard to carry out to the
satisfaction of all.
(d) A Money Bill deals with the regulation of
borrowing of money or giving any guarantee by (b) A mechanism for speeding up the activities
the government of India. of the Government whose responsibilities are
increasing day by day.
7. Consider the following statements: (c) A mechanism of parliamentary democracy
1. In the first Lok Sabha, the single largest party in for ensuring collective responsibility of the
the opposition was the Swatantra Party. Government to the people.
2. In the Lok Sabha, a “Leader of the Opposition” (d) A device for strengthening the hands of the head
was recognised for the first time in 1969. of the Government whose hold over the people is
3. In the Lok Sabha, if a party does not have a in a state of decline.
minimum of 75 members, its leader cannot be
recognized as the Leader of the Opposition. 12. Consider the following statements:
Which of the statements given above is/are correct? 1. In the election for Lok Sabha or State Assembly,
the winning candidate must get at least 50
(a) 1 and 3 only (c) 2 and 3 only
percent of the votes polled, to be declared elected.
(b) 2 only (d) 1, 2 and 3
2. According to the provisions laid down in the
Constitution of India, in Lok Sabha, the Speaker’s
8. The Parliament of India exercises control over the
post goes to the majority party and the Deputy
functions of the Council of Ministers through
Speaker’s to the Opposition.
1. Adjournment motion
Which of the statements given above is/are correct?
2. Question hour
3. Supplementary questions (a) 1 only

Select the correct answer using the code given below: (b) 2 only

(a) 1 only (c) 1 and 3 only (c) Both 1 and 2

(b) 2 and 3 only (d) 2 and 3 only (d) Neither 1 nor 2

9. With reference to the Parliament of India, consider 13. Which of the following statements is/are correct?
the following statements: 1. A bill pending in the Lok Sabha lapses on its
1. A private member’s bill is a bill presented by a prorogation.
Member of Parliament who is not elected but 2. A bill pending in the Rajya Sabha, which has not
only nominated by the President of India. been passed by the Lok Sabha, shall not lapse on
2. Recently, a private member’s bill has been passed dissolution of the Lok Sabha.
in the Parliament of India for the first time in its Select the correct answer using the code given below:
history.
(a) 1 only
Which of the statements given above is/are correct?
(b) 2 only
(a) 1 only (c) Both 1 and 2
(c) Both 1 and 2
(b) 2 only (d) Neither 1 nor 2
(d) Neither 1 nor 2

11
POLITY WORKBOOK www.iasscore.in

14. The Parliament of India acquires the power to (a) 1 only (c) Both 1 and 2
legislate on any item in the State List in the national (b) 2 only (d) Neither 1 nor 2
interest if a resolution to that effect is passed by
the 19. What will follow if the money bill is substantially
(a) Lok Sabha by a simple majority of its total amended by the Rajya Sabha?
membership (a) The Lok Sabha may send the bill to the Rajya
(b) Lok Sabha by a majority of not less than two- Sabha for reconsideration
thirds of its total membership (b) The Lok Sabha may still proceed with the bill,
(c) Rajya Sabha by a simple majority of its total accepting or not accepting the recommendations
membership of the Rajya Sabha
(d) Rajya Sabha by a majority of not less than two- (c) The president may call a joint sitting for passing
thirds of its members present and voting the Bill
(d) The Lok Sabha cannot consider the bill further
15. Consider the following statements:
1. The Rajya Sabha has no power either to reject or 20. Consider the following statements:
to amend a Money Bill. 1. An amendment to the Constitution of India can
2. The Rajya Sabha cannot vote on the Demands for be initiated by an introduction of a bill in the Lok
Grants. Sabha only.
3. The Rajya Sabha cannot discuss the Annual 2. If such an amendment seeks to make changes
Financial Statement. in the federal character of the Constitution, the
Which of the statements given above is/are correct? amendment also requires to be ratified by the
legislature of all the States of India.
(a) 1 only (c) 2 and 3 only
Which of the statements given above is/are correct?
(b) 1 and 2 only (d) 1, 2 and 3
(a) 1 only (c) Both 1 and 2
16. When a bill is referred to a joint sitting of both the (b) 2 only (d) Neither 1 nor 2
Houses of the Parliament, it has to be passed by
(a) A simple majority of members present and voting 21. The Parliament can make any law for whole or
any part of India for implementing international
(b) Three-fourths majority of members present and treaties:
voting
(a) with the consent of all the States
(c) Two-thirds majority of the Houses
(b) with the consent of the majority of States
(d) Absolute majority of the Houses
(c) with the consent of the States concerned
17. With reference to the Union Government, consider (d) without the consent of any State
the following statements:
1. The Department of Revenue is responsible for the 22. Consider the following statements: The
preparation of Union Budget that is presented to Parliamentary Committee on Public Accounts
the Parliament. (PAC)
2. No amount can be withdrawn from the 1. Consists of not more than 25 Members of the Lok
Consolidated Fund of India without the Sabha
authorization from the Parliament of India. 2. Scrutinizes appropriation and finance accounts
3. All the disbursements made from Public Account of Government
also need the authorization from the Parliament 3. Examines the report of the Comptroller and
of India. Auditor General of India.
Which of the statements given above is/are correct? Which of the statements given above is / are correct?
(a) 1 and 2 only (c) 2 only (a) 1 only (c) 3 only
(b) 2 and 3 only (d) 1, 2 and 3 (b) 2 and 3 only (d) 1 2 and 3

18. Consider the following statements regarding a No- 23. Consider the following statements:
Confidence Motion in India: 1. The Chairman and the Deputy Chairman of the
1. There is no mention of a No-Confidence Motion Rajya Sabha are not the members of that House.
in the Constitution of India. 2. While the nominated members of the two
2. A Motion of No-Confidence can be introduced in Houses of the Parliament have no voting right in
the Lok Sabha only. the presidential election, they have the right to
Which of the statements given above is / are correct? vote in the election of the Vice President.

12
www.iasscore.in POLITY WORKBOOK

Which of the statements given above is/are correct? 4. A periodic or at least a mid-year review
(a) 1 only (c) Both 1 and 2 of programme of the Government against
macroeconomic forecasts and expenditure by a
(b) 2 only (d) Neither 1 nor 2 Parliamentary Budget Office.

24. In the Parliament of India, the purpose of an 5. Introducing Finance Bill in the Parliament
adjournment motion is Select the correct answer using the codes given below:
(a) To allow a discussion on a definite matter of (a) 1, 2, 3 and 5 only (c) 3, 4 and 5 only
urgent public importance
(b) 1, 2 and 4 only (d) 1, 2, 3, 4 and 5
(b) To let opposition members collect information
from the ministers 28. A deadlock between the Lok Sabha and Rajya Sabha
(c) To allow a reduction of specific amount in calls for a joint sitting of the parliament during the
demand for grant passage of:
(d) To postpone the proceedings to check the 1. Ordinary Legislation
inappropriate or violent behaviour on the part of 2. Money Bill
some members 3. Constitution Amendment Bill

25. Regarding the office of the Lok Sabha speaker Select the correct answer using the codes given below.
consider the following statements: (a) 1 only
1. She/he holds the office during the pleasure of the (b) 2 and 3 only
President
(c) 1 and 3 only
2. She/he need not be a member of the house at
the time of his/her election but has to become (d) 1, 2 and 3
a member of the House within six months from
the date of his/her election 29. What is the difference between “vote-on-account”
3. If he/she intends to resign the letter of his/her and “interim budget”?
resignation has to be addressed to the Deputy 1. The provision of a “vote-on-account” is used by a
Speaker. Regular Government, while an “interim budget”
Which of the above statements given above is/are is a provision used by a caretaker Government.
correct? 2. A “vote-on-account” only deals with the
(a) 1 and 2 only (c) 1, 2 and 3 expenditure in Government’s budget, while an
“internal budget” includes both expenditure and
(b) 3 only (d) None receipts.
26. Which of the following special powers have been Which of the statements given above is/are correct?
conferred on the Rajya Sabha by the Constitution (a) 1 only
of India?
(b) 2 only
(a) To change the existing territory of a State and to
change the name of a State. (c) Both 1 and 2
(b) To pass a resolution empowering the Parliament (d) Neither 1 nor 2
to make laws in the State List and to create one or
more All India Services 30. The authorization for the withdrawal of funds
from the Consolidated Fund of India must come
(c) To amend the election procedure of the President
from
and to determine the pension of the President
after his/her retirement (a) The President of India
(d) To determine the functions of the Election (b) The Parliament of India
Commission and to determine the number of (c) The Prime Minister of India
Election Commissioners.
(d) The Union Finance Minister
27. Which of the following are the methods of
Parliamentary control over public finance in 31. All revenues received by the Union. Government
India? by way of taxes and other receipts for the conduct
1. Placing Annual Financial Statement before the of Government business are credited to the:
Parliament (a) Contingency Fund of India
2. Withdrawal of moneys from Consolidated Fund of (b) Public Account
India only after passing the Appropriation Bill
(c) Consolidated Fund of India
3. Provisions of supplementary grants and vote-on-
account (d) Deposits and Advances Fund

13
POLITY WORKBOOK www.iasscore.in

32. When the annual Union Budget is not passed by the contests in one Lok Sabha election from many
Lok Sabha constituencies, his/her party should bear the cost
of bye-elections to the constituencies vacated by
(a) The Budget is modified and presented again
him/her in the event of him/her winning in all
(b) The Budget is referred to the RajyaSabha for the constituencies.
suggestions
Which of the statements given above is/are correct?
(c) The Union Finance Minister is asked to resign,
(a) 1 only (c) 1 and 3 only
(d) The Prime Minister submits the resignation of
(b) 2 only (d) 2 and 3 only
Council of Ministers
38. Consider the following statements:
33. Which one of the following is responsible for the
preparation and presentation of Union Budget to 1. In India, there is no law restricting the candidates
the Parliament? from contesting in one Lok Sabha election from
three constituencies.
(a) Department of Financial Services
2. In 1991 Lok Sabha Election, Shri Devi Lal
(b) Department of Economic Affairs contested from three Lok Sabha constituencies.
(c) Department of Expenditure 3. As per the existing rules, if a candidate
(d) Department of Revenue contests in one Lok Sabha election from many
constituencies, his/her party should bear the cost
34. Raja Sabha has equal powers with Lok Sabha in of bye-elections to the constituencies vacated by
him/her in the event of him/her winning in all
(a) the matter of creating new All India Services
the constituencies.
(b) amending the Constitution
Which of the statements given above is/are correct?
(c) the removal of the government
(a) 1 only (c) 1 and 3 only
(d) making cut motions
(b) 2 only (d) 2 and 3 only
35. Along with the Budget, the Finance Minister also
39. Which of the following is/are the exclusive power(s)
places other documents before the Parliament
which include ‘The Macro Economic Framework of Lok Sabha?
Statement’. The aforesaid document is presented 1. To ratify the declaration of Emergency
because this is mandated by 2. To pass a motion of no-confidence against the
(a) Long standing parliamentary convention Council of Ministers
(b) Article 112 and Article 110(1) of the Constitution 3. To impeach the President of India
of India Select the correct answer using the code given below:
(c) Article 113 of the Constitution of India (a) 1 and 2 only (c) 1 and 3 only
(d) Provisions of the Fiscal Responsibility and Budget (b) 2 only (d) 3 only
Management Act, 2003
40. With reference to Deputy Speaker of Lok Sabha,
36. Consider the following statements: consider the following statements:
1. The President of India can summon a session of 1. As per the Rules of Procedure and Conduct of
the Parliament at such place as he/she thinks fit. Business in Lok Sabha, the election of Deputy
2. The Constitution of India provides for three Speaker shall be held on such date as the Speaker
sessions of the Parliament in a year, but it is not may fix.
mandatory to conduct all three sessions. 2. There is a mandatory provision that the election
3. There is no minimum number of days that the of a candidate as Deputy Speaker of Lok Sabha
Parliament is required to meet in a year. shall be from either the principal opposition
Which of the statements given above is/are correct? party or the ruling party.
3. The Deputy Speaker has the same power as of
(a) 1 only (c) 1 and 3 only
the Speaker when presiding over the sitting of
(b) 2 only (d) 2 and 3 only the House and no appeal lies against his rulings.
4. The well established parliamentary practice
37. Consider the following statements: regarding the appointment of Deputy Speaker
1. In India, there is no law restricting the candidates is that the motion is moved by the Speaker and
from contesting in one Lok Sabha election from duly seconded by the Prime Minister.
three constituencies.
Which of the statements given above are correct?
2. In 1991 Lok Sabha Election, Shri Devi Lal
contested from three Lok Sabha constituencies. (a) 1 and 3 only (c) 3 and 4 only
3. As per the existing rules, if a candidate (b) 1, 2 and 3 (d) 2 and 4 only

14
www.iasscore.in POLITY WORKBOOK

STATE EXECUTIVE

1. Which one of the following suggested that the 4. Which of the following are the discretionary
Governor should be an eminent person from powers given to the Governor of a State?
outside the State and should be a detached figure
1. Sending a report to the President of India for
without intense political links or should not have
imposing the President’s rule
taken part in politics in the recent past?
2. Appointing the Ministers
(a) First Administrative Reforms Commission (1966)
3. Reserving certain bills passed by the State
(b) Rajamannar Committee (1969)
Legislature for consideration of the President of
(c) Sarkaria Commission (1983) India
(d) National Commission to Review the Working of 4. Making the rules to conduct the business of the
the Constitution (2000) State Government

2. Consider the following statements: Select the correct answer using the code given below:

1. No criminal proceedings shall be instituted (a) 1 and 2 only (c) 2, 3 and 4 only
against the Governor of a State in any court (b) 1 and 3 only (d) 1, 2, 3 and 4
during his term of office.
2. The emoluments and allowance of the Governor 5. Which one of the following statements is correct?
of a State shall not be diminished during his
(a) In India, the same person cannot be appointed
term of office.
as Governor for two or more States at the same
Which of the statements given above is/are correct? time
(a) Only 1 (c) Both 1 and 2 (b) The Judges of the High Court of the States in India
(b) Only 2 (d) Neither 1 nor 2 are appointed by the Governor of the State just as
the Judges of the Supreme Court are appointed by
3. Consider the following statements: the President
1. The Chief Secretary in a State is appointed by the
(c) No procedure has been laid down in the
Governor of that State.
Constitution of India for the removal of a Governor
2. The Chief Secretary in a State has a fixed tenure from his/her post
Which of the statements given above is/are correct? (d) In the case of a Union Territory having a legislative
(a) Only 1 (c) Both 1 and 2 setup, the Chief Minister is appointed by the Lt.
(b) Only 2 (d) Neither 1 nor 2 Governor on the basis of majority support

STATE LEGISLATURE

1. Consider the following statements: above statements?


Statement-I: (a) Both Statement-I and Statement-II are correct
In India, prisons are managed by State Governments and Statement-II is the correct explanation for
with their own rules and regulations for the day-to- Statement-I
day administration of prisons.
(b) Both Statement-I and Statement-II are correct
Statement-II: and Statement-II is not the correct explanation
In India, prisons are governed by the Prisons Act, 1894 for Statement-I
which expressly kept the subject of prisons in the (c) Statement-I is correct but Statement-II is incorrect
control of Provincial Governments.
(d) Statement-I is incorrect but Statement-II is correct
Which one of the following is correct in respect of the

15
POLITY WORKBOOK www.iasscore.in

2. With reference to the Legislative Assembly of a Which of the statements given above is/are correct?
State in India, consider the following statements:
(a) Only 1 (c) Both 1 and 2
1. The governor makes a customary address to
(b) Only 2 (d) Neither 1 nor 2
members of the house at the commencement of
the first session of the year.
4. Consider the following statements:
2. When a State Legislature does not have a rule on
1. According to the Constitution of India, a person
a particular matter, it follows the Lok Sabha rule
who is eligible to vote can be made a minister
on that matter.
in a State for six months even if he/she is not a
Which of the statements given above is/are correct? member of the Legislature of that State.
(a) 1 only (c) Both 1 and 2 2. According to the Representation of People Act,
(b) 2 only (d) Neither 1 nor 2 1951, a person convicted of a criminal offence
and sentenced to imprisonment for five years
3. Consider the following statements: is permanently disqualified from contesting an
1. The Legislative Council of a State in India can election even after his release from prison.
be larger in size than half of the Legislative
Which of the statements given above is/are correct?
Assembly of that particular State.
(a) 1 only (c) 2 only
2. The Governor of a State nominates the Chairman
of Legislative Council of that particular State. (b) Both 1 and 2 (d) Neither 1 nor 2

LOCAL GOVERNMENT

1. Local self-government can be best explained as an Select the correct answer using the code given below:
exercise in
(a) Only 1 (c) Both 1 and 2
(a) Federalism
(b) Only 2 (d) Neither 1 nor 2
(b) Democratic decentralization
(c) Administrative delegation 4. The fundamental object of Panchayati Raj system
is to ensure which among the following?
(d) Direct democracy
1. People’s participation in development
2. Consider the following statements: 2. Political accountability
1. The minimum age prescribed for any person to 3. Democratic decentralization
be a member of Panchayat is 25 years.
4. Financial mobilization
2. A Panchayat reconstituted after premature
dissolution continues only for the remainder Select the correct answer using the code given below:
period. (a) 1, 2 and 3 only (c) 1 and 3 only
Which of the statements given above is/are correct? (b) 2 and 4 only (d) 1, 2, 3 and 4
(a) Only 1 (c) Both 1 and 2
5. The Government enacted the Panchayat Extension
(b) Only 2 (d) Neither 1 nor 2
to Scheduled Areas (PESA) Act in 1996. Which one
3. With reference to the ‘Gram Nyayalaya Act’, which of the following is not identified as its objective?
of the following statements is/are correct? (a) To provide self-governance
1. As per the Act, Gram Nyayalayas can hear only
(b) To recognize traditional rights
civil cases and not criminal cases
(c) To create autonomous regions in tribal areas
2. The Act allows local social activists as mediators
/ reconciliators. (d) To free tribal people from exploitation

16
www.iasscore.in POLITY WORKBOOK

6. The Constitution (Seventy-Third Amendment) Act, panchayat elections.


1992, which aims at promoting the Panchayati Raj 3. Establishment of State Finance Commissions.
Institutions in the country, provides for which of
the following? Select the correct answer using the codes given below:

1. Constitution of District Planning Committees. (a) Only 1 (c) 2 and 3 only


2. State Election Commissions to conduct all (b) 1 and 2 only (d) 1, 2 and 3

UNION TERRITORIES AND SPECIAL AREAS

1. With reference to ‘Scheduled Areas’ in India, 3. Consider the following statements:


consider the following statements: 1. Union territories are not presented in the Rajya
1. Within a State, the notification of an area as Sabah
Scheduled Area takes place through an Order of 2. It is within the purview of the Chief Election
the President. Commissioner to adjudicate the election disputes
2. The largest administrative unit forming the 3. According to the constitution of India, parliament
Scheduled Area is the District and the lowest is consists of Lok Sabha and Rajya Sabah only
the cluster of villages in the Block.
Which of the statements given above is/are correct?
3. The Chief Ministers of the concerned States
are required to submit annual reports to the (a) Only 1 (c) 1 and 3 only
Union Home Ministry on the administration of (b) 2 and 3 (d) None
Scheduled Areas in the States.
4. If a particular area is brought under the Fifth
How many of the above statements are correct?
Schedule of the Constitution of India, which
(a) Only one (c) All three one of the following statements best reflects the
(b) Only two (d) None consequence of it?
(a) This would prevent the transfer of land of tribal
2. The provisions in Fifth Schedule and Sixth Schedule people to non-tribal people.
in the Constitution of India are made in order to
(b) This would create a local self-governing body in
(a) protect the interests of Scheduled Tribes that area.
(b) determine the boundaries between States (c) This would convert that area into a Union
(c) determine the powers, authority and Territory.
responsibilities of Panchayats (d) The State having such areas would be declared a
(d) protect the interests of all the border States Special Category State.

THE JUDICIARY

1. Consider the following statements: 2. Consider the following statements:


1. The 44 Amendment to the Constitution of India
th 1. The motion to impeach a Judge of the Supreme
introduced an Article placing the election of the Court of India cannot be rejected by the Speaker
Prime Minister beyond judicial review. of the Lok Sabha as per the Judges (Inquiry) Act,
1968.
2. The Supreme Court of India struck down the 99th
2. The Constitution of India defines and gives
Amendment to the Constitution of India as being
details of what constitutes ‘incapacity and
violative of the independence of judiciary.
proved misbehaviour’ of the Judges of the
Which of the statements given above is/are correct? Supreme Court of India.
(a) 1 only (c) Both 1 and 2 3. The details of the process of impeachment of the
Judges of the Supreme Court of India are given in
(b) 2 only (d) Neither 1 nor 2 the Judges (Inquiry) Act, 1968.

17
POLITY WORKBOOK www.iasscore.in

4. If the motion for the impeachment of a Judge is Select the correct answer using the codes given below:
taken up for voting, the law requires the motion
(a) 1 and 2 (c) 1 and 4
to be backed by each House of the Parliament
and supported by a majority of total membership (b) 2 and 3 (d) 3 and 4
of that House and by not less than two-thirds of
total members of that House present and voting. 8. What is the provision to safeguard the autonomy
Which of the statements given above is/are correct? of the supreme court of India?
(a) 1 and 2 (c) 3 and 4 only 1. While appointing the Supreme Court judges, the
president of India has to consult the CJI.
(b) 3 only (d) 1, 3 and 4
2. The SC judges can be removed by the CJI only
3. With reference to the Constitution of India, 3. The salaries of judges are charged on the
consider the following statements: consolidated fund of India to which the
1. No High Court shall have the jurisdiction to legislature does not have to vote.
declare any central law to be constitutionally
4. All appointments of officers and staffs of the SC
invalid.
are made by the govt only after CJI
2. An amendment to the Constitution of India
cannot be called into question by the Supreme Choose the Correct options:
Court of India. (a) 1 and 3 only (c) Only 4
Which of the statements given above is/are correct?
(b) 3 and 4 only (d) 1, 2, 3 and 4
(a) 1 only (c) Both 1 and 2
(b) 2 only (d) Neither 1 nor 2 9. Consider the following statements :
The Supreme Court of India tenders advice to the
4. Who/Which of the following is the custodian of the President of India on matters of law or fact
Constitution of India?
1. On its own initiative (on any matter of larger
(a) The President of India public interest).
(b) The Prime Minister of India 2. If he seeks such an advice.
(c) The Lok Sabha Secretariat 3. Only if the matters relate to the Fundamental
(d) The Supreme Court of India Rights of the citizens.
Choose the correct answer using the codes given
5. The power of the Supreme Court of India to decide
disputes between the Centre and the States falls below:
under its (a) Only 1 (c) 3 only
(a) Advisory jurisdiction (b) Only 2 (d) 1 and 2 only
(b) Appellate jurisdiction
10. With reference to Lok Adalats, which of the
(c) Original jurisdiction
following statements is correct?
(d) writ jurisdiction
(a) Lok Adalats have the jurisdiction to settle the
6. The power to increase the number of judges in the matters at pre-litigative stage and not those
Supreme Court of India is vested in matters pending before any court
(a) The President of India (b) Lok Adalats can deal with matters which are civil
(b) The Parliament and not criminal in nature

(c) The Chief Justice of India (c) Every Lok Adalat consists of either serving or
retired judicial officers only and not any other
(d) The Law Commission
person
7. Which of the following are included in the original (d) None of the statements given above is correct
jurisdiction of the Supreme Court?
1. Dispute between the Government of India and 11. In India, separation of judiciary from the executive
one or more States is enjoined by
2. A dispute regarding elections to either House of (a) the Preamble of the Constitution
the parliament or that of Legislature of a State
(b) a Directive Principle of State Policy
3. A dispute between the Government of India and
Union Territory (c) the Seventh Schedule
4. A dispute between two or more States. (d) the conventional practice

18
www.iasscore.in POLITY WORKBOOK

12. With reference to Indian judiciary, consider the Which of the statements given above is/are correct?
following statements:
(a) 1 and 2 only (c) 3 and 4 only
1. Any retired judge of the Supreme Court of India
can be called back to sit and act as a Supreme (b) 1, 2 and 4 only (d) 3 only
Court judge by the Chief Justice of India with
prior permission of the President of India. 15. With reference to India, consider the following
2. A High Court in India has the power to review its statements:
own judgement as the Supreme Court does. 1. Government law officers and legal firms are
Which of the statements given above is/are correct? recognized as advocates, but corporate lawyers
and patent attorneys are excluded from
(a) 1 only (c) Both 1 and 2
recognition as advocates.
(b) 2 only (d) Neither 1 nor 2 2. Bar Councils have the power to lay down the
rules relating to legal education and recognition
13. With reference to Indian judiciary, consider the
following statements: of law colleges.

1. Any retired judge of the Supreme Court of India Which of the statements given above is/are correct?
can be called back to sit and act as a Supreme (a) 1 only (c) Both 1 and 2
Court judge by the Chief Justice of India with
prior permission of the President of India. (b) 2 only (d) Neither 1 nor 2
2. A High Court in India has the power to review its
own judgement as the Supreme Court does. 16. With reference to the writs issued by the Courts in
India, consider the following statements:
Which of the statements given above is/are correct?
1. Mandamus will not lie against a private
(a) 1 only (c) Both 1 and 2 organization unless it is entrusted with a public
(b) 2 only (d) Neither 1 nor 2 duty.
2. Mandamus will not lie against a Company even
14. Consider the following statements:
though it may be a Government Company.
1. Pursuant to the report of H.N. Sanyal Committee,
the Contempt of Courts Act, 1971 was passed. 3. Any public minded person can be a petitioner
to move the Court to obtain the writ of Quo
2. The Constitution of India empowers the Supreme
Warranto.
Court and the High Courts to punish for contempt
of themselves. Which of the statements given above are correct?
3. The Constitution of India defines Civil Contempt (a) 1 and 2 only (c) 1 and 3 only
and Criminal Contempt.
(b) 2 and 3 only (d) 1, 2 and 3
4. In India, the Parliament is vested with the
powers to make laws on Contempt of Court.

CONSTITUTIONAL AND NON-CONSTITUTIONAL BODIES

1. Consider the following organization/bodies in 2. With reference to India, consider the following
India: pairs:
1. The National Commission for Backward Classes
Action The Act under
2. The National Human Rights Commission
which it is covered
3. The National Law Commission
4. The National Consumer Disputes Redressal Authorized wearing of The official Secrets
Commission police and military uniforms Act, 1923

How many of the above are constitutional bodies? Knowingly misleading or The Indian Evidence
otherwise interfering with Act, 1872
(a) Only one
a police officer or military
(b) Only two officer when engaged in
(c) Only three their duties
(d) All four

19
POLITY WORKBOOK www.iasscore.in

public finances.
Celebratory gunfire which The Arms
can endanger the personal (Amendment) Act, 4. While dealing with audit and accounting of govt.
safety of others 2019 companies. CAG has certain judicial powers for
prosecuting those who violate the law.
How many of the above pairs are correctly matched?
Which of the above statements is/are correct?
(a) Only one
(a) 1, 3 and 4 only (c) 2 and 3 only
(b) Only two
(b) 2 only (d) 1, 2, 3 and 4
(c) All three
(d) None 6. With reference to the Finance Commission of India,
which of the following statements is correct?
3. Consider the following statements: (a) It encourages the inflow of foreign capital for
1. The Election Commission of India is a five- infrastructure development
member body.
(b) It facilitates the proper distribution of finances
2. Union Ministry of Home Affairs decides the among the Public Sector Undertakings
election schedule for the conduct of both general
(c) It ensures transparency in financial
elections and bye-elections.
administration
3. Election Commission resolves the disputes
relating to splits/mergers of recognized political (d) None of the statements (a), (b) and (c) given above
parties. is correct in this context

Which of the statements given above is/are correct? 7. Who of the following shall cause every
(a) 1 and 2 only (c) 2 and 3 only recommendations made by the Finance Commission
to be laid before each House of Parliament?
(b) 2 only (d) 3 only
(a) The President of India
4. The Government of India has established NITI (b) The Speaker of Lok Sabha
Aayog to replace the :
(c) The Prime Minister of India
(a) Human Rights Commission
(d) The Union Finance Minister
(b) Finance Commission
(c) Law Commission 8. With reference to the “Tea Board” in India,
consider the following statements:
(d) Planning Commission
1. The Tea Board is a statutory body.
5. What is the importance of the office of the CAG? 2. It is a regulatory body attached to the Ministry of
1. CAG exercises exchequer control on behalf of the Agriculture and Farmers Welfare.
parliament when the president of India declares 3. The Tea Board’s Head Office is situated in
national emergency/ financial emergency Bengaluru.
2. CAG reports on the execution of projects or 4. The Board has overseas offices at Dubai and
programmes by the ministries are discussed by Moscow.
the PAC/
Which of the statements given above are correct?
3. Information form CAG reports can be used by
(a) 1 and 3 only (c) 3 and 4 only
investigating agencies to press charges against
those who have violated the law while managing (b) 2 and 4 only (d) 1 and 4 only

GOVERNANCE & IR

1. Consider the following statements: How many of the above statements are correct?
1. The Government of India provides Minimum (a) Only one
Support Price for Niger (Guizotia abyssinica)
(b) Only two
seeds.
2. Niger is cultivated as a Kharif crop. (c) All three
3. Some tribal people in India use Niger seed oil for (d) None
cooking.

20
www.iasscore.in POLITY WORKBOOK

2. Consider the following statements in the context (a) Both statement-I and Statement-II are correct
of intervention being undertaken under Anaemia and Statement-II is the correct explanation for
Mukt Bharat Strategy: Statement-I
1. It provides prophylactic calcium supplementation (b) Both Statement-I and Statement-II are correct
for pre-school children, adolescents and and Statement-II is not the correct explanation
pregnant women. for Statement-I
2. It runs a campaign for delayed cord clamping at (c) Statement-I is correct but Statement-II is incorrect.
the time of child birth.
(d) Statement-I- is incorrect but Statement-II is
3. It provides for periodic deworming to children
correct.
and adolescents.
4. It addresses non-nutritional causes of anemia in 6. Consider the following statements:
endemic pockets with special focus on malaria,
1. Recently, all the countries of the United Nations
hemoglobinopathies and fluorosis.
have adopted the first-ever compact for
How many of the statements given above are correct? international migration, the ‘Global Compact for
(a) Only one (c) All three Safe, Orderly and Regular Migration (GCM)’.
2. The objective and commitments stated in the
(b) Only two (d) None
GCM are binding on the UN member countries.
3. Consider the following statements in relation to 3. The GCM address internal migration or
Janani Suraksha Yojana: internally displaced people also in its objectives
1. It is a safe motherhood intervention of the State and commitments.
Health Departments. How many of the above statements are correct?
2. Its objective is reduce maternal and neonatal (a) Only one (c) All three
mortality among poor pregnant women.
(b) Only two (d) None
3. It aims to promote institutional delivery among
poor pregnant women. 7. Consider the following statements:
4. Its objective includes providing public health
Statement-I:
facilities to sick infants up one year of age.
Recently, the United States of America (USA) and the
How many of the statements given above are correct?
European Union (EU) have launched the Trade and
(a) Only one (c) All three technology council.
(b) Only two (d) None Statement-II:

4. Consider the following statements about G-20: The USA and the EU claim that through this they are
trying to bring technological progress and physical
1. The G-20 group was originally established as productivity under their control.
platform for the Finance Ministers and Central
Bank Governors to discuss the international Which one of the following is correct in respect of the
economic and financial issues. above statements?
2. Digital public infrastructure is one of India’s (a) Both statement-I and Statement-II are correct
G-20 priorities. and Statement-II is the correct explanation for
Statement-I
Which of the statements given above is/are correct?
(b) Both Statement-I and Statement-II are correct
(a) 1 only
and Statement-II is not the correct explanation
(b) 2 only for Statement-I
(c) Both 1 and 2 (c) Statement-I is correct but Statement-II is incorrect.
(d) Neither 1 nor 2 (d) Statement-I- is incorrect but Statement-II is
correct.
5. Consider the following statements:
Statement-I: 8. Consider the following statements:
Israel has established diplomatic relations with some 1. Aadhaar card can be used as a proof of citizenship
Arab States. or domicile.
2. Once issued, Aadhaar number cannot be
Statement-II:
deactivated or omitted by the Issuing Authority.
The ‘Arab Peach Initiative’ mediated by Saudi Arabia
Which of the statements given above is/are correct?
was signed by Israel and Arab League.
(a) 1 only (c) Both 1 and 2
Which one of the following is correct in respect of the
above statements? (b) 2 only (d) Neither 1 nor 2

21
POLITY WORKBOOK www.iasscore.in

9. Consider the following statements: Which of the statements given above is/ are correct?
1. The food Safety and Standard Act, 2006 (a) 1 only (c) Both 1 and 2
replaced the Prevention of Food Adulteration
Act, 195 (b) 2 only (d) Neither 1 nor 2
2. The Food Safety and Standards Authority of
14. With reference to the ‘Prohibition of Benami
India (FSSAI) is under the charge of Director
Property Transactions Act, 1988 (PBPT Act)’,
General of Health Services in the Union Ministry
consider the following statements:
of Health and Family Welfare.
1. A property transaction is not treated as a benami
Which of the statements given above is/are correct?
transaction if the owner of the property is not
(a) 1 only (c) Both 1 and 2 aware of the transaction.
(b) 2 only (d) Neither 1 nor 2 2. Properties held benami are liable for confiscation
by the Government
10. India enacted The Geographical indicates of Goods 3. The Act provides for three authorities for
(Registration and Protection) Act, 1999 in order to
investigations but does not provide for any
comply with the obligations to
appellate mechanism.
(a) ILO (c) UNCTAD
Which of the statements given above is/are correct?
(b) IMF (d) WTO
(a) 1 only (c) 1 and 3 only
11. Consider the following statements: (b) 2 only (d) 2 and 3 only
1. As per the Right to Education (RTE) Act, to
be eligible for appointment as a teacher in 15. With reference to ‘National Skills Qualification
a State, a person would be required to posses Framework (NSQF)’, which of the statements given
the minimum qualification laid down by the below is/are correct?
concerned State council of Teacher Education. 1. Under NSQF, a learner can acquire the
2. As per the RTE Act, for teaching primary classes, a certification for competency only through
candidate is required to pass a Teacher Eligibility formal learning.
Test conducted in accordance with the National
2. An outcome expected from the implementation
Council of Teacher Education Guidelines.
of NSQF is the mobility between vocational and
3. In India, more than 90% of teacher education general education:
institutions are directly under the State
Governments. Select the correct answer using the code given below:
Which of the statements given above is/are correct? (a) 1 only (c) Both 1 and 2
(a) 1 and 2 only (c) 1 and 3 (b) 2 only (d) Neither 1 nor 2
(b) 2 only (d) 3 only
16. Consider the following in respect of ‘National
Career Service’ :
12. Which of the following is/are the aim/aims of
“Digital India” plan of the Government of India? 1. National Career Service is an initiative of
1. Formation of India’s own Internet companies the Department of Personnel and Training,
like china did. Government of India.
2. Established a policy framework to encourage 2. National Career Service has been launched in
overseas multinational corporations that collect a Mission Mode to improve the employment
big data to build their large data centers within opportunities to uneducated youth of the
our national geographical boundaries. country.
3. Connect many of our villages to the internet and Which of the above statements is/are correct?
bring WiFi to many of our schools, public places
and major tourist centers. (a) 1 only (c) Both 1 and 2

Select the correct answer using the code given below: (b) 2 only (d) Neither 1 nor 2

(a) 1 and 2 only (c) 2 and 3 only 17. Which of the following are the objectives of
(b) 3 only (d) 1, 2 and 3 ‘National Nutrition Mission’?
1. To create awareness relating to malnutrition
13. With reference to digital payments, consider the among pregnant women and lactating mothers.
following statements :
2. To reduce the incidence of anaemia among
1. BHIM app allows the user to transfer money to young children, adolescent girls and women.
anyone with a UPI-enabled bank account.
3. To promote the consumption of millets, coarse
2. While a chip-pin debit card has four factors of
cereals and unpolished rice.
authentication, BHIM app has only two factors
of authentication. 4. To promote the consumption of poultry eggs.

22
www.iasscore.in POLITY WORKBOOK

Select the correct answer using the code given below: 22. What is/are the purpose of ‘District Mineral
Foundations” in India?
(a) 1 and 2 only (c) 1, 2 and 4 only
1. Promoting mineral explorative activities in
(b) 1, 2 and 3 only (d) 3 and 4 only mineral-rich districts
18. What is the purpose of Vidyanjali Yojana’? 2. Protecting the interests of the persons affected
by mining operations
1. To enable the famous foreign campuses in India.
3. Authorizing State Governments to issue licenses
2. To increase the quality of education provided for mineral exploration
in government schools by taking help from the
private sector and the community. Select the correct answer using the code given below:
3. To encourage voluntary monetary contributions (a) 1 and 2 only (c) 1 and 3 only
from private individuals and organizations so (b) Only 2 (d) 1, 2 and 3
as to improve the infrastructure facilities for
primary and secondary schools. 23. ‘Rashtriya Garima Abhiyaan’ is a national
Select the correct answer using the code given below: campaign to
(a) 2 only (c) 1 and 2 only (a) Rehabilitate the homeless and destitute persons
and provide then with suitable sources of
(b) 3 only (d) 2 and 3 only livelihood
19. What is the aim of the programme ‘Unnat Bharat (b) Release the sex workers from the practice
Abhiyan’? and provide them with alternative sources of
livelihood
(a) Achieving 100% literacy by promoting
collaboration between voluntary organizations (c) Eradicate the practice of manual scavenging and
and government’s education system and local rehabilitate the manual scavengers
communities. (d) Release the bonded labourers free their bondage
(b) Connecting institutions of higher education and rehabilitate them
with local communities to address development
challenges through appropriate techhologies. 24. With reference to pre-packaged items in India, it
is mandatory to the manufacturer to put which
(c) Strengthening India’s scientific regearch of the following information on the main label, as
institutions in order to make India a scientific and per the Food Safety and Standards (Packaging and
technological Power. Labeling) Regulations, 2011?
(d) Developing human capital by allocating special 1. List of ingredients including additives
funds for health- care and education of rural and 2. Nutrition information
urban poor, and organizing skill development
3. Recommendations, if any made by the medical
programmes and vocational training for them.
profession about the possibility of any allergic
reactions
20. Which of the following is a most likely consequence
of implementing the ‘Unified Payments Interface 4. Vegetarian/non-vegetarian
(UPI)’? Select the correct answer using the code given below
(a) Mobile wallets will not be necessary for online (a) 1, 2 and 3 (c) 1, 2 and 4
payments.
(b) 2, 3 and 4 (d) 1 and 4 Only
(b) Digital currency will totally replace the physical
currency in about two decades. 25. Mission Indradhanush’ launched by the
(c) FDI inflows will drastically increase. Government of India pertains to

(d) Direct transfer of subsidies to poor people will (a) Immunization of children and pregnant women
become very effective. (b) Construction of smart cities across the country
(c) India’ own search for the Earth-like planets in
21. Regarding DigiLocker, sometimes seen in the news,
outer space
which of the following statements is/are correct?
(d) New Educational policy
1. It is a digital locker system offered by the
Government under Digital India Programme.
26. With reference to ‘Initiative for Nutritional
2. It allows you to access your e-documents Security through intensive Millets Promotion’,
irrespective of your physical location. which of the following statements is/are correct?
Select the correct answer using the code given below. 1. This initiative aims to demonstrate the improved
(a) 1 only (c) Both 1 and 2 production and post-harvest technologies, and
to demonstrate value addition techniques, in an
(b) 2 only (d) Neither 1 nor 2 integrated manner, with cluster approach.

23
POLITY WORKBOOK www.iasscore.in

2. Poor, small, marginal and tribal farmers have Select the correct answer using the codes given below:
larger stake in this scheme. (a) 1 and 2 only (c) 1 and 3 only
3. An important objective of the scheme is to
(b) Only 2 (d) 1, 2 and 3
encourage farmers of commercial crops to shift
to millet cultivation by offering them free kits of
31. Who among the following constitute the National
critical inputs of nutrients and micro-irrigation
Development Council?
equipment.
1. The Prime Minister
Select the correct answer using the code given below.
2. The Chairman, Finance Commission
(a) 1 only (c) 1 and 2 only
3. Ministers of the Union Cabinet
(b) 2 and 3 only (d) 1, 2 and 3
4. Chief Ministers of the States
27. Consider the following pairs : Select the correct answer using the codes given below:
Programme/ Project Ministry (a) 1, 2 and 3 only (c) 2 and 4 only
1. Drought-Prone : Ministry of Agriculture (b) 1, 3 and 4 only (d) 1, 2, 3 and 4
Area Programme
2. Desert Development : Ministry of 32. Consider the following statements:
Programme Environment and 1. National Development Council is an organ of the
Forests Planning Commission.
3. National Watershed : Ministry of Rural 2. The Economic and Social Planning is kept in the
Development Development Concurrent List in the Constitution of India.
Which of the above pairs are correct: 3. The Constitution of India prescribes that
(a) 1 and 2 only (c) 1, 2 and 3 Panchayats should be assigned the task of
preparation of plans for economic development
(b) Only 3 (d) None and social justice.
28. What are the benefits of implementing the Which of the statements given above is/are correct?
‘Integrated Watershed Development Programme’? (a) 1 only (c) 1 and 3 only
1. Prevention of soil runoff
(b) 2 and 3 only (d) 1, 2 and 3
2. Linking the country’s perennial rivers with
seasonal rivers 33. With reference to National Legal Services
3. Rainwater harvesting and recharge of Authority, consider the following statements:
groundwater table 1. Its objective is to provide free and competent
4. Regeneration of natural vegetation legal services to the weaker sections of the
society on the basis of equal opportunity .
Select the correct answer using the code given below:
2. It issues guidelines for the State Legal Services
(a) 1 and 2 only (c) 1, 3 and 4 only
Authorities to implement the legal programs and
(b) 2, 3 and 4 only (d) 1, 2, 3 and 4 schemes throughout the country.
Which of the statements given above is / are correct?
29. Which of the following are associated with
‘Planning’ in India? (a) 1 only (c) Both 1 and 2
1. The Finance Commission (b) 2 only (d) Neither 1 nor 2
2. The National Development Council
3. The Union Ministry of Rural Development 34. In the areas covered under the Panchayat
(Extension to the Scheduled Areas) Act, 1996, what
4. The Union Ministry of Urban Development is the role/power of Gram Sabha?
5. The Parliament
1. Gram Sabha has the power to prevent alienation
Select the correct answer using the code given below: of land in the Scheduled Areas.
(a) 1, 2 and 5 only (c) 2 and 5 only 2. Gram Sabha has the ownership of minor forest
produce.
(b) 1, 3 and 4 only (d) 1, 2, 3, 4 and 5
3. Recommendation of Gram Sabha is required for
30. Which of the following bodies does not/do not find granting prospecting license or mining lease for
mention in the Constitution? any mineral in the Scheduled Areas.
1. National Development Council Which of the statements given above is/are correct?
2. Planning Commission (a) Only 1 (c) 2 and 3 only
3. Zonal Councils (b) 1 and 2 only (d) 1, 2 and 3

24
www.iasscore.in POLITY WORKBOOK

35. The National Green Tribunal Act, 2010 was 4. Selling/offloading the shares of Public Sector
enacted in consonance with which of the following Undertakings
provisions of the Constitution of India?
Which of the above can be used as measures to control
1. Right to healthy environment, construed as a the fiscal deficit in India?
part of Right to life under Article 2
(a) 1, 2 and 3 only (c) 1, 2 and 4 only
2. Provision of grants for raising the level of
administration in the Scheduled Areas for the (b) 2, 3 and 4 only (d) 3 and 4 only
welfare of Scheduled Tribes under Article 275(1).
3. Powers and functions of Gram Sabha as 40. Consider the following statements:
mentioned under Article 243(A). 1. Aadhaar metadata cannot be stored for more
Select the correct answer using the codes given below: than three months.

(a) 1 only (c) 1 and 3 only 2. State cannot enter into any contract with private
corporations for sharing of Aadhaar data.
(b) 2 and 3 only (d) 1, 2 and 3
3. Aadhaar is mandatory for obtaining insurance
36. With reference to consumer’s rights / privileges products.
under the provision of law in India which of the 4. Aadhaar is mandatory for getting benefits funded
following statements correct? out of the Consolidated Fund of India.
1. Consumer are empowered to take samples for Which of the statements given above is/are correct?
food testing
(a) 1 and 4 only (c) 3 only
2. When consumer files a complaint in any
consumer forum, no fee is required to be paid. (b) 2 and 4 only (d) 1, 2 and 3 only
3. In case of death of consumer, his/her legal heir
can file a complaint in the consumer forum on 41. With reference to the funds under Members of
his/her behalf. Parliament Local Area Development Scheme
(MPLADS), which of the following statements are
Select the correct answer using the codes given below:
correct?
(a) Only 1 (c) 1 and 3 only 1. MPLADS funds must be used to create durable
(b) 2 and 3 only (d) 1, 2 and 3 assets like physical infrastructure for health,
education, etc.
37. Consider the following:
2. A specified portion of each MP’s fund must
1. Right to education. benefit SC/ST populations.
2. Right to equal access to public service. 3. MPLADS funds are sanctioned on yearly basis
3. Right to food. and the unused funds cannot be carried forward
Which of the above is/are Human Right/Human Rights to the next year.
under “Universal Declaration of Human Rights”? 4. The district authority must inspect at least 10%
(a) 1 and 2 only (c) 1, 2 and 3 of all works under implementation every year.

(b) Only 1 (d) Only 3 Select the correct answer using the code given below:
(a) 1 and 2 only (c) 1, 2 and 3 only
38. With reference to the Constitution of India,
consider the following: (b) 3 and 4 only (d) 1, 2 and 4 only
1. Fundamental rights 42. In India, Legal Services Authorities provide free
2. Fundamental Duties legal services to which of the following type of
3. Directive Principles of State Policy citizens?
Which of the above provisions of the Constitution of 1. Person with an annual income of less than Rs.
India is/are fulfilled by the National Social Assistance 1,00,000
Programme launched by the government of India? 2. Transgender with an annual income of less than
(a) Only 1 (c) 1 and 3 only Rs. 2,00,000
(b) Only 3 (d) 1, 2 and 3 3. Member of Other Backward Classes (OBC) with
an annual income of less than Rs. 3,00,000
39. In the context of governance, consider the 4. All Senior Citizens
following:
Select the correct answer using the code given below:
1. Encouraging Foreign Direct Investment inflows
(a) 1 and 2 only (c) 2 and 3 only
2. Privatization of higher educational Institutions
3. Down-sizing of bureaucracy (b) 3 and 4 only (d) 1 and 4 only

25
POLITY WORKBOOK www.iasscore.in

43. Consider the following pairs: 2. In 1970, the Department of Personnel was
International Subject constituted on the recommendation of the
agreement/set-up Administrative Reforms Commission, 1966,
and this was placed under the Prime Minister’s
1. Alma-Ata Declaration — Healthcare of the
charge.
people
2. Hague Convention — Biological and Which of the statements given above is/are correct?
chemical weapon (a) 1 only (c) Both 1 and 2
3. Talanoa Convention — Global climate change (b) 2 only (d) Neither 1 nor 2
4. Under2 Coalition — Child rights
Which of the pairs given above is/are correctly 48. At the national level, which ministry is nodal
matched? agency to ensure effective implementation of the
Scheduled Tribes and Other Traditional Forest
(a) 1 and 2 only (c) 1 and 3 only Dwellers (Recognition of Forest Rights) Act, 2006?
(b) 4 only (d) 2, 3 and 4 only (a) Ministry of Environment, Forest and Climate
Change
44. In the context of India, which one of the following
is the characteristic appropriate for bureaucracy? (b) Ministry of Panchayati Raj
(a) An agency for widening the scope of parliamentary (c) Ministry of Rural Development
democracy (d) Ministry of Tribal Affairs
(b) An agency for strengthening the structure of
federalism 49. With reference to Ayushman Bharat Digital
(c) An agency for facilitating political stability and Mission, consider the following statements:
economic growth 1. Private and public hospitals must adopt it.
(d) An agency for the implementation of public 2. As it aims to achieve universal health coverage,
policy every citizen of India should be part of it
ultimately.
45. With reference to the Union Government, consider 3. It has seamless portability across the country.
the following statements:
Which of the statements given above is/are correct?
1. N. Gopalaswamy Iyengar Committee suggested
that a minister and a secretary be designated (a) 1 and 2 only (c) 1 and 3 only
solely for pursuing the subject of administrative (b) 3 only (d) 1, 2 and 3
reform and promoting it.
2. In 1970, the Department of Personnel was 50. Consider the following :
constituted on the recommendation of the
1. Aarogya Setu
Administrative Reforms Commission, 1966,
and this was placed under the Prime Minister’s 2. CoWIN
charge. 3. DigiLocker
Which of the statements given above is/are correct? 4. DIKSHA
(a) 1 only (c) Both 1 and 2 Which of the above are built on top of open-source
(b) 2 only (d) Neither 1 nor 2 digital platforms?
(a) 1 and 2 only (c) 1, 3 and 4 only
46. At the national level, which ministry is nodal
agency to ensure effective implementation of the (b) 2, 3 and 4 only (d) 1, 2, 3 and 4
Scheduled Tribes and Other Traditional Forest
Dwellers (Recognition of Forest Rights) Act, 2006? 51. Consider the following statements:
(a) Ministry of Environment, Forest and Climate 1. The India Sanitation Coalition is a platform to
Change promote sustainable sanitation and is funded by
the Government of India and the World Health
(b) Ministry of Panchayati Raj Organization.
(c) Ministry of Rural Development 2. The National Institute of Urban Affairs is an
(d) Ministry of Tribal Affairs apex body of the Ministry of Housing and Urban
Affairs ‘in Government of India and provides
47. With reference to the Union Government, consider innovative solutions to address the challenges of
the following statements: Urban India.
1. N. Gopalaswamy Iyengar Committee suggested Which of the statements given above is/are correct?
that a minister and a secretary be designated
solely for pursuing the subject of administrative (a) 1 only (c) Both 1 and 2
reform and promoting it. (b) 2 only (d) Neither 1 nor 2

26
www.iasscore.in POLITY WORKBOOK

52. With reference to the “United Nations Credentials status to the non-member States.
Committee”, consider the following statements: 2. Inter-governmental organisations can seek
1. It is a committee set up by the UN Security observer status in the UN General Assembly.
Council and works under its supervision.
3. Permanent Observers in the UN General
2. It traditionally meets in March, June and Assembly can maintain missions at the UN
September every year.
headquarters.
3. It assesses the credentials of all UN members
before submitting a report to the General Which of the statements given above are correct ?
Assembly for approval. (a) 1 and 2 only (c) 1 and 3 only
Which of the statements given above is/are correct? (b) 2 and 3 only (d) 1, 2 and 3
(a) 3 only (c) 2 and 3
55. With reference to the United Nations Convention on
(b) 1 and 3 (d) 1 and 2
the Law of Sea, consider the following statements:
53. Which one of the following statements best 1. A coastal state has the right to establish the
describes the ‘Polar Code’? breadth of its territorial sea up to a limit not
(a) It is the international code of safety for ships exceeding 12 nautical miles, measured from
operating in polar waters. baseline determined in accordance with the
convention.
(b) It is the agreement of the countries around the
North Pole regarding the demarcation of their 2. Ships of all states, whether coastal or land-locked,
territories in the polar region. enjoy the right of innocent passage through the
(c) It is a set of norms to be followed by the countries territorial sea.
whose scientists undertake research studies in 3. The Exclusive Economic Zone shall not extend
the North Pole and South Pole. beyond 200 nautical miles from the baseline
from which the breadth of the territorial sea in
(d) It is a trade and security agreement of the member
measure.
countries of the Arctic Council.
Which of the statements given above are correct?
54. With reference to the United Nations General (a) 1 and 2 only (c) 1 and 3 only
Assembly, consider the following statements:
(b) 2 and 3 only (d) 1, 2 and 3
1. The UN General Assembly can grant observer

MISCELLANEOUS

1. Consider the following statements in respect of the 2. Arjuna Award - For the lifetime achievement by
44th Chess Olympiad, 2022 a sportperson
1. It was the first time that Chess Olympaid was 3. Dronocharya Award - To honour eminent
held in India. coaches who have successfully trained
sportsperson or teams
2. The official mascot was named ‘Thambi’.
4. Rashtriya Khel Protsahan Puraskar- To recognize
3. The trophy for the winning team in the open
the contribution made by sportspersons even
section is the Vera Menchik Cukp.
after their retirement
4. The trophy for the winning team in the women’s
How many of the above pairs are correctly matched?
section is the Hamilton-Russell Cup.
(a) Only one (c) All three
How many of the statements given above are correct?
(b) Only two (d) All four
(a) Only one (c) Only three
(b) Only two (d) All four 3. Consider the following statements in respect of
Bharat Ratna and Padma Awards:
2. Consider the following pairs with regard to sports 1. Bharat Ratna and Padma Awards are titles under
awards. the Article 18(1) of the Constitution of India.
1. Major Dhyan Chand Khel Ratna Award 2. Padma Awards, which were instituted in the
- For the most spectacular and outstanding year 1954, were suspended only once.
performance by a sportsperson over period of 3. The number of Bharat Ratna Awards is restricted
last four years to a maximum of five in a particular year.

27
POLITY WORKBOOK www.iasscore.in

Which of the above statements are not correct? winner of this award.
(a) 1 and 2 only (c) 1 and 3 only 2. The award was received mostly by ‘Formula
One’ players so far.
(b) 2 and 3 only (d) 1, 2 and 3
3. Roger Federer received this award maximum
4. Consider the following statements in respect of the number of times compared to others.
ICC World Test Championship: Which of the above statements are correct?
1. The finalists were decided by the number of (a) 1 and 2 only (c) 1 and 3 only
matches they won.
(b) 2 and 3 only (d) 1, 2 and 3
2. New Zealand was ranked ahead of England
because it won more matches than England. 6. Consider the following statements in respect of the
32nd Summer Olympics
Which of the above statements is/are correct?
1. The official motto for this Olympics is ‘A New
(a) 1only (c) Both 1 and 2 World’.
(b) 2 only (d) Neither 1 nor 2 2. Sport Climbing, Surfing, Skateboarding, Karate
and Baseball are included in this Olympics.
5. Consider the following statements in respect of the
Which of the above statements is/are correct?
Laureus World Sports Award which was instituted
in the year 2000: (a) 1 only (c) Both 1 and 2
1. American golfer Tiger Woods was the first (b) 2 only (d) Neither 1 nor 2

**********

28
POLITY & GOVERNANCE
ANSWERS

EVOLUTION OF THE CONSTITUTION

1. Exp. (b) Schedule IV to the Constitution of India.


 The 42nd Constitutional Amendment Act 3. Exp: (b)
was enacted to overcome the judgement of
 According to constitution at the time of its
KeshavandaBharati Case. commencement on 26th January 1950, India
 This amendment resulted in judicial review was a Sovereign, Democratic, Republic. It got
restricted the power of review provided to the constitutional status of being Socialist and Secular
Judiciary by Constitution of India. after 42nd amendment act 1976.

2. Exp : (b) 4. Exp: (d)

 The directive principles are like instruments of  The core element of constitutional government is,
instructions which were issued to the Governor the existence of a “Rule-of Law” or set of “basic
General and Governors of colonies of India by the laws” that binds both public office-holders and
British Government under the 1935 Act under the all members of a society (i.e. citizens) within a
Draft Constitution. given territory. Presently most states avail of a
constitution, which directs the organization of
 It was proposed to issue such instructions to the state, the relations between the public offices
the president and governors. The text of these within the state, as well the human and civil rights
instruments of the instructions is found in of the individual.

SALIENT FEATURES OF INDIAN CONSTITUTION

1. Exp, (c)  In India the adoption of the principle of Due


Process of Law was adopted in the Menaka Gandhi
 The chief objective of the Constitution is to
establish a limited government i.e. a government case.
which does not have the right to encroach in all 3. Exp. (a)
spheres of a citizen’s life.
 Ninth Schedule was introduced by 1st constitutional
2. Exp. (a) amendment and this was under reign of Nehru.
 Due process refers to just, rational, fair, and fair
treatment under the regular judicial process. It 4. Exp. (b)
essentially means due process of law needs the Fifth Schedule
legislation to follow the principle of natural justice.
 The key objective is to provide protection to
 The Principle of Due Process of Law was originally the tribals living in the Scheduled Areas from
adopted by the American Constitution. Its founding alienation of their lands and natural resources to
fathers stated that the United States Constitution non-tribals.
guarantees that the government cannot take away
a person’s basic rights to “life, liberty or property,  In these areas, the Governor has been given
without due process of law.” For the purpose plenary powers as far as their administration is
of safeguarding these rights the Constitution of concerned.
United States provided power to Judiciary to look  Under this schedule the transfer of tribal land to
into fairness of law. private parties for mining be declared null and void

29
POLITY WORKBOOK www.iasscore.in

 The executive power of the Union extends to (validation of certain Acts and Regulations) and is
the giving of directions to the State as to the not subject to judicial scrutiny.
administration of these areas.  The Ninth Schedule (Article 31-B) was introduced
 Governor of these states need make report to the by the former Prime Minister Jawaharlal Nehru
President annually or as needed by President to keep certain laws particularly those on land
regarding the administration of the Scheduled reforms beyond the scope of judicial review.
Areas in that State.  The mandate of ninth schedule is to prevent judicial
scrutiny but in a landmark ruling in IR Coelho
5. Exp. (d) versus State of Tamil Nadu, 2007, the Supreme
 Option (a) and (b) entail negative conception of Court of India ruled that all laws (including those
liberty and have minimalist overtone. Option (c) in the Ninth Schedule) would be open to Judicial
Review if they violated the basic structure of the
is related to anarchy and lawlessness. Option (D)
constitution. The Supreme Court judgment laid
provides for positive conception of liberty that our
that the laws placed under Ninth Schedule after
constitution provides for. Provision of reservation,
April 24, 1973. Shall be open to challenge in court
protection of minority rights etc. provide for if they violated fundamental rights guaranteed
creating an enabling atmosphere where individual under Article 14, 19, 20 and 21 of the Constitution
can develop themselves fully.
 The court also said that by using the power to
amend the Constitution, Parliament cannot alter
6. Exp. (b) its basic or essential features like federal structure,
 “One of the important instances of application separation of power between the three organs of
by the Supreme Court of Article 142 was in the the states and judicial review, among other things.
Union Carbide case — relating to the victims of the
Bhopal gas tragedy — where the Court felt a need 9. Exp :(b)
to deviate from existing law to bring relief to the  Economic liberty is not embodied in the preamble
thousands of persons affected by the gas leak. In to the constitution of India.
this judgment, the Supreme Court, while awarding
compensation of $470 million to the victims, went 10. Exp : (a)
to the extent of saying that to do complete justice, it  Democracy is a form of government in which the
could even override the laws made by Parliament rulers are elected by the people. In a democracy,
by holding that, “prohibitions or limitations or the final decision-making power must rest with
provisions contained in ordinary laws cannot, those elected by the people. Democracy must
ipso facto, act as prohibitions or limitations on be based on a free and fair election where those
the constitutional powers under Article 142.” By currently in power have a fair chance of losing.
this statement the Supreme Court of India placed
itself above the laws made by Parliament or the 11. Exp : (c)
legislatures of the States.” Parliamentary form of government
 The democratic system of government can be
7. Exp :(b)
divided into the parliamentary and the presidential
 Option (b) is a appropriate choice because it was system based on the relationship between the
a statement made by John Locke. He, in Second executive and the legislature. In a parliamentary
Treatise of Civil Government, wrote: “where system, executive is a part of legislature, which
there is no law, there is no freedom” In 1689, he implements the law and plays an active role in
also wrote that “the end of law is not to abolish or framing it as well.
restrain, but to preserve and enlarge freedom.” No  The main advantage of the parliamentary form
law would give LICENSCE (Unregulated liberty). of government is that the executive remains
Therefore, B is correct. responsible to the legislature.
 But, then such statements are not to be considered
objective realities. If there are no laws, there can be 12. Exp : (a)
either complete liberty, because nothing restricts  The mind of the makers of the Constitution of India
the actions of citizens, or NO liberty because then is reflected in the Preamble.
lawlessness can heavily curtail liberty. It really
depends. It is an extreme statement to say that 13. Exp : (a)
when there are no laws, there is no liberty (Can we
say there is no liberty in a Jungle?) There can be  One of the implications of equality in society is the
liberty despite the absence of laws. absence of privileges.
Equality has three Basic elements:
8. Exp :(a)  Absence of special privileges in society.
 Once a law is enacted and included in the Ninth  Presence of adequate and equal opportunities for
Schedule, it gets protection under Article 31-B development of all.

30
www.iasscore.in POLITY WORKBOOK

 Equal satisfaction of basic needs of all 18. Exp : (d)


Anti-Defection Law
14. Exp : (c)
 The 52nd amendment, 1985 to the Constitution
 Rights are reasonable claims of a person which are
added the Tenth Schedule, which laid down the
recognized by society and sanctioned by law.
process by which legislators may be disqualified
 This means that an unreasonable claim of a person on grounds of defection. A Member of Parliament
cannot become a right. Moreover, definition and or state legislature was deemed to have defected
scope of rights change from one society to another if he either voluntarily resigned from his party or
and from a period to different period. disobeyed the directives of the party leadership on
a vote. That is, they may not vote on any issue in
15. Exp : (d) contravention to the party’s whip. Independent
Federalism in India members would be disqualified if they joined a
 Indian model of federalism is called quasi-federal political party.
system as it contains major features of both a  The law also made a few exceptions. Any person
federation and union. Indian federation was not elected as speaker or chairman could resign from
a product of coming together of states to form the his party, and rejoin the party if he demitted that
federal union of India. It was rather a conversion post. A party could be merged into another if at
of a unitary system into a federal system. least two-thirds of its party legislators voted for
Federal Features of the India Union the merger. The law initially permitted splitting of
parties, but that has now been outlawed.
 Two governments i.e. Union Government and
State governments 19. Exp : (b)
 Division of powers between the union and its  ‘Economic Justice’ as one of the objectives of the
constituents (Seventh Schedule of the Constitution
Indian Constitution has been provided in the
contains three lists such as the Union List, State
Preamble and the Directive Principles of State
List, and Concurrent List)
Policy
 Supremacy of the Constitution (Basic structure
of the Constitution is made indestructible by the 20. Exp : (d)
Judiciary)
 Constitutional government by definition is a
 Partial rigidity of the Constitution limited government.
 Independent Judiciary
21. Exp : (d)
 Bicameralism
 Statement 1 is incorrect: Basic Structure has
16. Exp : (c) nowhere been defined in the constitution of Indian.
It is a Judicial Innovation in 1976 in Keshvanand
 Right to vote and to be elected in India is a
Bharti case.
Constitutional Right
 Some of the features of the Constitution termed as
 Constitution of India Article 325: No person to
“basic” are listed below:
be ineligible for inclusion in, or to claim to be
included in a special, electoral roll on grounds of  Supremacy of the Constitution
religion, race, caste or sex .
 Rule of law
17. Exp : (d)  The principle of Separation of Powers
Parliamentary system of Government:  The objectives specified in the Preamble to the
 The Parliamentary system ensures a better Constitution of India
representation of the populace and well suited for  Judicial Review
countries with huge diversity.
 Articles 32 and 226
 There is a Parliamentary System of Government
 Federalism (including financial liberty of states
in India because the Council of Ministers is
under Articles 282 and 293)
responsible to the Lok Sabha
 Secularism
 India did not just copy the British Constitution,
instead the Constituent Assembly took inspirations  The Sovereign, Democratic, Republican structure
from various constitutions and modified them  Freedom and dignity of the individual
as per India’s own requirements. Comparing a
 Unity and integrity of the Nation
monarch with a president is a basic mistake for
the simple reason that the former is hereditary  The principle of equality, not every feature of
while the latter is elected. equality, but the quintessence of equal justice;

31
POLITY WORKBOOK www.iasscore.in

 The “essence” of other Fundamental Rights in Part the Parliament are referred to the Constitution
III Bench by the Supreme Court. A Constitution
 The concept of social and economic justice — to Bench is a bench of the Supreme Court having
build a Welfare State: Part IV in toto five or more judges on it. These benches are not
a routine phenomenon. A vast majority of cases
 The balance between Fundamental Rights and before the Supreme Court are heard and decided
Directive Principles by a bench of two judges (called a Division Bench),
 The Parliamentary system of government and sometimes of three. Constitution Benches
are set up when the case involves a substantial
 The principle of free and fair elections
question of law pertaining to the interpretation of
 Limitations upon the amending power conferred the Constitution (Article 145(3) of the Constitution,
by Article 368 which mandates that such matters be heard by a
 Independence of the Judiciary bench of not less than five judges).
 Effective access to justice  Presently, Constitution Benches are set up on
an ad hoc basis as and when the need arises.
 Powers of the Supreme Court under Articles 32,
136, 141, 142 The idea behind a Constitution Bench is clear: it
is constituted in rare cases to decide important
 Legislation seeking to nullify the awards made questions of fact or legal and/or constitutional
in exercise of the judicial power of the State by interpretation.
Arbitration Tribunals constituted under an Act.
24. Exp: (a)
Statement 2 is incorrect:
 It is nowhere mentioned in constitution that Key features of federalism:
‘Judicial review’ is to preserve the ideals on which  There are two or more levels (or tiers) of
the Constitution is based. government.
 Judicial Review is the power of Courts to pronounce  Different tiers of government govern the same
upon the constitutionality of legislative and citizens, but each tier has its own jurisdiction
executive acts of the government which fall within in specific matters of legislation, taxation and
their normal jurisdiction. Judicial review plays administration.
very important role as protector for safeguarding
 The jurisdictions of the respective levels or tiers
the rights of people.
of government are specified in the constitution.
So the existence and authority of each tier of
22. Exp : (a)
government is constitutionally guaranteed.
 Kesavananda Bharati Case: In this case, for the first
 The fundamental provisions of the constitution
time, a bench of 13 judges was assembled to hear
cannot be unilaterally changed by one level of
a writ petition. The Court held that: The Preamble
of the Constitution will now be considered as part government. Such changes require the consent of
of the Constitution but won’t be having any legal both the levels of government.
effect.  Courts have the power to interpret the constitution
and the powers of different levels of government.
23. Exp: (c) The highest court acts as an umpire if disputes
 Statement 1 is correct: Parliamentary sovereignty arise between different levels of government in
is a principle of the UK constitution. It the exercise of their respective powers. The most
makes Parliament the supreme legal authority in important feature of the federal system adopted
the UK, which can create or end any law. Generally, by the Indian Constitution is the principle that
the courts cannot overrule its legislation and no relations between the States and the centre would
Parliament can pass laws that future Parliaments be based on cooperation. And for this, Independent
cannot change. Judiciary is the prerequisite. Hence, Option (a) is
 Indian Parliament is not a sovereign body like correct.
the British Parliament. The Indian Parliament  Sources of revenue for each level of government are
may, in exercise of its constituent power, amend clearly specified to ensure its financial autonomy.
by way of addition, variation or repeal any
provision of the Constitution in accordance 25. Exp: (a)
with the procedure laid down for the purpose.  Statement 1 is incorrect: The true position was
However, the Parliament cannot amend those what was elucidated by the Supreme Court in
provisions which form the ‘basic structure’ of the D.P. Joshi v. State of Madhya Bharat, What was
Constitution. This was ruled by the Supreme Court explained in that case was that the concept of an
in the Kesavananda Bharati case (1973). Indian domicile does not do away with the concept
 Statement 2 is correct: In India, matters related to of subsidiary domiciles such as the domicile of
the constitutionality of an amendment of an act of the States and that there may be a domicile of a

32
www.iasscore.in POLITY WORKBOOK

State for certain purposes notwithstanding there Parliament the supreme legal authority in the UK,
is the larger and the more comprehensive Indian which can create or end any law. Parliament of
domicile. India is a creation of the Constitution and hence
 Statement 3 is correct: The Citizenship Act offers for that reason itself, it has its limitations too.
other categories of citizenship like Citizenship  Statement 2 is incorrect: The Constitution
by Registration (Section 5) and Citizenship by (Forty-second Amendment) Act, 1976, inserted
Naturalisation (Section 6). These are basically for various articles in the Constitution to curtail,
foreigners who wish to settle in India and seek both directly and indirectly, the jurisdiction of the
Indian citizenship or persons of Indian origin Supreme Court and the High Courts to review the
living abroad who want to return to India and constitutionality of laws.
live as citizens in this country. It says that if the
registration or certificate of naturalization was 27. Exp. (b)
obtained by “means of fraud, false representation  Statement 1 is incorrect: No prior approval of
or the concealment of a material fact; or that citizen the President is required for the Constitutional
has shown himself by act or speech to be disloyal Amendment Act.
or disaffected towards the Constitution of India as
by law established; or that citizen has, during any  Statement 2 is correct: It is obligatory for the
war in which India may be engaged, unlawfully President to give his assent, when a Constitutional
traded or communicated with an enemy; or that Amendment Bill is presented before him. The
citizen has been ordinarily resident out of India President cannot exercise any veto power
for a continuous period of seven years”, that regarding the Constitutional Amendment Bill.
person’s citizenship can be cancelled.  Statement 3 is correct: Constitutional
Amendment Bill needs to be passed by both the
26. Exp: (a) houses separately by a special majority, and no
 Statement 1 is correct: Parliamentary sovereignty joint sitting is allowed regarding Constitutional
is a principle of the UK constitution. It makes Amendment Bill.

UNION & ITS TERRITORY

1. Exp : (c) seats in a Legislative Assembly may also change.


What is Delimitation?  The orders of the Delimitation Commission cannot
 Delimitation is the act of redrawing boundaries of be challenged in a Court of Law.
Lok Sabha and state Assembly seats to represent  When the orders of the Delimitation Commission
changes in population. are laid before the Lok Sabha or State Legislative
 In this process, the number of seats allocated to Assembly, they cannot effect any modifications in
different states in Lok Sabha and the total number the orders.

FUNDAMENTAL RIGHTS

1. Exp. (b) suffer in body or goods except for distinct breach


of law and no man is above the law. The term Rule
 “The right to marry a person of one’s choice is of Law thus, means the paramountcy of Law over
integral to Article 21 (right to life and liberty) of Government.
the Constitution”, the Supreme Court said Monday
 The main features of the “Rule of Law” are
and set aside a 2017 order of the Kerala High Court
which annulled the marriage of Kerala Muslim Limitation of powers , Equality before law and
convert girl Hadiya and Shefin Jahan. Liberty and civil rights:
 Three principles proposed by A.V. Dicey:-
2. Exp : (c)
 Absolute supremacy of Law.
Rule of law:
 Equality before law.
 The Rule of Law according to Dicey means that
no man is punishable or can be lawfully made to  Predominance of legal spirit.

33
POLITY WORKBOOK www.iasscore.in

3. Exp : (c) law “inconsistent with or in derogation of the


What is right to Privacy? fundamental rights” as void.

 Right to Privacy refers to respecting and ensuring  Under Article 13, the term ‘law’ includes any
the privacy of the individual. It is not explicitly “Ordinance, order, bye-law, rule, regulation,
mentioned in the Constitution. The nine-judge notification, custom or usage” having the force of
Constitution Bench of the Supreme Court in a law in India.
landmark unanimous decision has declared  Examples of Judicial Review: The striking down
right to privacy a fundamental right under the of the Section 66A of the IT Act as it was against
constitution overruling SC’s past 2 judgements in the Fundamental Rights guaranteed by the
MP Sharma case (1954) and Kharak Singh case
constitution.
(1961) .
 Article 21 and the freedoms guaranteed in Part III 7. Exp : (b)
implies that the right to privacy is protected as an
 In the Constitution of India, promotion of
intrinsic part of right to Life and personal liberty.
international peace and security is included in the
4. Exp : (a) Directive Principles of State Policy.

 Rights are correlative with Duties. 8. Exp : (c)

5. Exp : (c)  Education provisions the Constitution of India


consists of following provisions which have a
 Exploitation means the misuse of services
bearing on education:
rendered by others with the help of force. The
practice of exploitation violates the basic concept  Fundamental Rights [Part III]
of the Indian Constitution, the Preamble and
 Directive Principles of State Policy [Part IV]
opposes the Directive Principle of State Policy
given under Article 39 of the Indian Constitution  Fundamental Duties [Part IV A]
which stimulates economic equality among the
individuals.  Rural (Panchayati Raj) & Urban (Municipal)
Local Bodies [Part IX & Part IX A]
 Article 23 - Prohibition of ‘Traffic in Human Beings’
and Forced Labour  Scheduled & Tribal Areas [Part X]
 Article 23 of the Indian Constitution expressly  Centre-State Relations [Part XI]
prohibits human trafficking, forced labour and
other similar activities. It also states that any 9. Exp : (c)
violation of this provision will be considered as an
offence and the person acting in contravention of  To preserve the rich heritage of our composite
the law will be penalized in accordance with the culture, To develop the scientific temper and
law. spirit of inquiry, To strive towards excellence in
all spheres of individual and collective activity
6. Exp : (a) are among the fundamental Duties of citizens laid
down in the Indian Constitution.
 The power of the Judiciary to pronounce upon the
constitutionality of laws and executive orders
10. Exp : (d)
Judicial Review
 Though legislature has the power to make laws, 11. Exp : (d)
this power is not absolute. Judicial Review is the  Right to equality incorporates protection against
process by which the Judiciary review the validity untouchability as a form of discrimination Under
of laws passed by the legislature. Article 17
 The power of judicial review is evoked to protect
12. Exp: (c)
and enforce the fundamental rights guaranteed in
Part III of the Constitution.  Right to privacy is protected as an intrinsic part of
the right to life and personal liberty under Article
 Article 13 of the Constitution prohibits the
21 and as a part of the freedoms guaranteed by
Parliament and the state legislatures from
Part III of the Constitution.
making laws that “may take away or abridge the
fundamental rights” guaranteed to the citizens of 13. Exp: (c)
the country.
 Statement 1 is correct: The right to the city is
 The provisions of Article 13 ensure the protection at the heart of the New Urban Agenda agreed at
of the fundamental rights and consider any Habitat III. UN-Habitat partners with national

34
www.iasscore.in POLITY WORKBOOK

and local governments, academia, civil society  States do have separate prison/parole rules because
and the private sector in the implementation and “prison and persons detained” is a state subject of
monitoring of the commitment made under the the 7th schedule.
New Urban Agenda and SDG 11.
16. Exp: (a)
 The Universal Declaration of Human Rights
commits to social justice through the promotion,  The court exercises control over delegation of
defense and fulfillment of all human rights related discretionary powers to the administration by
to habitat, including the Human Right to Adequate adjudicating upon the constitutionality of the
Housing, Land and the Right to the City in every law under which such powers are delegated with
reference to the fundamental rights enunciated
region of the world.
in Part III of the Indian Constitution. Therefore,
 There is also a “World Charter for the Right to the if the law confers vague and wide discretionary
City” which provides a progressive framework to power on any administrative authority, it may be
rethink cities and urbanization. declared ultra vires Article 14, Article 19 and other
 Statement 2 is correct: The Right to the City is provisions of the Constitution.
the right of all inhabitants (present and future,  Article 14 of the Constitution guarantees equality
permanent and temporary) to inhabit, use, occupy, before law but the courts have permitted reasonable
produce, transform, govern and enjoy cities, towns classification to be made. Where the law is valid
and human settlements that are just, inclusive, safe, under the article, a discriminatory action would
sustainable and democratic, defined as common still be violative of the equality clause.
goods for enjoying life with dignity and peace. The  Hence, option (a) is correct.
right to the city further implies responsibilities
on governments and people to claim, defend, and  Also, by elimination, Art. 28, and Art.44 are
promote this right. unrelated to the question and Art. 44 is to protect
the FR of the citizens.
 Statement 3 is incorrect: The ‘Right to the City’ does
not mean the guarantee of any public service or 17. Exp: (b)
facility to the unauthorized colonies in a city. It  In political science, the term “State” has a more
guarantees the common goods and services which specific and definite meaning- “word State means
are essential for the quality of life. a community or society politically organized under
14. Exp: (b) one independent government within a definite
territory.
 Statement 1 is incorrect: Police Custody means
 It alone has the prerogative of making laws. The
that police have the physical custody of the
law-making power derives from sovereignty,
accused while Judicial Custody means an accused
which is the most distinctive characteristic of the
is in the custody of the concerned Magistrate. In
State.
the former, the accused is lodged in a police station
lockup while in latter, it is the jail.  According to Maclver, “a State is the fundamental
association for the maintenance and development
 When Police take a person into custody, the
of social order, and to this end, its central
Cr.P.C kicks-in and they were produced before a
institution is endowed with the united power of
Magistrate within 24 hours of the arrest.
the community”.
 Police Custody with permission to interrogate:
During Judicial Custody, the police officer in charge 18. Exp: (b)
of the case is not allowed to interrogate the suspect.  The Forty Fourth Constitutional Amendment, 1978,
However, the court may allow the interrogations deleted Articles 19(1)(f) and 31 from Part III, the
to be conducted if it opines the interrogation being chapter on Fundamental Rights in the Constitution.
necessary under the facts produced before the Instead, it inserted Article 300A in a new chapter
court. IV of Part XII of the Constitution, thereby depriving
15. Exp: (b) the ‘right to property’ of its ‘fundamental right’
status. Article 300A directs that - Persons not to be
 Statement 1 is incorrect: The provision of Parole deprived of property save by authority of law.—No
is a privilege/concession but not a right of any person shall be deprived of his property save by
convicted prisoner. authority of law.

*******

35
POLITY WORKBOOK www.iasscore.in

DIRECTIVE PRINCIPLES OF STATE POLICY (DPSP)

1. Exp. (b) 5. Exp : (b)

 Statement 1 is correct: As per the Section 33 of Welfare State


the WildLife Protection Act (WLPA), once the  A welfare state is a concept of the government in
Centre notifies an area as a community reserve, which the state plays a key role in the protection
the Chief Wildlife Warden of the state becomes the and promotion of the economic and social well-
being of its citizens.
governing authority of the forest, whose consent
is required for all decisions pertaining to the area.  The Directive Principles of State Policy (DPSP)
provides guidelines to the central and the state
 Statement 2 is correct: After a forest has been governments of India, to be kept in mind while
made into a community reserve, people are not framing laws and policies. The concept behind
allowed to hunt there, thus hunting is prohibited DPSP is to establish a “Welfare State” rather than
in community reserves. a “Police State” such as of colonial era. In other
 Statements 3 is correct and 4 is incorrect: The words, motive behind inclusion of DPSP is to
establish social and economic democracy rather
people of community reserves are allowed to
than Political democracy.
collect non-timber forest produce and those people
are not allowed to use community reserves for 6. Exp : (c)
traditional agricultural practices such as shifting
 The Constitution lays down certain Directive
(jhum) cultivation.
Principles of State Policy, which though not
 Community reserves in India are terms denoting justiciable, are ‘fundamental in governance of the
protected areas of India which typically act as country’, and it is the duty of the State to apply
buffer zones to or connectors and migration these principles in making laws.
corridors between established national parks,
wildlife sanctuaries and reserved and protected 7. Exp : (b)
forests of India. Directive principles of state policy
 Part IV, Articles 36-51 of the Indian constitution
2. Exp : (b)
constitutes the Directive Principles of State Policy
 Participation of workers in the management of which contain the broad directives or guidelines to
industries was added to the Directive Principles be followed by the State while establishing policies
of State Policy by the 42nd Amendment to the and laws. The legislative and executive powers of
constitution the state are to be exercised under the purview of
the Directive Principles of the Indian Constitution.
3. Exp : (d)  Inspired by the Constitution of Ireland, the Directive
 The Directive Principles of State Policy does not Principles contain the very basic philosophy of
constitute limitations upon legislative function the Constitution of India, and that is the overall
development of the nation through guidelines
and executive function.
related to social justice, economic welfare, foreign
 They do not limit but act as the guidelines to the policy, and legal and administrative matters.
State for the welfare of public The Directive Principles are codified versions of
democratic socialist order as conceived by Nehru
4. Exp : (c) with an admixture of Gandhian thought.
 These provisions are contained in Part IV (Article
8. Exp : (a)
36-51) of the Constitution of India). Directive
Principles of State Policies are not enforceable by  The Universal Declaration of Human Rights (1948
any court, but the principles laid down therein are consists of the following: The preamble sets out the
considered fundamental in the governance of the historical and social causes that led to the necessity
country, making it the duty of the State to apply of drafting the Declaration.
these principles in making laws to establish a just  Articles 1–2 established the basic concepts of
society in the country. dignity, liberty, and equality.
 This concept has been derived from the Directive  Articles 3–5 established other individual rights,
Principles given in the Constitution of Ireland, such as the right to life and the prohibition of
which provides for social justice), economic slavery and torture.
welfare foreign policy and legal and administrative  Articles 6–11 refer to the fundamental legality of
matters. human rights with specific remedies cited for their

36
www.iasscore.in POLITY WORKBOOK

defence when violated. which the citizens can lead a good life. They also
 Articles 12–17 established the rights of the aim to establish social and economic democracy
individual towards the community, including through a welfare state. The Directive Principles
freedom of movement. of State Policy is guidelines/principles given to the
central and state governments of India, to be kept
 Articles 18–21 sanctioned the so-called in mind while framing laws and policies.
“constitutional liberties” and spiritual, public, and
political freedoms, such as freedom of thought, 10. Exp : (d)
opinion, religion and conscience, word, and
peaceful association of the individual.  Statement 1 is incorrect: The provisions
contained in this Part cannot be enforced by any
 Articles 22–27 sanctioned an individual’s economic, court
social and cultural rights, including healthcare.
It upholds an expansive right to a standard of  Statement 2 is correct: They shall not be enforced
living, provides for additional accommodations by any court
in case of physical debilitation or disability, and  Statement 3 is correct: The principles therein
makes special mention of care given to those in laid down are nevertheless fundamental in the
motherhood or childhood. governance of the country and it shall be the duty
 Articles 28–30 established the general means of of the State to apply these principles in making
exercising these rights, the areas in which the laws.
rights of the individual cannot be applied, the duty 11. Exp: (d)
of the individual to society, and the prohibition of
the use of rights in contravention of the purposes  In the liberal approach which is inherent in
of the United Nations Organization. democracy, all forms of power are rooted in the will
of the people. This approach enhances the rule of
 The Preamble of the constitution mentions about
law as one of the basic foundations of democracy;
liberty, equality, fraternity, justice - social, economic,
it affirms the separation of powers as a vehicle for
political and many such things which overlap with
their restraint, and it promotes individuals’ rights
the 1948 declaration as seen in Articles 1-2, 18-
and freedoms as a prerequisite for their dignity.
21, etc. Similarly, the 1948 declaration thus talks
of not just political rights but also of social and 12. Exp: (b)
economic rights especially under Articles 22-27. In
case of Indian constitution, these socio-economic  Part Iv (Directive Principles Of State Policy)- Article
rights are provided in the Directive Principles of 39- says that the operation of the economic system
state policy. Lastly, the declaration talks of not just does not result in the concentration of wealth and
rights but also duties of individual towards the means of production to the common detriment.
society as seen in the Articles 28-30. Therefore, all 13. Exp: (d)
the three parts reflect the spirit of the declaration.
Therefore the correct answer is (d).  In the liberal approach which is inherent in
democracy, all forms of power are rooted in the will
9. Exp : (a) of the people. This approach enhances the rule of
law as one of the basic foundations of democracy;
 Directive Principles of State Policy (DPSPs) aim
it affirms the separation of powers as a vehicle for
to create social and economic conditions under
their restraint, and it promotes individuals’ rights

FUNDAMENTAL DUTIES

1. Exp : (d) is necessary for their implementation.


 The duties are imposed upon the citizen and not  For example, mandamus cannot be sought against
upon the State. an individual who does not observe his duties
under this article. With respect to the duty under
 Legislation is necessary for their implementation.
clause (a) of this article, it has been held that
The fundamental duties : “proper respect is shown to the National Anthem
 The fundamental duties which were added by by standing up when the National Anthem is sung.
the Forty-second Amendment of the Constitution It will not be right to say that disrespect is shown
in 1976, in addition to creating and promoting by not joining in the singing.
a culture, also strengthen the hands of the  Even though brought in by the Constitution
legislature in enforcing these duties vis-a-vis the (Forty Second Amendment) Act 1976 during the
fundamental rights. Since the duties are imposed operation of the proclamation of Emergency, Part
upon the citizen and not upon the State legislation IV-A, laying down certain duties of the citizens, is

37
POLITY WORKBOOK www.iasscore.in

one of the most valuable parts of the Constitution. service when called upon to do so.
It is also the most neglected.  To promote harmony and the spirit of common
brotherhood amongst all the people of India
2. Exp : (d) transcending religious, linguistic and regional
 To uphold and protect the Sovereignty, Unity or sectional diversities; to renounce practices
and Integrity of India” is a provision made in the derogatory to the dignity of women.
Fundamental Duties.  To value and preserve the rich heritage of our
composite culture.
3. Exp : (a)
 To protect and improve the natural
 According to the constitution, following are the environment including forests, lakes, rivers,
duties to be followed by every citizen of India: wildlife and to have compassion for living
 To abide by the Constitution and respect its creatures.
ideals and institutions, the National Flag and  To develop the scientific temper, humanism
the National Anthem. and the spirit of inquiry and reform.
 To cherish and follow the noble ideals which  To safeguard public property and to abjure
inspired our national struggle for freedom. violence.
 To uphold and protect the sovereignty, unity,  To strive towards excellence in all spheres
and integrity of India. of individual and collective activity, so that
 To defend the country and render national the nation constantly rises to higher levels of
endeavor and achievement

SYSTEM OF GOVERNANCE

1. Exp. (c) evidence can supersede an explicit confession


 Statement 1 is correct: The Supreme Court in from an accused.
Nagaraj & others vs Union of India and UP Power 4. Exp. (d)
Corp Ltd vs Rajesh Kumar &Ors has held that
 If the President of India exercise his power as
there needs to be a balance between reservation
provided under article 356 of the constitutional
provided under Article 16 of the Constitution and in respect of a particular state, then The president
efficiency under Article 335. can make laws relating to that state.
 Statement 2 is incorrect: Article 335 does not Effects of Imposition of President’s Rule in a State:
define efficiency.
 The President can assume to himself all or any
2. Exp. (c) of the functions of the State Government or he
may vest all or any of those functions with the
 As per the provisions of Article 117 of the Governor or any other executive authority.
Constitution – Rajya Sabha cannot reject a Finance
 The President may dissolve the State Legislative
Bill. Hence Statement 1 is incorrect.
Assembly or put it under suspension. He may
 Statement 2 is correct, as Rajya Sabha can’t amend authorize the Parliament to make laws on
but only recommend amendment to a Money Bill behalf of the State Legislature.
as per the provisions of Article 110.  The Parliament can delegate the power to
 Statement 3 is also correct according to the make laws for the state to the President or any
provisions of Article 110. other body specified by him when the state
legislature is suspended or dissolved.
3. Exp. (a)
5. Exp : (b)
 Option: (a) Article 355, It shall be the duty of
the Union to protect every State against external  When the President’s Rule is imposed in a state,
the President dismisses the state council of
aggression and internal disturbance and to ensure
ministers headed by the chief minister. The state
that the government of every State is carried on in governor, on behalf of the President, carries
accordance with the provisions of this Constitution. on the state administration with the help of
 Option b :-It doesn’t exempt the chief secretary of the state or the advisors
appointed by the President. This is the reason why
 Option C: -A confession is an explicit admission a proclamation under Article 356 is popularly
of guilt from an accused. Evidence extracted from known as the imposition of ‘President’s Rule’ in
an accused often proves to be substantive as no a state. Further, the President either suspends

38
www.iasscore.in POLITY WORKBOOK

or dissolves the state legislative assembly. The of the House if he joins any political party after
Parliament passes the state legislative bills the expiry of six months from the date on which
and the state budget. The President’s Rule he takes his seat in the House. This means that he
does not lead to dissolution of local bodies. may join any political party within six months of
Hence, the answer is 1 and 3 only. taking his seat in the House without inviting this
disqualification.
6. Exp. (b)
 Statement 2 is correct: The law does not specify a
 Statement 1 is incorrect: A nominated member of time-period for the Presiding Officer to decide on a
a House becomes disqualified for being a member disqualification plea.

UNION EXECUTIVE

1. Exp. (d) assemblies of states and union territories,


 Article 71 of the Constitution states that Matters including Delhi, Puducherry and Jammu and
relating to, or connected with, the election of a Kashmir. If the number of elected Assembly seats
president or Vice President increased, the value of vote of each MLA of that
state will decrease.
 All doubts and disputes arising out of or in
connection with the election of a president  Statement 3 is incorrect: The value of vote of
or vice President shall be inquired into and each MLA of Madhya Pradesh is 131 which is less
decided by the Supreme court whose decision than that of Kerala where it is 152.
shall be final  Statement 4 is correct: The value of vote of each
 If the election of a person as President or Vice MLA of Puducherry (16) is higher than that of
President is declared void by the Supreme Arunachal Pradesh (8) because the ratio of total
court, acts done by him in the exercise and population to total number of elective seats in
performance of the powers and duties of the Puducherry is greater as compared to Arunachal
office of President or Vice President, as the case Pradesh.
may be, on or before the date of the decision of
the Supreme Court shall not be invalidated by 3. Exp : (a)
reason of that declaration
What is the value of each vote and how is it
 Subject to the provisions of this constitution, calculated?
Parliament may by law regulate any matter
 A vote cast by each MP or MLA is not calculated as
relating to or connected with the election of a
one vote.
President or Vice President
 The fixed value of each vote by an MP of the Rajya
 The election of a person as President or Vice
Sabha and the Lok Sabha is 700.
President shall not be called in question on
the ground of the existence of any vacancy for  Meanwhile, the vote value of each MLA differs
whatever reason among the members of the from State to State based on a calculation that
electoral college electing him. factors in its population vis-a-vis the number of
members in its legislative Assembly.
2. Exp. (a)
 As per the Constitution (Eighty-fourth
 Statement 1 is incorrect: As per Article 54 of the
Amendment) Act 2001, currently, the
Constitution, the President of India is elected by
population of States is taken from the figures
the Members of an Electoral College consisting
of (a) the elected members of both Houses of of the 1971 Census. This will change when the
Parliament, and (b) the elected members of the figures of the Census taken after the year 2026
Legislative Assemblies of all States [including are published.
National Capital Territory of Delhi and the Union  The value of each MLA’s vote is determined
Territory of Puducherry]. The Members nominated by dividing the population of the State by the
to either House of Parliament or the Legislative
number of MLAs in its legislative Assembly, and
Assemblies of States, including NCT of Delhi and
the quotient achieved is further divided by 1000.
Union Territory of Puducherry, are not eligible to
be included in the Electoral College.  Uttar Pradesh for instance, has the highest vote
 Statement 2 is incorrect: The value of the vote value for each of its MLAs, at 208. The value
of an MP in a presidential election is based on of one MLA’s vote in Maharashtra is 175, while
the number of elected members in legislative that in Arunachal Pradesh is just 8.

39
POLITY WORKBOOK www.iasscore.in

4. Exp : (d)  The Council of Ministers comprises Ministers


Powers and Function of the Prime Minister: who are members of Cabinet, Ministers of State
(independent charge), Ministers of State and
 Executive power of the Union is vested in the
President, and is exercised by him either directly or Deputy Ministers.
through officers subordinate to him in accordance
with the Constitution. Supreme command of 7. Exp : (a)
defence forces of the Union also vests in him.  The Prime Minister of India, at the time of his/ her
 The highest ranking civil servant is the Cabinet appointment need not necessarily be a member
Secretary. He is ex-officio Chairman of the of one of the Houses of the Parliament but must
Civil Services Board; the chief of the Indian become a member of one of the Houses within six
Administrative Service and head of all civil services months
under the rules of business of the Government of
India. He also holds the 11th position in the Order Supplementary notes:
of Precedence of India. Prime minister:
 He is the head of government or the real executive
5. Exp : (c)
in the Indian system.
Cabinet Secretariat:
 President appoints the prime minister however no
 The PM is the head of the secretariat but
system of appointment is given in the constitution.
administrative head is cabinet secretary who
is also chairman of the civil service board. The However by convention of a parliamentary
cabinet secretariat functions directly under the democracy the leader of the largest party of
PM. parliament becomes the PM.
 Cabinet secretary is the head of the civil service.  The president can exercise discretion when no
Functions of the Cabinet Secretariat party has clear majority. He appoints a person and
asks him to prove his majority in the house. If the
 The work allocated to Cabinet Secretariat is
Secretarial assistance to Cabinet and Cabinet PM dies and no successor is in sight then again
Committees; and Rules of Business. the president can appoint a suitable person at his
discretion as caretaker for continuity.
 The Cabinet Secretariat is responsible for the
administration of the Government of India  However if the winning party has a candidate
(Transaction of Business) Rules, 1961 and the then the president has no choice.
Government of India (Allocation of Business)
 To be a PM a person need not be an MP but he has
Rules, 1961.
to become one within 6 months of being appointed
 It works in facilitating smooth transaction or else his appointment become void.
of business in ministries/departments of the
Government by ensuring rules are followed.  Though the PM occupies his post during the
pleasure of the president he can’t be removed till
6. Exp : (b) he commands the majority in the house.
Executive
8. Exp : (c)
 The Union executive consists of the President, the
Vice-President, and the Council of Ministers with  According to the Constitution of India, it is the duty
the Prime Minister as the head to aid and advise of the President of India to cause to be laid before
the President. the Parliament are the Recommendations of the
Council of Ministers Union Finance Commission
 There is a Council of Ministers headed by the  The Report of the Comptroller and Auditor
Prime Minister to aid and advise the President in General, The Report of the National Commission
exercise of his functions. for the Scheduled Castes
 The Prime Minister is appointed by the President,
9. Exp. (b)
who also appoints other ministers on the advice
of Prime Minister. The Council is collectively  Statement 1 is incorrect: Constitution of India
responsible to the Lok Sabha. does not define any categorisation of ministers.
 It is the duty of the Prime Minister to communicate  Statement 2 is correct: The total number of
to the President all decisions of Council of Ministers
ministers including prime minister should not
relating to administration of affairs of the Union
exceed 15% of the total number of members of
and proposals for legislation and information
relating to them. LokSabha.

40
www.iasscore.in POLITY WORKBOOK

10. Exp. (d) removal of Attorney General mentioned in the


Constitution except that he/she holds office during
 Statement 1 is Incorrect: Solicitor General does
the pleasure of the president. Conventionally
not participate in the meetings of Parliament.
(but not mandatorily), he/she resigns when the
 Statement 2 is Incorrect: There is no provision government (council of ministers) resigns or is
regarding thethe procedure and grounds for the replaced, as he is appointed on its advice.

UNION LEGISLATURE

1. Exp. (b) (MLA) or Member of Parliament (MP). The post can


yield salaries, perquisites and other benefits. It has
 Statement 1 is incorrect: The Delhi Home Guards
not been defined in Constitution or Representation
has been raised under the Bombay Home Guards
of the People Act, 1951.
Act 1947 as extended to the NCT of Delhi in
1959. The Delhi Home Guards Rules were framed 3. Exp. (a)
in 1959.
 Department related Standing Committees: The
 Statement 2 is correct: The role of Home Committee system of Parliament is often used in
Guards is to serve as an auxiliary to the police several countries for oversight of regulators. In
in maintenance of internal security, help the India, there are 24 Department Related Standing
community in any kind of emergency such as an Committees that comprise members from both
air-raid, fire, cyclone, earthquake, epidemic etc., Houses of Parliament. These committees are
help in maintenance of essential services, promote ministry specific, and may review the working of
communal harmony and assist the administration
regulators within their respective departments.
in protecting weaker sections, participate in socio-
economic and welfare activities and perform Civil  The Ad hoc committees are temporary and
Defence duties. perform specific task. The Ad Hoc Committees
dissolve, once their task is done. These committees
 Statement 3 is correct: Fifteen Border Wing
are Ministries’ specific and review the working of
Home Guards (BWHG) Battalions have been
regulators within their respective departments.
raised in the border States viz. Punjab (6 Bns.),
Parliament may establish ad - hoc committees
Rajasthan ( 4Bns.), Gujarat (2 Bns.) and one each
which may examine the working of regulators.
Battalion for Meghalaya, Tripura and West Bengal
The two parliamentary committees on finance
to serve as an auxiliary to Border Security Force
which exercise oversight of regulators are:
for preventing infiltration on the international
border/coastal areas, guarding of VA/VPs and lines  The Committee on Estimates;
of communication in vulnerable area at the time
 The Public Accounts Committee (PAC).
of external aggression.
 The Committee on Estimates reviews budgetary
2. Exp. (a) estimates of government departments. Such
 Article 102 of the Constitution provides that a estimates include the budget of regulators. Most
person shall be disqualified from being chosen laws establishing independent regulators require
as a Member of Parliament (MP) if he holds an the Comptroller and Auditor General (CAG) to
office of profit under the government of India or prepare annual audit reports on the accounts of
the government of a state. However, Parliament the regulators. These reports are tabled before
can declare by law that the holding of certain Parliament and reviewed by the PAC. The PAC may
offices will not incur this disqualification. The require the regulator’s officers to depose before
Parliament (Prevention of Disqualification) the Committee. For instance, the Chairman and
Act, 1959 lists certain offices of profit under senior officers of SEBI deposed before the PAC
the central and state governments, which do when it was examining the working of SEBI.
not disqualify the holders from being an MP.  Finance Commission, Financial Sector Legislative
The Chairperson of the National Commission Reforms Commission and NITI Aayog have no role
for Scheduled Castes and Scheduled Tribes is in reviewing the independent regulators.
exempted from disqualification as per this list.
 The Parliament (Prevention of Disqualification) 4. Exp : (b)
Act, 1959 was amended five times.  Every house of the parliament has a Committee
 Office of profit is a position in government which on subordinate legislation whose main function is
cannot be held by Member of Legislative Assembly to examine the rules and regulations enacted by

41
POLITY WORKBOOK www.iasscore.in

the executive to fill the gaps in the laws enacted arises. Moreover the decision is immune from
by the parliament and report how far these rules judicial review (Supreme Court’s judgment in
are within limits prescribed in the main law. This Mohd. Saeed Siddiqui vs State of UP).
committee has 15 members each in Rajya Sabha
and Lok Sabha. 7. Exp : (b)
 1952 elections The Congress had won 364 out of 489
5. Exp : (a) LokSabha seats in the firstever general election
 Speaker of Assembly vacates his office earlier in held in India. Its vote share was 45 per cent. The
any of the following three cases: CPI was the second party with just 16 seats.
 if he ceases to be a member of the assembly;  The Swatantra Party was an Indian liberal-
conservative political party that existed from
 if he resigns by writing to the deputy speaker; 1959 to 1974
 if he is removed by a resolution passed by  Leader of opposition was recognized for the first
a majority of all the then members of the time in 1969.
assembly.  To become leader of opposition, a political party
 Such a resolution can be moved only after giving needs atleast 10% strength of the house. House
14 days advance notice. total 10% seat means
 Whenever the Assembly is dissolved, the Speaker  Rajya Sabha 245 25 (10%)
does not vacate his office and continues till the
 Lok Sabha 543 55 (10%)
newly elected member meets.
8. Exp : (d)
6. Exp : (c)
 India has opted for the Parliamentary form of
Money Bill Government, in which the executive (Council
 Under Article 110 of the Indian constitution, a of Ministers) is responsible to the lower House
bill is considered a Money Bill if it contains only of Parliament (Lok Sabha), because Lok Sabha
provisions dealing with all or any of the following is directly elected by people. The following
matters, namely: means are deployed by Parliament to control the
government:
 The imposition, abolition, remission, alteration
or regulation of any tax;  By asking questions about the affairs of the
Government Affairs. in Zero Hour (11.00–
 The regulation of the borrowing of money or 12.00 noon) a member of Parliament can ask
the giving of any guarantee by the Government any question in any urgent matter of public
of India; interest.
 The custody of the consolidated Fund or the  By Passing different motions against
Contingency Fund of India, the payment of government like censure motion, calling
moneys into or the withdrawal of moneys from attention motion, Adjournment Motion.
any such Fund;
 By passing the Annual Budget of the
 The appropriation of moneys out of the government. No Money can be withdrawn by
consolidated Fund of India; the government from the Consolidated Fund
of India (CFI) or no tax can be increased or
 The declaring of any expenditure to be imposed by the government without prior
expenditure charged on the Consolidated Fund approval of the Parliament.
of India or the increasing of the amount of any
such expenditure;  In extreme, cases by passing the No-Confidence
Motion against the Council of Ministers
 The receipt of money on account of the
Consolidated Fund of India or the public 9. Exp : (d)
account of India or the custody or issue of such Private members Bill
money or the audit of the accounts of the Union
or of a State; or  A bil introduced by the member of Parliament who
is not a Minister, i.e. a non-government member
 If a proposed legislation contains other is known as the Private memberts Bill. Members
features, ones that are not merely incidental to of Parliament other thatn ministers are private
the items specifically outlined, such a draft law members.
cannot be classified as a money bill List of Private Members Bills
 The decision of the Speaker is final on whether  Since 1952, only 14 private member’s bills have
a bill is a money bill or not in case any question become laws. Out of the 300 odd private members

42
www.iasscore.in POLITY WORKBOOK

bills that were introduced in the 14th Lok Sabha, Cases when a bill lapse:
only about 4% were discussed and the rest 96%
 A bill originated in the Lok Sabha but pending
lapsed without any debate.
in the Lok Sabha – lapses.
 In 2018, MP from Thiruvananthapuram Shashi
Tharoor introduced a private member’s bill to  A bill originated and passed by the Rajya Sabha
regulate betting in sports and penalise match but pending in Lok Sabha – lapses.
fixing.  A bill originated and passed by the Lok Sabha
but pending in the Rajya Sabha – lapses.
10. Exp : (c)
 A bill originated in the Rajya Sabha and
Who can file a nomination? returned to that House by the Lok Sabha with
 Anyone who is 25 years of age or older is eligible amendments and still pending in the Rajaya
to contest an election to a Lok Sabha seat. The Sabha on the date of the dissolution of Lok
candidate must be a registered elector of a Sabha- lapses.
constituency and a citizen of India. However, if s/ Cases when a bill does not lapse:
he is a registered voter in a particular state, s/he
can contest from any seat in any state.  A bill pending in the Rajya Sabha but not
passed by the Lok Sabha does not lapse.
11. Exp : (c)  If the president has notified the holding of
 A Parliamentary government is also known as the a joint sitting before the dissolution of Lok
Cabinet form of government because the cabinet is Sabha, does not lapse.
the real Executive in it. It is also called ‘Responsible
 A bill passed by both Houses but pending
government’. Since the Cabinet always remains
assent of the president does not lapse.
responsible to the Legislature for its activities.
 Responsibility of the Executive to the Legislature:  A bill passed by both Houses but returned
Since the Legislature gives birth to the Executive, by the president for reconsideration of Rajya
the Legislature has the authority to hold the Sabha does not lapse.
Executive responsible for all its actions. Thus  Some pending bills and all pending assurances
the Council of Ministers is responsible to Lok that are to be examined by the Committee on
Sabha. It is responsible to Rajya Sabha also. The Government Assurances do not lapse on the
responsibility is further conditioned through its dissolution of the Lok Sabha.
collectivity i.e. the responsibility of every Minister
is the responsibility of the entire Council of 14. Exp : (d)
Ministers.  The Parliament of India acquires the power to
 Dual Executive: The Parliamentary form of legislate on any item in the State List in the national
government provides for two Executives – the real interest if a resolution to that effect is passes by
Executive and the nominal or titular Executive. the RajyaSabha by a majority of not less than two-
The nominal Executive is represented by the head thirds of its members present and voting
of the State who may either be a hereditary or an
elective one; legally, the head of the State possesses 15. Exp : (b)
all powers and privileges which the Constitution Rajya Sabha in Financial Matters:
and laws may confer upon him. But in practice,
 A Money Bill can be introduced only in Lok
all powers are exercised by the real Executive
Sabha. After it is passed by that House, it is
represented by the Prime Minister and the Council
transmitted to Rajya Sabha for its concurrence or
of Ministers.
recommendation. The power of Rajya Sabha in
respect of such a Bill is limited.
12. Exp : (d)
 Rajya Sabha has to return such a Bill to Lok Sabha
 A First-Past-The-Post (FPTP; sometimes FPP, or within a period of fourteen days from its receipt.
winner takes all) electoral system is one in which If it is not returned to Lok Sabha within that
voters indicate on a ballot the candidate of their time, the Bill is deemed to have been passed by
choice, and the candidate who receives the most both Houses at the expiration of the said period
votes wins, even if they did not receive a majority in the form in which it was passed by Lok Sabha.
of the votes. Again, Rajya Sabha cannot amend a Money Bill;
it can only recommend amendments and Lok
13. Exp : (b) Sabha may either accept or reject all or any of the
Lapsing of bills : recommendations made by Rajya Sabha.
 Articles 107 and 108 of the Indian Constitution  Rajya Sabha does not vote on Demands for Grants
deals with these provisions. The position with of various Ministries - a matter exclusively
respect to lapsing of bills is as follows: reserved for Lok Sabha - no money, however, can

43
POLITY WORKBOOK www.iasscore.in

be withdrawn from the Consolidated Fund of India 20. Exp : (d)


unless the Appropriation Bill has been passed by  As per the procedure laid down in the Constitution,
both the Houses. Constitution Amendment Bills can be of three
 Similarly, the Finance Bill is also brought before types viz.,
Rajya Sabha. Besides, the Department-related
 Requiring simple majority for their passage in
Parliamentary Standing Committees that examine
each House;
the annual Demands for Grants of the Ministries/
Departments are joint committees having ten  Requiring special majority for their passage
members from Rajya Sabha. in each House i.e., a majority of the total
membership of a House and by a majority of
16. Exp : (a) not less than two-thirds of the members of that
Joint sitting: House present and voting (article 368); and
 Parliament of India is a bicameral. Consensus of  Requiring special majority for their passage
both the houses is required to pass the bill except and ratification by Legislatures of not less than
money bill. Constitution makers having realized one-half of the States by resolutions to that
the deadlock that may come in future, provided effect passed by those Legislatures (proviso
for the joint sitting of both the houses to resolve to clause (2) of article 368). A Constitution
the deadlock. Amendment Bill under article 368 can be
 Joint Sitting can be ordered by President of India introduced in either House of Parliament and
to consider a particular bill in case of following has to be passed by each House by special
reasons: majority.
 A bill is passed by one house and is rejected by the
21. Exp : (d)
other.
 Article 253 gives a carte blanche to the Centre:
 The amendments suggested by one house is not
“Notwithstanding anything in the foregoing
accepted by the house in which bill originated.
provisions of this Chapter (on Centre-State
 If the bill remains un passed for more than six relations in the legislative sphere), Parliament
months has power to make any law for the whole or any
 In a joint sitting a bill is passed simply by the part of the territory of India for implementing
majority of the both the members of the house any treaty, agreement or convention with any
present and voting. Since Lok Sabha has more other country/countries or any decision made at
members as compared to Rajya Sabha therefore any international conference, association or other
its influence prevails. body.”

17. Exp :(c) 22. Exp : (b)


The Parliamentary Committee on Public Accounts:
18. Exp : (c)
 In 1950, the Public Accounts Committee consisted
 In a parliamentary democracy, government can be of 15 members and all of them belonged to the Lok
in power only if it commands majority in directly Sabha. But in 1953, this number was increased to 22
elected House. with a view to give representation to Rajya Sabha.
 Article 75(3) of our Constitution of India embodies  All members of the Committee enjoy equal status
this rule by specifying that Council of Ministers is in matter of elaboration and voting. The members
collectively responsible to Lok Sabha. from Rajya Sabha are also under the control of the
 But there is no mention of a no-confidence motion Speaker of Lok Sabha insofar as their functioning
in the constitution. in the committee is concerned.
 The Rule 198 of Rules of Procedure and Conduct  Thus at present, the twenty two members are
of Business of Lok Sabha provides mechanism for elected on the basis of proportional representation
testing this collective responsibility. by means of single transferable vote.
 Rajya Sabha does not have procedure for moving  The tenure of the Committee is one year. The
of no-confidence motion against Government and chairman of the Committee is appointed by the
also adjournment motion, censure motion. Speaker of the Lok Sabha from amongst the
members of Committee. The qualification needed
19. Exp : (b) tor the chairmanship is only to be a member of the
 If the money bill is substantially amended by the Committee.
Rajya Sabha The Lok Sabha may still proceed  The chairmanship of the Public Accounts
with the bill, accepting or not accepting the Committee has been given to the Opposition, a
recommendations of the Rajya Sabha. practice which is considered democratic.

44
www.iasscore.in POLITY WORKBOOK

23. Exp : (b) present and voting declaring that it is necessary


 The Chairman and the Deputy Chairman of the or expedient in the national interest to create one
Rajya Sabha are the members of that House. or more All India Services common to the Union
and the States, Parliament becomes empowered to
 While the nominated members of the two Houses create by law such services.
of the Parliament have no voting right in the
presidential election, they have the right to vote in 27. Exp : (a)
the election of the Vice President
Specific Methods of Financial Control
24. Exp : (a) Annual Financial Statement
 The primary object of an adjournment motion  According to Article 112 of the Indian Constitution,
is to draw the attention of the House to a recent the President of India causes to be laid before
matter of urgent public importance having serious both the Houses of Parliament an ‘annual
consequences and in regard to which a motion or financial statement’ containing the statement
a resolution with proper notice will be too late. of the estimated receipts and expenditure of the
Government of India for that year.
 The matter proposed to be raised should be of
such a character that something very grave which Consolidated Fund
affects the whole country and its security has  As mentioned in Article 113, the estimates of
happened and the House is required to pay its expenditure charged upon the Consolidated
attention immediately by interrupting the normal Fund of India are not submitted to the vote of
business of the House. Parliament, although there can be discussion on
 The adjournment motion is thus an extraordinary the same in either House of Parliament.
procedure which, if admitted, leads to setting aside Demands for Grants
the normal business of the House for discussing a  On the recommendation of the President of India,
definite matter of urgent public importance. the estimates of expenditure, other than those
specified for the Consolidated Fund of India, are
25. Exp : (b) presented to the Lok Sabha in the form of demands
Speaker of the Lok Sabha: for grants. Under Article 113, the Lok Sabha has
the power to assent to or to reject, any demand, or
 The Speaker is elected by the Lok Sabha from to assent to any demand, subject to a reduction of
amongst its members (as soon as may be, after its the amount specified.
first sitting). Whenever the office of the Speaker
Supplementary, Additional or Excess Grants
falls vacant, the Lok Sabha elects another member
to fill the vacancy. The date of election of the  Article 115 of the Constitution lays down that
Speaker is fixed by the President. statements showing the estimates of expenditure
for the supplementary, additional or excess grants
 Usually, the Speaker remains in office during the have to be presented to the Lok Sabha
life of the Lok Sabha. However, he has to vacate his
Cut Motions
office earlier in any of the following three cases:
 Cut motions are introduced to discuss the policy
 if he ceases to be a member of the Lok Sabha; pursued in regard to a specific matter by the
 if he resigns by writing to the Deputy Speaker; minister concerned or to ventilate grievances or
and to suggest economies. Each such motion has to
focus on one demand and one-matter only, which
 if he is removed by a resolution passed by a needs to be precisely stated. It must not relate to
majority of all the members of the Lok Sabha. the expenditure charged on the Consolidated Fund
Such a resolution can be moved only after of India or make suggestions for the amendment
giving 14 days’ advance notice. or repeal of existing laws.
Appropriation Bill
26. Exp : (b)  According to Article 114 of the Constitution, when
Special Powers of Rajya Sabha the demand for grants has been voted for, the
 However, if Rajya Sabha passes a resolution by a Appropriation Bill authorises the withdrawal of
the funds from the Consolidated Fund of India,
majority of not less than two-thirds of members
as regards both the votable and the charged
present and voting saying that it is “necessary or
items. Notably, the Appropriation Bill provides
expedient in the national interest” that Parliament an occasion for a full-fledged discussion on topics
should make a law on a matter enumerated in the chosen by the various parties in the House.
State List, Parliament becomes empowered to make
a law on the subject specified in the resolution, for 28. Exp : (a)
the whole or any part of the territory of India.
 Joint sitting is an extraordinary machinery
 If Rajya Sabha passes a resolution by a majority provided by the Constitution to resolve a deadlock
of not less than two-thirds of the members between the two Houses over the passage of a bill.

45
POLITY WORKBOOK www.iasscore.in

 It must be noted here that the provision of joint 32. Exp : (d)
sifting is applicable to ordinary bills or financial How Budget is Made and Passed
bills only and not to money bills or Constitutional
amendment bills.  The department of economic affair under the
Ministry of Finance is the nodal agency responsible
for producing the Union Budget.
29. Exp : (c)
 Budget is made through a consultative process
 Full Budget deals with both expenditure and
involving the finance ministry, the NITI Aayog, and
revenue side but Vote-on-account deals only with the various spending ministries.
the expenditure side of the government’s budget.
 According to Article 112, The President of India is
 The vote-on-account is normally valid for two responsible of presenting Budget in the Lok Sabha.
months but a full budget is valid for 12 months (a However, Article 77 (3) says the union Finance
financial year). Minister of India has been made responsible by the
 As a convention, a vote-on-account is treated as a President of India to prepare the annual financial
formal matter and passed by Lok Sabha without statement and present it in Parliament.
discussion. But passing for budget happens only
33. Exp : (d)
after discussions and voting on demand for grants.
 Government solely survives on faith of Lok Sabha.
 A vote-on-account cannot alter direct taxes since
Further, Annual Union Budget has to be passed by
they need to be passed through a finance bill. Lok Sabha alone.
Under the regular Budget, fresh taxes may be
imposed and old ones may go.  Thus is Lok Sabha refuses to pass annual budget,
then it will mean the government of the day has
 As a convention, a vote-on-account is treated as a lost the faith of Lok Sabha i.e. it does not have
formal matter and passed by the Lok Sabha without majority of Lok Sabha. It will force the PM to resign
discussion. But the full budget is passed only after and request Lok Sabha to pass vote on accounts
discussions and voting on demand for grants. to take care of finances till the new arrangement
(new government or elections) is found.
30. Exp : (b)
Consolidated Fund of India 34. Exp : (b)
 Option B is correct: RS has equal powers with LS
 This is the chief account of the Government of
in amending the constitution.
India. The inflow to this fund is by way of taxes
like Income Tax, Central Excise, Customs and also 35. Exp : (d)
non-tax revenues which arise to the government
in connection with the conduct of its business.  The Macro-Economic Framework Statement,
Loans raised by issue of treasury bills are also presented to Parliament as per the Fiscal
Responsibility and Budget Management Act, 2003.
received in this fund. The government meets all its
expenditure including loan repayments from this
36. Exp : (c)
fund. No amount can be withdrawn from the fund
without the authorisation from the Parliament. 37. Exp: (b)
This fund is formed under the provision of Aricle
 Statement 1 is incorrect: As per Section 33(7) of the
266 (1) of the Indian Constitution
RPA, one candidate can contest from a maximum
of two constituencies (more constituencies were
31. Exp : (c) allowed until 1996 when the RPA was amended to
Consolidated Fund of India set the cap at two constituencies).
 Consolidated Fund of India is the most important  Statement 3 is incorrect: If a candidate is contesting
of all government accounts. Revenues received from more than one constituency, he has to lodge
by the government and expenses made by it, a separate return of election expenses for every
excluding the exceptional items, are part of the election which he has contested. The election for
Consolidated Fund. each constituency is a separate election. (Refer :
Section 77 of the Representation of People Act, 1951).
 Description: This fund was constituted under
Article 266 (1) of the Constitution of India. All 38. Exp: (b)
revenues received by the government by way of  Statement 1 is incorrect: As per Section 33(7) of the
direct taxes and indirect taxes, money borrowed RPA, one candidate can contest from a maximum
and receipts from loans given by the government of two constituencies (more constituencies were
flow into the Consolidated Fund of India. allowed until 1996 when the RPA was amended to
 All government expenditure is made from this set the cap at two constituencies).
fund, except exceptional items which are met  Statement 3 is incorrect: If a candidate is
from the Contingency Fund or the Public Account. contesting from more than one constituency, he
Importantly, no money can be withdrawn from has to lodge a separate return of election expenses
this fund without the Parliament’s approval. for every election which he has contested. The

46
www.iasscore.in POLITY WORKBOOK

election for each constituency is a separate He is elected after the election of the Speaker has
election. (Refer : Section 77 of the Representation taken place. The date of election of the Deputy
of People Act, 1951). Speaker is fixed by the Speaker. Whenever the
office of the Deputy Speaker falls vacant, the Lok
39. Exp. (b)
Sabha elects another member to fill the vacancy.
 Statement 1 is incorrect: For the ratification of  Upto the 10th Lok Sabha, both the Speaker and
the proclamation of emergency, a resolution for the Deputy Speaker were usually from the ruling
the same must be passed by special majority by the
party. Since the 11th Lok Sabha, there has been a
Both Houses separately.
consensus that the Speaker comes from the ruling
 Statement 3 is incorrect: Both the Houses have party (or ruling alliance) and the post of Deputy
the role in the impeachment of the President. Speaker goes to the main opposition party.
After the impeachment resolution is passed by a
 The Deputy Speaker performs the duties of the
majority of two-thirds of the total membership of
Speaker’s office when it is vacant. He also acts as
a House, it is sent to the other House, which should
the Speaker when the latter is absent from the
investigate the charges. If the other House also
sitting of the House. In both cases, he assumes all
sustains the charges and passes the impeachment
the powers of the Speaker. He also presides over
resolution by a majority of two-thirds of the total
the joint sitting of both the Houses of Parliament,
membership, then the President stands removed
in case the Speaker is absent from such a sitting.
from his office.
 At any time before noon on the day preceding
 Statement 2 is correct: Article 75 of the
the date so fixed, any member may give notice
Constitution says that the council of ministers shall
in writing, addressed to the Secretary-General,
be collectively responsible to the Lok Sabha.
of a motion that another member be chosen as
40. Exp. (a) the Deputy Speaker of the House and the notice
shall be seconded by a third member and shall be
Deputy Speaker of Lok Sabha accompanied by a statement by the member whose
 Like the Speaker, the Deputy Speaker is also elected name is proposed in the notice that the member
by the Lok Sabha itself from amongst its members. proposed is willing to serve as Deputy Speaker.

STATE EXECUTIVE

1. Exp. (c)  The consequences of the proclamation of a


Financial Emergency are as follows:
Sarkaria Commission provides for these:
 The executive authority of the Centre extends
 While selecting Governors, the Central Government
(a) to directing any state to observe such canons
should adopt the following strict guidelines as
of financial propriety as are specified by it; and
recommended in the Sarkaria Commission report
(b) to directions as the President may deem
and follow its mandate in letter and spirit:
necessary and adequate for the purpose.
 He should be eminent in some walk of life
 Any such direction may include a provision
 He should be a person from outside the state requiring (a) the reduction of salaries and
allowances of all or any class of persons
 He should be a detached figure and not too serving in the state; and (b) the reservation of
intimately connected with the local politics of all money bills or other financial bills for the
the states consideration of the President after they are
 He should be a person who has not taken passed by the legislature of the state.
too great a part in politics generally and  The President may issue directions for the
particularly in the recent past. reduction of salaries and allowances of (a) all
or any class of persons serving the Union; and
2. Exp : (c) (b) the judges of the Supreme Court and the
 Like the President, the governor is also entitled high court.
to a number of privileges and immunities. He
enjoys personal immunity from legal liability for 3. Exp : (d)
his official acts. During his term of office, he is Chief Secretary in a State
immune from any criminal proceedings, even in  In India each state and some Union Territories
respect of his personal acts. have Chief Secretaries. Chief Secretary serves as

47
POLITY WORKBOOK www.iasscore.in

head of all government staff in the state and is the  Dismissal of a Ministry: A minister holds
Secretary of the State Cabinet of Ministers. offices during the pleasure of the Governor.
 The post of Chief Secretary is encadred within the When the ministry losses support of the house,
the governor will dismiss the ministry. But
Indian Administrative Service (IAS), meaning that
he cannot dismiss it unitl it losses majority
only an IAS officer may hold this position.
support.
 The Chief Secretary holds the same rank as a
Secretary to the Government of India and the post  Advising the President for proclamation
falls within the “Apex Scale”. Other positions in this of Emergency: The Governor advises the
payscale are Additional or Special Chief Secretary President to proclaim emergency when he is
satisfied that the Government cannot carried
and Special Secretary to the Government of India.
on in accordance with the provisions of the
 By tradition the senior most IAS officer of the state constitution, under Article 356.
cadre is chosen as the Chief Secretary but in many
cases this is not so. The Chief Secretary heads the  Reservation of a bill for the consideration
Department of General Administration as well. of the President: However, situation are
mentioned in Article 200, when he will reserve
 Chief Secretary is appointed by the Chief Minister the bill, yet he can use, discretion regarding
and the Ministers. this matter.
 No fixed tenure is imposed on the office but term
can be extended 5. Exp : (c)
Governor’s appointment and removal
4. Exp : (b)
 No procedure has been laid down in the
Discretionary power of Governor: Constitution of India for the removal of a Governor
 The Governor of state has more discretionary from his/her post.
powers than the President of India. He is not bound  The President appoints the Chief Minister and
to act on the advice of the council of Ministers in Ministers for Union Territories, who will hold
certain circumstances, even he need not seek its office during the President’s reign.
advise.
 The Constitutional position on the removal of
 The constitution does not specify these matters but the Governor is that a Governor of a state is an
the matters in which he can act without the advice appointee of the President, and he or she holds
of the council of ministers are as follows: office “during the pleasure of the President”
 Appointment of the Chief Minister: Generally, (Article 155 and 156). If the Governor continues to
the leader of the party with majority is enjoy the pleasure of the President, s/he can have
appointed as the Chief Minister. But in situation a normal term of five years. Now ,the President
where no party gets absolute majority, the is bound by the aid and advice of the Council
Governor exercises his discretionary powers of Ministers under Article 74, so it is the central
government which appoints or removes the
in appointing the Chief Minister.
Governor.

STATE LEGISLATURE

1. Exp. (a)  The Address of the Governor contains a review of


 Prisons Act, 1894 makes it expressly clear that the activities and Achievements of the Government
States would have general and specific control during the previous year and their policy with
over prisons in India. regard to important internal problems as well as
 It is also important to note that the Subject of a brief account of the programme of Government
Prisons is mentioned as Entry 4 in List II in Seventh Business for the session.
Schedule.  When a State legislature does not have a rule on a
2. Exp. (c) particular matter, it follows the Lok Sabha rule on
that matter.
 Article 176(1) of the Constitution of India enjoins
that the Governor shall Address both the Houses
assembled together at the commencement of the 3. Exp: (d)
first Session after each general election to the How are members of the Council elected?
Assembly and at the commencement of the first
 Membership may vary, but the Legislative Council
session of each year and inform the Legislature
of the causes of its Summons. must not have more than a third of the total

48
www.iasscore.in POLITY WORKBOOK

membership of the Assembly of that state, and in  If he is not a member of Parliament, then he
no case fewer than 40 members. should be elected to the Lok Sabha or the Rajya
 About 1/3rd of members are elected by members Sabha within six months after his appointment as
of the Assembly, another 1/3rd by electorates a minister.
consisting of members of municipalities, district
A person found guilty of any of the following shall
boards and other local authorities in the state,
be disqualified for a period of 6 years to contest
1/12th by an electorate consisting of teachers, and
1/12th by registered graduates. The remaining elections after release from jail:
members are nominated by the Governor from  Promoting hatred and enmity between classes
among those who have distinguished themselves
 Influencing elections
in literature, science, art, the cooperative
movement, and social service. Legislative Councils  Bribery
are permanent Houses, and like Rajya Sabha, one-  Rape or other grave crimes against women
third of their members retire every two years.
 Spreading religious disharmony
4. Exp : (d)
 Practising untouchability
Qualifications for a person should possess to
 Importing or exporting prohibited goods
become eligible for appointment as ministers are:
 He should be a citizen of India.  Seling or consuming illegal drugs as well as other
chemicals
 He should be above 25 years of age
 Engaging in terrorism in any form
 He should not hold any office of profit under the
government of India  have been imprisoned for at least two years

LOCAL GOVERNMENT

1. Exp : (b) Jurisdiction


 Local Self Government is a form of democratic  The Gram Nyayalaya shall be a mobile court and
decentralization where the participation of even shall exercise the powers of both Criminal and
the grass root level of the society is ensured in the Civil Courts.
process of administration.
 The Gram Nyayalaya shall try criminal cases, civil
2. Exp : (b) suits, claims or disputes which are specified in the
First Schedule and the Second Schedule to the Act.
 The Panchayati Raj System, as established in
accordance with the 73rd Amendment, is a three-  The Central as well as the State Governments have
tier structure based on direct elections at all the been given power to amend the First Schedule
three tiers: village, intermediate and district.
and the Second Schedule of the Act, as per their
 The minimum age for any person to be a member respective legislative competence.
of Panchayat is 21 years.
 The Gram Nyayalaya shall exercise the powers of
 The Amendment provides for the continuous
existence of Panchayats. The normal term of a a Civil Court with certain modifications and shall
Panchayat is five years. If a Panchayat is dissolved follow the special procedure as provided in the
earlier, elections are held within six months. Act.
There is a provision for State Election Commission,
for superintendence, direction, and control of  The Gram Nyayalaya shall try to settle the disputes
the preparation of electoral rolls and conduct of as far as possible by bringing about conciliation
elections to Panchayats. between the parties and for this purpose; it shall
make use of the conciliators to be appointed for
3. Exp : (b) this purpose.
Gram nyayalayas
4. Exp : (c)
 Gram Nyayalayas are mobile village courts in
India established for speedy and easy access to Panchayat Raj:
justice system in the rural areas of India.  Financial mobilization was never the basis for the
 They are aimed at providing inexpensive justice to establishment of panchayati raj. Local financial
people in rural areas at their doorsteps. mobilization (by way of octroi taxes etc. ) is the

49
POLITY WORKBOOK www.iasscore.in

outcome of the establishment of the PRIs, not its 24 December 1996 to enable Tribal Self Rule in
objective. these areas. The Act extended the provisions of
Panchayats to the tribal areas of nine states that
 Political accountability can be ensured in any
have Fifth Schedule Areas. Most of the North
system which is democratic/rule-based, it need not
eastern states under Sixth Schedule Areas (where
necessarily be via a local
autonomous councils exist) are not covered by
 Capacity building through skill oriented training PESA, as these states have their own Autonomous
programmes on Management Development, councils for governance
Technology Support, Skill Up- gradation, etc., for
self- employment in rural areas by taking 6. Exp : (d)
Important Features of the Constitution 73rd and
5. Exp : (c)
74th Amendments
PESA Act of 1996 (Extension Act)
 Independent Election Commission in each State
 Village level democracy became a real prospect for superintendence, direction and control of the
for India in 1992 with the 73rd amendment to electoral rolls (Article 243K).
the Constitution, which mandated that resources,
 74th Amendment provides for a District Planning
responsibility and decision making be passed on
Committee to consolidate the plans prepared by
from central government to the lowest unit of
Panchayats and Municipalities (Article 243ZD).
the governance, the Gram Sabha or the Village
Assembly. A three tier structure of local self-  Establish a Finance Commission in each State to
government was envisaged under this amendment. determine the principles on the basis of which
adequate financial resources would be ensured
 Since the laws do not automatically cover the
for panchayats and municipalities (Article 243I).
scheduled areas, the PESA Act was in acted on

UNION TERRITORIES AND SPECIAL AREAS

1. Exp. (b) for certain ‘scheduled areas’ in mainland and


certain ‘tribal areas’ in north eastern India.
 Statement 1- is correct: As per the provisions of
Article 244 (1) of the Indian Constitution defines
3. Exp :(d)
Scheduled Areas as the areas defined so by the
President of India and are mentioned in the fifth  Delhi, J&K and Pondicherry are represented in
schedule of the Constitution. RS. It is not within the purview of Chief Election
Commissioner.
 Statement 2- is correct The largest administrative
unit forming the scheduled areas has been the 4. Exp. (a)
district and the lowest the cluster of villages in the
block. Most of the districts form scheduled areas  Statement (a) is correct: Governor can make
only partially. regulations for the peace and good government
 Statement 3- is incorrect: As the report on of a scheduled area after consulting the tribes
administration of schedule areas is send by advisory council. Such regulations may prohibit or
Governor of the State to President of India. restrict the transfer of land by tribal to non-tribal
2. Exp : (a) members or among members of the scheduled
 The provisions in Fifth Schedule and Sixth tribes, regulate the allotment of land to members
Schedule in the Constitution of India are made in of the scheduled tribes.
order to protect the interests of Scheduled Tribes
 Statement (b) is incorrect: Tribal advisory
 Fifth and Sixth Schedules were discussed and council is an advisory body, not a governing body.
passed by Constituent Assembly between
September 5-7, 1949. These two schedules provide  Statement (c) and (d) are incorrect (Self-
for alternate or special governance mechanisms explanatory)

******

50
www.iasscore.in POLITY WORKBOOK

THE JUDICIARY

1. Exp. (b)  Article 124(1) states that there shall be a Supreme


Court of India consisting of a Chief Justice of India
 44 amendment didn’t provide for Article placing
th
and, until Parliament by law prescribes a large
the election of the Prime Minister beyond judicial
number, of not more than seven other judges.
review.
 Article 124(2) states that every judge of the
 99th amendment was declared null and void for Supreme Court shall be appointed by the President
violating independence of judiciary by bringing in by warrant under his hand and seal after
executive in selection process of judges. consultation with such number of the judges of
Supreme Court and of the High Courts (in states).
2. Exp. (c)
 The Parliament is competent to increase the
 Speaker can refuse to accept the motion number of judges if it deems necessary.
 Incapacity and proved misbehavior haven’t been
defined. 7. Exp : (c)
Original Jurisdiction of the SC in India
3. Exp. (d)
 It involves the following cases:
 “The 42nd Amendment Act of 1976 curtailed the
judicial review power of high court. It debarred  Any dispute between the Indian Government
the high courts from considering the constitutional and one or more States.
validity of any central law. However, the 43rd  Any dispute between the Indian Government
Amendment Act of 1977 restored the original and one or more States on one side and one or
position.” more States on the other side.
 Justice Syed Shah Mohamed Quadri has classified
 Any dispute between two or more States.
the judicial review into the following three
categories “Judicial review of constitutional  Article 32 of the Constitution provides original
amendments.” jurisdiction to the SC for matters regarding the
enforcement of Fundamental Rights.
4. Exp : (d)
 The SC can issue writs, directions, or orders
 The Supreme Court or the apex court of India acts including writs in the nature of mandamus,
as the guardian of the Constitution. habeas corpus, quo warranto, prohibition and
 Supreme Court is the guarantor and protector certiorari.
of fundamental rights of citizens. An aggrieved  The SC also has the power to direct the transfer
person can directly approach the Supreme Court of a criminal or civil case from the High Court
in case of infringement of his/her fundamental in one State to the High Court in another State.
rights. Under article 32 of the Indian Constitution,
SC has the power of writ jurisdiction for the  It can also transfer cases from one subordinate
enforcement of fundamental rights. These are the court to another State High Court
writs of habeas corpus, mandamus, prohibition,
certiorari and quo warranto  If the SC deems that cases involving the same
questions of law are pending before it and
5. Exp : (c) one or more High Courts, and that these are
significant questions of law, it can withdraw
Original Jurisdiction- (Art 131)
the cases before the High Court or Courts and
 This jurisdiction extends to cases originating in dispose off all these cases itself.
SC only and states that Indian SC has original and
exclusive jurisdiction in cases between:  The Arbitration and Conciliation Act, 1996 gives
the SC the authority to initiate international
 Government on one hand and one or more
commercial arbitration.
states on the other
 Government and one or more states on one 8. Exp : (a)
side and other states on the other Appointment of acting Chief Justice (Article 126):
 Two or more states  When the Office of the Chief Justice of India is
vacant or when the Chief Justice, by reason of
6. Exp : (b) absence or otherwise, is unable to perform the
Constitutional Provisions duties of his office, the duties of the office shall

51
POLITY WORKBOOK www.iasscore.in

be performed by such one of the other Judges of its own procedure for the determination of any
the Court as the President may appoint for the dispute coming before it.
purpose.
11. Exp : (b)
Appointment of Ad hoc Judges (Article 127):
 Article 50 of Constitution of India is a directive
 If at any time there is a lack of quorum of the principle of state policy. It gives a direction to
Judges of the Supreme Court to hold or continue the State to keep Judiciary independent of the
any session in the Court, the Chief Justice of India Executive, particularly in judicial appointments.
may with the previous consent of the President
and after consultation with the Chief Justice of 12. Exp: (c)
the High Court concerned, request in writing the  Statement 1 is correct: Under Article 128 of the
attendance at the sittings of the Court, as an ad Constitution, the Chief Justice of India may, at any
hoc Judge, for such period as may be necessary, time, with the previous consent of the President,
of a Judge of a High Court duly qualified for request any person who has held the office of a
Judge of the Supreme Court to sit and act as a Judge
appointment as a Judge of the Supreme Court to be
of the Supreme Court.
designated by the Chief Justice of India.
 Statement 2 is correct: Being a Court Of Record,
Attendance of retired Judges at sittings of the High Court can review its own judgments under
Supreme Court (Article 128): Article 226 of the Constitution. The Kerala High
 The Chief Justice of India with the previous Court has iterated the legal proposition that High
consent of the President may request a retired Courts as Courts of Record could review their
own orders. A Division Bench of Chief Justice S
Judges of the Supreme Court or Federal Court or a
Manikumar and Shaji P Chaly were confronted
retired Judge of a High Court who is duly qualified
with an appeal against a review petition.
to be appointed as a Judge of the Supreme Court
to sit and act as a Judge of the Supreme Court. 13. Exp: (c)
Such a judge is entitled to such allowances as the  Statement 1 is correct: Under Article 128 of the
President may determine Constitution, the Chief Justice of India may, at any
time, with the previous consent of the President,
9. Exp : (b) request any person who has held the office of a
 The President may obtain the opinion of the Judge of the Supreme Court to sit and act as a Judge
Supreme Court on any question of law or fact that of the Supreme Court.
has arisen or likely to arise.  Statement 2 is correct: Being a Court Of Record,
High Court can review its own judgments under
 Said question of law or fact is of such nature or Article 226 of the Constitution. The Kerala High
of such public importance that it is expedient to Court has iterated the legal proposition that High
obtain the opinion of Supreme Court. Courts as Courts of Record could review their
own orders. A Division Bench of Chief Justice S
 The Supreme Court, on receiving such reference
Manikumar and Shaji P Chaly were confronted
may, after such hearing as it thinks fit, report its with an appeal against a review petition.
opinion to the President
14. Exp. (b)
10. Exp : (d)  Statement 1 is correct: Contempt of Courts Act,
Powers of Lok Adalats 1971 was passed on the recommendation of H. N.
Sanyal Committee.
 The Lok Adalat shall have the powers of a civil
court under the Code of Civil Procedure 1908,  Statement 2 is correct: Constitution Of India
while trying a suit, in respect of the following empowers Supreme Court and High Courts to
matters: punish for contempt of themselves. High courts
have the power to punish for contempt for lower
 Power to summon and enforce the attendance courts under respective jurisdiction.
of any witness and to examine him/her on  Statement 3 is incorrect: Constitution of India
oath. does not define any type of contempt of court,
 Power to enforce the discovery and production neither Civil contempt nor criminal contempt.
of any document.  Statement 4 is correct: In India it is the Parliament
who has the power to legislate over Contempt of
 Power to receive evidence on affidavits,
Court.
 Power for requisitioning of any public record
or document or copy thereof or from any court. 15. Exp. (b)
 Statement 1 is incorrect: Government law
 Such other matters as may be prescribed
officers, legal firms, corporate lawyers and patent
 Every Lok Adalat shall have the power to specify attorneys all are recognised as advocates.

52
www.iasscore.in POLITY WORKBOOK

 Statement 2 is correct: Bar councils have a contractual obligation; (e) against the president
the powers to lay down rules relating to legal of India or the state governors; and (f) against
education and recognition of law colleges. the chief justice of a high court acting in judicial
capacity.
16. Exp. (c)
Quo-Warranto
Mandamus
 Quo-Warranto In the literal sense, it means ‘by
 It literally means ‘we command’. It is a command what authority or warrant’. It is issued by the
issued by the court to a public official asking him court to enquire into the legality of claim of a
to perform his official duties that he has failed person to a public office. Hence, it prevents illegal
or refused to perform. It can also be issued usurpation of public office by a person. The writ
against any public body, a corporation, an can be issued only in case of a substantive public
inferior court, a tribunal or government for the office of a permanent character created by a
same purpose. The writ of mandamus cannot statute or by the Constitution. It cannot be issued
be issued (a) against a private individual or in cases of ministerial office or private office.
body; (b) to enforce departmental instruction that Unlike the other four writs, this can be sought
does not possess statutory force; (c) when the duty by any interested person and not necessarily by
is discretionary and not mandatory; (d) to enforce the aggrieved person.

CONSTITUTIONAL AND NON-CONSTITUTIONAL BODIES

1. Exp. (a) government servants, citizens framed with the


charges of sedition, threatening the integrity of
 A Constitutional body is a body which is
the nation, spying, unlawful use of government
established by the Constitution of India. Such
uniform, causing interventions in the armed
Constitutional bodies can only be created or
forces, and so on.
changed after a Constitutional Amendment bill is
passed and not by a regular government bill or a  Pair 2 is incorrectly matched: Under the Official
private bill. Secrets Act, 1923, No person in the vicinity of any
prohibited place shall obstruct, knowingly mislead
 National Commission for Backward Classes (NCBC) or otherwise interfere with or impede, any police
was initially constituted by the Central Govt by the officer, or any member.
National Commission for Backward Classes Act,
 Pair 3 is correctly matched: It seeks to decrease
1993 (27 of 1993) dated 2.4.1993 and so far the
the number of licensed firearms allowed per
Commission had been reconstituted 7 times up to
person and increase penalties for certain offences
2016.
under the Act.
 At present Commission has been accorded
Constitutional Status and constituted through 3. Exp : (d)
“The Constitution (One Hundred and Second  The Election Commission of India is an autonomous
Amendment) Act, 2018” Act dated 11.8.2018, constitutional authority responsible for administering
whereby Article 338B has been inserted, forming Union and State election processes in India.
a Commission for the socially and educationally
backward classes to be known as National  The body administers elections to the Lok Sabha,
Commission for Backward Classes. Rajya Sabha, and State Legislative Assemblies in
India, and the offices of the President and Vice
 National Human Rights Commission is a statutory
President in the country.
body established by Protection of Human Rights
Act, 1993  Originally the commission had only one election
commissioner but after the Election Commissioner
 National Law Commission is neither established
Amendment Act 1989, it has been made a multi-
a Constitution nor a statutory. It is formed on the
member body.
directive of Union Law Ministry.
 The commission consists of one Chief Election
 National Consumer Dispute Redressal Commission
Commissioner and two Election Commissioners.
has been established by Consumer Protection Act,
1986.
4. Exp : (d)
2. Exp. (b) NITI Aayog
 Pair 1 is correctly matched: The Indian Official  Planning Commission was replaced by a new
Secrets Act, 1923, applies to government officials, institution – NITI AAYOG on January 1, 2015 with

53
POLITY WORKBOOK www.iasscore.in

emphasis on ‘Bottom –Up’ approach to envisage referred to the Commission by the President.
the vision of Maximum Governance, Minimum  The Commission’s recommendations along with
Government, echoing the spirit of ‘Cooperative an explanatory memorandum with regard to the
Federalism’. actions done by the government on them are laid
before the Houses of the Parliament.
5. Exp : (c)  The FC evaluates the rise in the Consolidated Fund
 CAG is an independent authority under the of a state in order to affix the resources of the state
Constitution of India. Panchayats and Municipalities.
 He is the head of the Indian audit & account  The FC has sufficient powers to exercise its
department and chief Guardian of Public purse. functions within its activity domain.
 It is the institution through which the accountability  As per the Code of Civil Procedure 1908, the FC has
of the government and other public authorities (all all the powers of a Civil Court. It can call witnesses,
those who spend public funds) to Parliament and ask for the production of a public document or
State Legislatures and through them to the people record from any office or court.
is ensured.
7. Exp : (a)
 CAG derives its audit mandate from different
Recommendations of the Finance Commission.
sources like–
 The President shall cause every recommendation
 Constitution (Articles 148 to 151) made by the Finance Commission under the
 The Comptroller and Auditor General’s (Duties, provisions of this Constitution together with an
Powers and Conditions of Service) Act, 1971 explanatory memorandum as to the action taken
thereon to be laid before each House of Parliament.
 Important Judgments
8. Exp. (d)
 Instructions of Government of India
 Statement 1 is correct: The Tea Board of India is
 Regulations on Audit & Accounts-2007 an autonomous and statutory body created under
the Tea Act, 1953.
6. Exp : (d)
 Statement 2 is incorrect: Board of India is a state
Powers, Functions and Responsibilities of Fimance
agency of the Government of India under the
Commission
control of Ministry of Commerce and Industry.
 The Commission decides the basis for sharing the
divisible taxes by the centre and the states, and  Statement 3 is incorrect: Headquarters is in
the principles that govern the grants-in-aid to the Kolkata
states every five years.  Statement 4 is correct: Offices are located in
 Any matter in the interest of sound finance may be Kolkata, London, Moscow and Dubai.

GOVERNANCE & IR

1. Exp. (c) ■ De-husked coconut


 Statement 1 is correct: Government announces ■ Sugarcane (Fair and remunerative price)
minimum support prices (MSPs) for 22 mandated
■ Virginia flu cured (VFC) tobacco
crops.
 Statement 2 is Correct: In India, Niger is grown
 Cereals (7) - paddy, wheat, barley, jowar, bajra,
on an area of 2.61 lakh ha mainly during kharif.
maize and ragi
However, in Odisha it is a Rabi crop.
 Pulses (5) - gram, arhar/tur, moong, urad and  Statement 3 is correct: The tribal population uses
lentil Niger seed oil for cooking.
 Oilseeds (8) - groundnut, rapeseed/mustard,
2. Exp. (c)
toria, soyabean, sunflower seed, sesamum,
safflower seed and nigerseed Anameia Mukt Bharat Strategy
■ Raw cotton  Statement 1 is incorrect: Prophylactic Iron Folic
Acid Supplementation to all six beneficiaries
■ Raw jute age group - pre-school children, adolescents and
■ Copra pregnant women.

54
www.iasscore.in POLITY WORKBOOK

 Statement 2 is correct: It runs a campaign for  Green Development, Climate Finance &LiFE
promotion and monitoring of delayed clamping of
 Accelerated, Inclusive & Resilient Growth
the umbilical cord for at least 3 minutes (or until
cord pulsations cease) for newborns across all  Accelerating progress on SDGs
health facilities will be carried out for improving
the infant’s iron reserves up to 6 months after birth.  Technological Transformation &Digital Public
Simultaneously, all birth attendants should make Infrastructure
an effort to ensure early initiation of breastfeeding  Multilateral Institutions for the 21st century
within 1 hour of birth.
 Women-led development
 Statement 3 is correct: Bi-annual mass deworming
for children in the age groups between 1-19 years 5. Exp. (b)
is carried out on designated dates – 10th February
 Both the statement is correct
and 10th August every year under National
Deworming Day (NDD) programme.  Saudi Arabia, as the main broker of the peace plan,
has an obvious stake in its success.
 Statement 4 is correct: It addressing non-
nutritional causes of anemia in endemic pockets,  While they remain proponents of resolving the
with special focus on malaria, haemoglobinopathies conflict with Israel through negotiations, they
and fluorosis.  have also made it clear that such resolution will
depend on Israel’s response to the peace plan.
3. Exp. (b)
Janani SurakshaYojana 6. Exp. (a)
 Statement 1 is incorrect: Janani Suraksha Yojana  The Global Compact for Safe, Orderly and
(JSY) is a safe motherhood intervention under the Regular Migration (A/RES/73/195), is the first
National Rural Health Mission (NHM). intergovernmental agreement, prepared under
the auspices of the United Nations, to cover all
 Statement 2 is correct: JSY is a safe motherhood
dimensions of international migration in a holistic
intervention under the National Rural Health
and comprehensive manner. It was adopted at an
Mission (NRHM) being implemented with the
intergovernmental conference on migration in
objective of reducing maternal and neo-natal
Marrakesh, Morocco on 10 December 2018.
mortality by promoting institutional delivery
among the poor pregnant women.  It is a non-binding document that respects states’
sovereign right to determine who enters and stays
 Statement 3 is correct: Janani Suraksha Yojana
in their territory and demonstrates commitment to
(JSY), a demand promotion and conditional cash
international cooperation on migration.
transfer scheme for promoting institutional
delivery. 7. Exp. (b)
 Statement 4 is incorrect: Janani Shishu Suraksha
 Statement 1 is correct: Trade and Technology
Karyakaram (JSSK) has been launched with the
Council was launched by the the United States of
objective to eliminate out of pocket expenses for
America and the European Union during the EU-
both pregnant women and sick infants accessing
US Summit on 15 June 2021 in Brussels.
public health institution for treatment.
 Statement 2 is incorrect: Trade and Technology
 The JSSK entitlement for pregnant women and council, the USA and EU does not aim to bring
sick infantsupto one year of age is operational technological progress and physical productivity
across all states, resulting in considerable under their control. Rather the initiative intends
reduction in out of pocket expenditures. to promote pooling of digital resources such
4. Exp. (c) as AI models and computing power, and make
them available to partner countries to address
 Statement 1 is correct: The G20 forum was challenges in areas such as climate change and
established in 1999 by the finance ministers extreme weather, healthcare or smart agriculture.
and central bank governors of seven countries
– Canada, France, Germany, Italy, Japan, the U.K., 8. Exp : (d)
and the U.S. The forum initially dealt with matters
related to macroeconomics, but over the years, 9. Exp : (a)
its agenda has expanded to cover issues relating to  Food Safety and Standards Authority of India
trade, climate change, sustainable development, (FSSAI) is an autonomous body established under
health, agriculture. the Ministry of Health, Family Welfare, Government
 Statement 2 is correct: G20 India has put forth of India. The FSSAI has been established under
six agenda priorities for the G20 dialogue in 2023 the Food Safety and Standards Act, 2006 which
which include – is a consolidating statute related to food safety

55
POLITY WORKBOOK www.iasscore.in

and regulation in India. FSSAI is responsible for  Availability of digital resources / services in Indian
protecting and promoting public health through languages
the regulation and supervision of food safety.
 Collaborative digital platforms for participative
 Prevention of Food Adulteration Act, 1954 governance
 Fruit Products Order, 1955  Citizens not required to physically submit Govt.
documents / certificates
 Meat Food Products Order, 1973
 Vegetable Oil Products (Control) Order, 1947 13. Exp : (a)

 Edible Oils Packaging (Regulation) Order 1988  Bharat Interface for Money (BHIM) is a mobile
payments application based on NPCI’s Unified
 Solvent Extracted Oil, De Oiled Meal and Edible Payments Interface (UPI). It provides the facility
Flour (Control) Order, 1967 to easily send or receive money from other
 Milk and Milk Products Order, 1992 customers using the UPI. BHIM is developed by
National Payments Corporation of India (NPCI)—
10. Exp : (d) the umbrella organisation for all retail payments
 Under the Uruguay and Singapore round system in India.
agreement, the creation of TRIPS agreement and  The BHIM app is tied to the mobile number and
its expansion has brought the following under device ID. So using the app on a different handset
WTO rulings. will require a separate authentication process.
 Copyright and Related Rights  Three-point authentication via mobile number,
 Trademarks device ID and UPI PIN.
 Fingerprint-based biometric verification of
 Geographical Indications
transactions via Aadhaar will prevent fraudulent
 Industrial Designs transactions.
 Patents  UPI PIN is needed for both opening the app as well
as for transactions.
 Layout Designs (Topographies) of Integrated
Circuits  Resetting UPI pin requires debit card details and
OTP SMS to authenticate.
 Protection of Undisclosed Information
 Control of AntiCompetitive Practices in 14. Exp : (b)
Contractual Licences  The benami (without a name) property refers to
 Thus the 1999 act for protection of GI was formed. property purchased by a person in the name of
some other person. The person on whose name
11. Exp :(b) the property has been purchased is called the
 Eligibility conditions of teachers are determined benamdar and the property so purchased is called
by the National Council for Teacher Education. the benami property. The person who finances the
deal is the real owner.
12. Exp : (b)  Assets of any kind — movable, immovable,
Digital Infrastructure as a Utility to Every Citizen tangible, intangible, any right or interest, or
legal documents. As such, even gold or financial
 Availability of high speed internet as a core utility
for delivery of services to citizens securities could qualify to be benami

 Cradle to grave digital identity that is unique,  The Act prohibits benami transactions and
lifelong, online and authenticable to every citizen provides legal provisions for confiscating benami
properties.
 Mobile phone & bank account enabling citizen
participation in digital & financial space  It defines a benami transaction as a transaction
where a property is held by or transferred to
 Easy access to a Common Service Centre
a person, but has been provided for or paid by
 Shareable private space on a public cloud another person.
 Safe and secure cyberspace Digital Empowerment
of Citizens 15. Exp : (b)
 The National Skills Qualifications Framework
 Universal digital literacy
(NSQF) is a competency-based framework that
 Universally accessible digital resources organizes all qualifications according to a series

56
www.iasscore.in POLITY WORKBOOK

of levels of knowledge, skills and aptitude. These  There are number of schemes but there is lack in
levels, graded from one to ten, are defined in terms creating synergy and linking between schemes to
of learning outcomes which the learner must achieve common goal.
possess regardless of whether they are obtained
 NNM through robust convergence mechanism and
through formal, non-formal or informal learning.
other components will strive to create synergy.
Specific outcomes expected from implementation
of NSQF are: 18. Exp : (a)
 Mobility between vocational and general  Vidyanjali - (School Volunteer Programme) is
education by alignment of degrees with NSQF an initiative of the Ministry of Human Resource
 Recognition of Prior Learning (RPL), allowing Development, Department of School Education &
transition from non-formal to organised job Literacy to enhance community and private sector
market involvement in Government run elementary
schools across the country under the overall aegis
 Standardised, consistent, nationally acceptable of the Sarva Shiksha Abhiyan. Union Minister for
outcomes of training across the country through a Human Resource Development, ‘Vidyanjali’- a
national quality assurance framework school volunteer programme to the nation.
 Global mobility of skilled workforce from India,  The aim of the Vidyanjali Yojana is to develop the
through international equivalence of NSQF society from the poor education system prevailing
 Mapping of progression pathways within sectors in some areas, mainly in rural districts. The
and cross-sectorally teacher recruitment process is there but, there is
 Approval of NOS/QPs as national standards for a huge demand of teachers mainly in the primary
skill training sections of the govt. schools in every state. So this
scheme not only provides a solution to the scarcity
16. Exp : (b) of teachers in govt. schools for now, but also gives
a noble path to thousands of aspirants to provide
National Career Service
teaching services in schools without being selected
 It is being implemented by Ministry of Labour in regular teaching recruitments.
and Employment (MoLE) since 2015 for linking
employment exchanges and other institutions 19. Exp : (b)
using technology.  Unnat Bharat Abhiyan is a flagship programme
 It aims to provide a variety of employment related of the Ministry of HRD, which aims to link the
services like job postings, career counselling, Higher Education Institutions with a set of at least
vocational guidance, skill courses, apprenticeship, 5 villages so that these institutions can contribute
etc. to the economic and social betterment of these
 These services are available online on the National village communities using their knowledge base.
Career Service Portal accessible to both employers  It is a significant initiative where all Higher
as well as job seekers. Learning Institutes have been involved for
 The NCS service can be availed directly through participation in development activities,
the Employment Exchanges/ Career Centres and particularly in rural areas.
Common Service Centres.  It also aims to create a virtuous cycle between the
society and an inclusive university system, with
17. Exp : (a) the latter providing knowledge base; practices
National Nutrition Mission: for emerging livelihoods and to upgrade the
 NNM is an apex body under Ministry of Women capabilities of both the public and private sectors.
and Child Development  Currently, 748 Institutions are participating under
 It will monitor, supervise, fix targets and guide the scheme.
nutrition related interventions across the The objective of the scheme is:
Ministries.
 To engage the faculty and students of Higher
 It will monitor various schemes contributing Educational Institutions in understanding rural
towards addressing malnutrition. realities;
Why needed?  Identify and select existing innovative technologies,
 There are number of schemes directly and enable customization of technologies, or devise
indirectly affecting nutritional status of children implementation methods for innovative solutions,
(0-6 years age) and pregnant women and lactating as required by people; and
mothers.  To allow Higher Educational Institutions to contribute
 Inspite of these, level of malnutrition and related to devising systems for smooth implementation of
problems in country is high. various Government Programs.

57
POLITY WORKBOOK www.iasscore.in

20. Exp : (a) activities taken under “polluter pays principle”


 The Unified Payment Interface is a payment system cannot be included under PMKKKY.
that allows money transfer between any two bank  60% of the funds will be utilised for high priority
accounts through the medium of a smartphone. areas such as drinking water supply, health care,
 UPI allows a customer to pay directly from a sanitation, education, skill development, women
bank account to different merchants, both online and child care, welfare of aged and disabled
and offline, without the hassle of typing credit people and environment conservation.
card details, IFSC code, or net banking/wallet  40% of the fund will be utilised for physical
passwords. infrastructure, irrigation, energy and watershed
 The new interface is built on the same development.
infrastructure as the Immediate Payment Service  The projects implemented under PMKKKY will
(IMPS), which is currently used by banks for real- help create a congenial mining environment,
time transfer of cash. Though the transaction limit ameliorate the condition of the affected persons
for IMPS is Rs.2 lakh per transaction, for UPI the and create a win-win situation for the stakeholders.
limit has been set at Rs.1 lakh.
 UPI has come as a boon to the banks which were 23. Exp : (c)
smarting under the onslaught of mobile wallets  Rashtriya Garima Abhiyan is National Campaign
like Citrus Pay, Oxigen, PayTM, Mobikwik etc. It for Dignity and Eradication of Manual Scavenging.
has helped them retrieve their lost ground. The practice continues in the country in spite
 It will facilitate micropayments and person-to- of efforts of several people, implementation of
person payment government schemes such as the National Scheme
for Liberation and Rehabilitation of Scavengers
21. Exp : (c) since 1992 and Self Employment Scheme for
Rehabilitation of Manual Scavengers since
 Digital Locker is a service launched by the Indian
2007, and regardless of it being banned in 1993
government in February 201 This service offers
through Employment of Manual Scavengers and
a safe and exclusive personal electronic storage
Construction of Dry Latrines (prohibition) Act.
space for resident citizens of India to store
documents.
24. Exp : (c)
 The maximum storage space is 10 MB. It is linked
to the Aadhaar number of user. This digital space 25. Exp : (a)
can be utilized for storing personal documents
Mission Indradhanush
such as university certificates, PAN cards, voter
IDs, the URIs of the e-documents issued by various  To give maximum protection to the children
departments. against Vaccine Preventable Diseases (VPDs). The
government has launched ‘Mission Indradhanush’
 There is also a related facility for e-signing of
in December 2014 to fully immunize more than
documents. The objective of the service is to
89 lakh children who are either unvaccinated or
decrease the use of physical documents and also to
partially vaccinated under UIP.
offer authenticity to e-documents.
 The mission was launched in 2014 and targets
 This service also provides a secure access to
children under 2 years of age and pregnant women
documents issued by the government. It also aims
for immunization.
to deflate the expenses on the administrative front
of the various governmental departments.  Mission Indradhanush provides vaccination
against 7 diseases diphtheria, whooping cough,
 Digital Locker is one of the major initiatives under
tetanus, polio, tuberculosis, measles and hepatitis
the Digital India Programme of the Government
B.
of India, and was released by the Electronics and
Information Technology Department of the Indian  In addition, vaccination against Japanese
Government. Encephalitis and Haemophilus influenzae type B is
being provided in selected districts of the country.
22. Exp : (b) Vaccination against tetanus is also provided to the
 District Mineral Foundation (DMF) is a trust set pregnant women.
up under Mines and Minerals (Development &  Mission Indradhanush aims to increase full
Regulation) Amendment Act (MMDRA) 2015 as a immunization coverage in India to at least 90%
non-profit body in those districts affected by the children by December 201
mining operations.  No separate funds are allocated for Mission
 The DMFs have been directed to take all major Indradhanush. Funds allocated for Routine
decisions in a participatory mode, in consultation Immunization are being utilized by the states to
with the gram sabhas of the respective villages and carry out activities under Mission Indradhanush.

58
www.iasscore.in POLITY WORKBOOK

 Mission Indradhanush does not targets to reduce DPAP: ministry of Rural development
postnatal death rate but targets to reduce diseases Objective
and death due to vaccine preventable diseases.
 The basic objective of the programme is to minimise
 Earlier the increase in full immunization coverage the adverse effects of drought on production of
was 1% per year which has increased to 7% per crops and livestock and productivity of land,
year through the first two phases of ‘Mission water and human resources ultimately leading
Indradhanush’. to drought proofing of the affected areas. The
programme also aims to promote overall economic
26. Exp : (a)
development and improving the socio-economic
 Initiative for Nutritional Security through Intensive conditions of the resource poor and disadvantaged
Millets Promotion (INSIMP) - a sub-scheme of sections inhabiting the programme areas.
RKVY has been launched from 2011-12 with aim
National Watershed Development
to demonstrate the improved production and post-
harvest technologies in an integrated manner with  Watershed management is the process of guiding
an allocation of Rs. 300 crores in 16 States. and organizing the use of land and other resources
in a watershed to provide desired goods and
 Millets are largely grown under rain¬fed condition
services without adversely affecting soil and water
and can thrive with rain fall of 300 mm, wherein resources. Each project under the programme is
major cereals like wheat and rice cannot be grown. a micro-level effort to achieve this objective by
Therefore, millets supplement the nutritional treating the under productive or unproductive
security of both human as well as livestocks in land and taking up allied activities for the benefit
such areas. of the landless. The programmes adopt a common
 The cultivation of millets is also supported by strategy of multi resource management involving
the Government through other programmes like all stakeholders within the watershed who,
Macro¬ Management in Agriculture (MMA), which together as a group, co-operatively identify the
provides support for organizing demonstrations of resource issues and concerns of the watershed as
improved package of practices, supply of certified well as develop and implement a watershed plan
seeds, seed minikits and supply of micro nutrients, with solutions that are environmentally, socially
gypsum and farmers training for production and economically sustainable.
and productivity improvement of coarse cereals  IWMP aims to restore ecological balance by
including millets. utilising, conserving and developing degraded
 Millets are the hardy crops against the disease/ natural resources such as soil, vegetation and
pests and also grown as trap crops in Oilseed water and helps in the prevention of soil run-off,
and Pulses for control of disease and pest regeneration of natural vegetation, rainwater
management. Therefore, they require lesser harvesting and recharging of the groundwater
support for chemicals, fertilizers and pesticides. table. These advantages enable farmers to
The operational guidelines of INSIMP provides introduce multi-cropping and start diverse agro-
flexibility to the States, to modify the contents of based activities, which can help provide sustainable
the input kits as per local situations in consultation livelihoods to a large number of people residing in
with their State Agriculture Universities/lCAR the watershed area
institutions.  Thus, the management of natural resources at
the watershed scale produces multiple benefits in
27. Exp : (d) terms of increasing food production, improving
 Desert Development Programme (DDP): water livelihoods, protecting the environment,
shed development department addressing gender and equity issues along with
biodiversity concerns
28. Exp : (c)  The Government of India has formulated
 To mitigate the adverse effects of desertification and guidelines for the implementation of an IWMP,
adverse climatic conditions on crops, human and which focuses not only on soil and water
livestock population, combating desertification. conservation but also converges all other relevant
developmental schemes such as planting trees,
 To restore ecological balance by harnessing, improving agriculture, forming self-help groups,
conserving and developing natural resources improving hygiene and sanitation, and managing
i.e. land, water, vegetative cover and rising land drinking water.
productivity.
 To implement developmental works through the 29. Exp : (c)
watershed approach, for land development, water  The draft five-year plan, prepared by the planning
resource development and afforestation/pasture commission ‘s first submitted to the union Cabinet.
developmen After its approval, it is placed before the National

59
POLITY WORKBOOK www.iasscore.in

development Council, for its acceptance. Then, the  To consider important questions of social
plan is presented to Parliament, with its approval, and economic policy affecting national
it emerges as the official plan and published in the development
official Gazette.
 To review the working of the Plan from time
30. Exp : (d) to time
 To recommend such measures that are
31. Exp : (b) necessary for achieving the aims and targets
 National Development Council (NDC) is an set out in the National Plan.
executive body established by the Government
of India in August 1952, which is neither a 32. Exp : (b)
constitutional nor a statutory body. It is the apex
body to take decisions on matters related to 33. Exp : (c)
approval of five year plans of the country. Prime
 The National Legal Services Authority (NALSA) was
minister is the ex-officio chairman of the NDC.
constituted under the Legal Services Authorities
Composition Act, 1987 which came into force on 9th November,
 National Development Council is composed of the 1995 to establish a nationwide uniform network
members mentioned below: for providing free and competent legal services to
the weaker sections of the society.
 Prime Minister of India (Chairman of NDC)
 The Chief Justice of India is the Patron-in-Chief and
 Chief Ministers of all states the second senior most Judge of Supreme Court of
 Administrators of all Union Territories India is the Executive Chairman of the Authority.
 Article 39 A of the Constitution provides for free
 All cabinet ministers
legal aid to the poor and weaker sections of the
 Members of the Planning Commission society, to promote justice on the basis of equal
 The secretary of the Planning Commission is also opportunity. Article 14 and Article 22 (1), obligates
the secretary of the NDC. The administrative State to ensure equality before law.
assistance is also provided by the Planning
Commission. 34. Exp :(a)

Objectives  Constitution mentions that Gram Sabha exercises


such powers and performs such functions at the
 NDC is an advisory body to the Planning village level as the Legislature of a State may,
Commission. The major objectives of NDC can be
by law, provide. For instance, they approve of
listed below:
the plans, programmes and projects for social
 To strengthen and mobilize the effort and and economic development before such plans,
resources of the nation in support of the Plan. programmes and projects are taken up for
implementation by the Panchayat at the village
 To promote common economic policies in all
level (Panchayat at the village level is otherwise
vital spheres.
known as Gram Panchayat).
 To ensure the balanced and rapid development Following are the important and specific functions
of all parts of the country. of Gram Sabha:
 In addition to this, NDC provides a platform  To help implementation of the development
to all the states to discuss their problems and programmes and schemes of the Panchayat.
issues related to development. Thus, it secures
the cooperation of the states in the execution of  To identify beneficiaries for different programmes
developmental plans. and schemes. However, if the Gram Sabha fails to
identify such beneficiaries within a reasonable
Functions
time, the Gram Panchayat shall identify the
 To meet its objectives, the NDC has been assigned beneficiaries.
below functions:
 To solicit support — in cash or kind or both and
 To prescribe guidelines for the formulation of voluntary labour — from the public for community
the National Plan, including the assessment of welfare programmes.
resources for the Plan
 To support the programmes of mass education and
 To consider the National Plan as formulated by family welfare.
the Planning Commission  To promote unity and harmony among all sections
 To make an assessment of the resources of the society in the village.
required to implement the plan and the way to  To seek clarification from the Mukhiya, Up-
augment the resources. Mukhiya and other members of the Gram

60
www.iasscore.in POLITY WORKBOOK

Panchayat about any particular activity, scheme, 36. Exp : (c)


income and expenditure.  Consumer rights are protected in the way of legal
 To discuss and recommend appropriate action action – Consumer Protection right act, 198 It
with regard to reports of the Vigilance Committee. protects the six basic rights for consumers.
 Other related matters brought to the notice of the  Consumers have the right to conduct the test of
Gram Sabha. sample food. When a consumer files a complaint in
 To consider levy of taxes, rates, rents & fees & any consumer forum, some considerable amount
enhancement of rates thereof. of fee is required.

 To consider all such matters as may be referred by  In case the consumer dies, his or her legal heir
the Gram Panchayat for its decision. can approach the consumer forum and file the
complaint on his or her behalf.
35. Exp :(a)
37. Exp : (c)
 National Green Tribunal (NGT) is a specialised
body set up under the National Green Tribunal  The Universal Declaration of Human Rights
Act (2010) for effective and expeditious disposal has 30 Articles. The relevant articles are being
of cases relating to environmental protection and reproduced below:
conservation of forests and other natural resources. Article 26.
 With the establishment of the NGT, India became  Everyone has the right to education. Education
the third country in the world to set up a specialised shall be free, at least in the elementary and
environmental tribunal, only after Australia and fundamental stages. Elementary education
New Zealand, and the first developing country to shall be compulsory. Technical and professional
do so. education shall be made generally available and
 NGT is mandated to make disposal of applications higher education shall be equally accessible to all
or appeals finally within 6 months of filing of the on the basis of merit.
same. Article 21.
 The NGT has five places of sittings, New Delhi is  Everyone has the right to take part in the
the Principal place of sitting and Bhopal, Pune, government of his country, directly or through
Kolkata and Chennai are the other four. freely chosen representatives.
Structure of NGT  Everyone has the right of equal access to public
 The Tribunal comprises of the Chairperson, the service in his country.
Judicial Members and Expert Members. They  The will of the people shall be the basis of the
shall hold office for term of five years and are not authority of government; this will shall be
eligible for reappointment. expressed in periodic and genuine elections which
 The Chairperson is appointed by the Central shall be by universal and equal suffrage and shall
Government in consultation with Chief Justice of be held by secret vote or by equivalent free voting
India (CJI). procedures.

 A Selection Committee shall be formed by central Article 25.


government to appoint the Judicial Members and  Everyone has the right to a standard of living
Expert Members. adequate for the health and well-being of himself
 There are to be least 10 and maximum 20 full time and of his family, including food, clothing, housing
Judicial members and Expert Members in the and medical care and necessary social services, and
tribunal. the right to security in the event of unemployment,
sickness, disability,
Powers & Jurisdiction
 The Tribunal has jurisdiction over all civil 38. Exp : (c)
cases involving substantial question relating to  The correct option is C Only 3.
environment (including enforcement of any legal
right relating to environment).  National Social Assistance Programme started in
1995 carries National Old Age Pension Scheme,
 Being a statutory adjudicatory body like Courts, National Family Benefit Scheme and National
apart from original jurisdiction side on filing of an Maternity Benefit Scheme. The Programme ensures
application, NGT also has appellate jurisdiction to minimum national standard of social assistance
hear appeal as a Court (Tribunal). which are related to Directive Principles of State
 The Tribunal is not bound by the procedure laid Policy for social causes that are manifested in
down under the Code of Civil Procedure 1908, but National Social Assistance Programme, so option
shall be guided by principles of ‘natural justice’. B. is correct

61
POLITY WORKBOOK www.iasscore.in

39. Exp : (d) District Authority should involve the MPs in the
 In general, fiscal deficit occurs when the total inspections of projects to the extent feasible.
expenditures of the government exceed its
revenue. The government takes various measures 42. Exp : (a)
to reduce the fiscal deficit such as increasing tax-  The National Legal Services Authority (NALSA)
based revenue, reducing subsidies, disinvestment, has been constituted under the Legal Services
etc. Downsizing of bureaucracy as well as selling/ Authorities Act, 1987 to provide free Legal
offloading the shares of public sector undertaking Services to the weaker sections of the society and
directly contributes to reduction in fiscal deficit. to organize Lok Adalats for amicable settlement of
Without knowing the destination and the effect disputes.
of FDI inflows, it is difficult to determine its actual  The maximum income ceiling for providing legal
impact on the fiscal deficit. Privatisation of higher aid services is 1 lakh (in the Supreme Court Legal
educational institutions may improve the situation Services Committee the limit is Rs. 1,25,000/-).
but its impact may not be effective in reduction of
 Low income(Annual income less than Rs. 1,00,000
fiscal deficit. Hence, statements 3, 4 are correct
and statements 1, 2 are not correct. Therefore,  Senior Citizen (Annual income less than Rs. 2
option (d) is the correct answer. Lac)
 Transgender (Annual income less than Rs. 2
40. Exp : (b)
Lac)
 Statement 1 is incorrect: Supreme Court had
ruled that Aadhaar metadata cannot be stored 43. Exp : (c)
for more than six months.
 Statement 1 is correct: Alma-Ata declaration-The
 Statement 2 is correct: The Supreme Court struck International Conference on Primary Health Care,
down part of section 57 which allowed providing meeting in Alma-Ata this twelfth day of September
private corporations to verify Aadhaar data as in the year Nineteen hundred and seventy-eight,
unconstitutional. expressing the need for urgent action by all
 Statement 3 is incorrect: The same 2018 governments, all health and development workers,
judgement also held making that Aadhaar and the world community to protect and promote
mandatory for other services apart from welfare the health of all the people of the world.
schemes as unconstitutional. Subsequently, IRDAI  Statement 2 is incorrect: The Hague Convention
issued an advisory in January 2019 that clarified protects children and their families against the
that Aadhaar is not mandatory for obtaining risks of illegal, irregular, premature or ill-prepared
insurance. adoptions abroad.
 Statement 4 is correct: The Supreme court in the  Statement 3 is correct: The Talanoa Dialogue is
Aadhaar verdict had defined the welfare schemes a process designed to help countries implement
as those funded from the consolidated fund of and enhance their Nationally Determined
India. And it upheld the validity of Section 7 of Contributions by 2020.
the Aadhaar Act which states that Central or State
 Statement 4 is incorrect: The Under2 Coalition is a
Governments can make possession of an Aadhaar
global community of state and regional governments
number or Aadhaar authentication mandatory for committed to ambitious climate action in line with
receipt of subsidies, benefits or services funded the Paris Agreement. The coalition brings together
out of the Consolidated Fund of India. more than 220 governments who represent over
1.3 billion people and 43% of the global economy.
41. Exp : (d)
 Statement 1 is correct: The emphasis is on 44. Exp : (d)
creating durable community assets based on  They are politically neutral and their main
locally felt needs. job is to facilitate the effective working of
 Statement 2 is correct: MPs are to recommend different government departments and policy
every year, works costing at least 15 per cent of implementations.
the MPLADS entitlement for the year for areas
45. Exp: (d)
inhabited by Scheduled Caste population and 7.5
per cent for areas inhabited by S.T. population.  Statement 1 is incorrect: Ayyangar was part of
the 7-member drafting committee of the Indian
 Statement 3 is incorrect: The funds released
constitution appointed on August 29, 1947.
under the Scheme are non-lapsable
Sh. N. Gopalaswamy Ayyangar, in his Report
 Statement 4 is correct: The District Authority namely ‘Reorganization of the Machinery of
would be responsible for overall coordination Central Government’ in 1950 recommended
and supervision of the works under the scheme for the grouping of ministries, improvement in
at the district level and inspect at least 10% of the capabilities of the personnel, and also in the
the works under implementation every year. The working of the O&M Division.

62
www.iasscore.in POLITY WORKBOOK

 Statement 2 is incorrect: In 1970, on the basis  The Ministry of the Central Government dealing
of the recommendations of the Administrative with Tribal Affairs or any officer or authority
Reforms Commission, the Department of Personnel authorised by the Central Government on
was set up in the Cabinet Secretariat (NOT PM). this behalf shall be the nodal agency for the
implementation of the provisions of this Act.
46. Exp: (d)
49. Exp. (b)
Forest Rights Act, 2006
 The Forest Rights Act (FRA), 2006 recognizes the  Statement 1 is incorrect: Participation in ABDM is
rights of the forest dwelling tribal communities voluntary including for citizens. Participation of a
and other traditional forest dwellers to forest healthcare facility or an institution is also voluntary
resources, on which these communities were and shall be taken by the respective management
dependent for a variety of needs, including (government or private management). However,
livelihood, habitation and other socio-cultural once the management decides to register the
needs. respective healthcare facility/institution in ABDM,
it is essential for all the healthcare professionals
 The forest management policies, including the serving the said facility/institution to register
Acts, Rules and Forest Policies of Participatory in Healthcare Professionals Registry so that the
Forest Management policies in both colonial and institution can become fully integrated with the
post-colonial India, did not, till the enactment of National Digital Health Ecosystem (NDHE).
this Act, recognize the symbiotic relationship of the
 By elimination, Option (b) is correct
STs with the forests, reflected in their dependence
on the forest as well as in their traditional wisdom 50. Exp. (d)
regarding conservation of the forests.
 An open source platform is any platform that
 The Ministry of the Central Government dealing allows access to its source code to any other
with Tribal Affairs or any officer or authority users or developers. An open source platform is
authorised by the Central Government on one aspect of a wide availability of open source
this behalf shall be the nodal agency for the products. In contrast with closed source software,
implementation of the provisions of this Act. which is a type of proprietary software that
47. Exp: (d) reserves rights only to authorized individuals,
open source software aims to allow equal access to
 Statement 1 is incorrect: Ayyangar was part of anyone and everyone. Open source products are
the 7-member drafting committee of the Indian oftentimes part of the free software movement,
constitution appointed on August 29, 1947. and the associated Free Software Foundation
Sh. N. Gopalaswamy Ayyangar, in his Report founded by Richard Stallman in 1985.
namely ‘Reorganization of the Machinery of
 Rather than being concerned with the cost of
Central Government’ in 1950 recommended
the software, the free software movement, and
for the grouping of ministries, improvement in
associated foundation, aim to ensure that software
the capabilities of the personnel, and also in the
users are free to run software, study the software,
working of the O&M Division.
modify the software, and share such modifications.
 Statement 2 is incorrect: In 1970, on the basis of the However, not all open source platforms are part
recommendations of the Administrative Reforms of the free software movement, but they are
Commission, the Department of Personnel was set almost always part of the open source software
up in the Cabinet Secretariat (NOT PM). movement.

48. Exp: (d)  AarogyaSetu , India’s coronavirus ( COVID-19 )


contact-tracing and self-assessment app is now
Forest Rights Act, 2006 open source .
 The Forest Rights Act (FRA), 2006 recognizes the  DigiLocker is an initiative by the government to
rights of the forest dwelling tribal communities offer Indian citizens a free platform to store and
and other traditional forest dwellers to forest access important documents. The platform uses
resources, on which these communities were several open source technologies to deliver a mass
dependent for a variety of needs, including solution and contributes back to the ever-growing
livelihood, habitation and other socio-cultural community.
needs.  DIKSHA is built using Sunbird ED, a fully functional
 The forest management policies, including the solution building block available as open-source
Acts, Rules and Forest Policies of Participatory software under MIT license and is part of Sunbird,
Forest Management policies in both colonial and a “Made in India, Made for the World” digital
post-colonial India, did not, till the enactment of public good (DPG).
this Act, recognize the symbiotic relationship of the  Co-WIN, which is owned by the Ministry of Health
STs with the forests, reflected in their dependence and Family Welfare, was developed using open
on the forest as well as in their traditional wisdom source software. This type of software allows users
regarding conservation of the forests. to modify the source code and distribute it.

63
POLITY WORKBOOK www.iasscore.in

51. Exp. (b) international organizations and other entities


Permanent Observer Status.
 Statement 1 is incorrect: ISC is a multi-
stakeholder platform that brings together the  Statement 2 is correct: General Assembly
private sector, government, financial institutions, decided that observer status would be confined
civil society groups, media, donors/bi-lateral/ to States and intergovernmental organizations
multilateral, and experts etc. to work in the whose activities cover matters of interest to the
sanitation space to drive sustainable sanitation Assembly.
through a partnership model.
 Statement 3 is correct: Permanent Observers
 Statement 2 is correct: The National Institute of may participate in the sessions and workings of
Urban Affairs (NIUA) is India’s leading national the General Assembly and maintain missions at
think tank on urban planning and development. the UN Headquarters.
As a hub for generation and dissemination of
cutting-edge research in the urban sector, NIUA 55. Exp. (d)
seeks to provide innovative solutions to address
 Statement 1 is correct: Every State has the right
the challenges of a fast urbanizing India.
to establish the breadth of its territorial sea up to
 It is against this backdrop that in 1976, NIUA a limit not exceeding 12 nautical miles, measured
was appointed as an apex body to support and from baselines determined in accordance with
guide the Government of India in its urban this Convention.
development plans. Since then, it has worked
closely with the Ministry of Housing and Urban  Statement 2 is correct: The innocent passage has
Affairs, alongside other government and civil been codified in the United Nations Convention
sectors, to identify key areas of research, and on the Law of the Sea (UNCLOS III) was adopted
address the lacunae in urban policy and planning. in 1982, it is also known as the Law of the Sea
Treaty. Its purpose is to establish a comprehensive
52. Exp. (a) set of rules governing the oceans and to replace
 Statement 1 is incorrect: The United Nations previous U.N. Conventions on the Law of the Sea,
Credentials Committee is a committee of the 1958 (UNCLOS I) which was adopted in 1958 and
United Nations General Assembly. another in 1960 (UNCLOS II), since these two
conventions were believed to be inadequate. The
 Statement 2 is incorrect: A Credentials
Committee is appointed at the beginning of each right of innocent passage of foreign ships through
regular session of the General Assembly. the territorial waters of a coastal state is one of the
oldest and most universally recognized rules of
 Statement 3 is correct: Its main purpose is to public international law.
report to the Assembly regarding the credentials
of the body’s representatives.  Statement 3 is incorrect: The continental shelf
and the exclusive economic zone (EEZ) are
53. Exp. (a) distinct maritime zones. The continental shelf
 Statement (a) is correct: Polar Code is includes only the seabed and subsoil; whereas the
International Code for Ships Operating in Polar EEZ includes the water column. Also, while the
Waters. The Polar Code covers the full range of maximum extent of the EEZ is 200 nautical miles,
design, construction, equipment, operational, the continental shelf may extend beyond 200
training, search and rescue and environmental nautical miles from the coastline, depending on
protection matters relevant to ships operating the depth, shape, and geophysical characteristics
in the inhospitable waters surrounding the two of the seabed and sub-sea floor. The ECS is,
poles. therefore, not an extension of the EEZ. Some of the
sovereign rights that a coastal State may exercise
54. Exp. (d) in the EEZ, especially rights to the resources of the
 Statement 1 is correct: The United Nations water column (e.g., pelagic fisheries), do not apply
General Assembly may grant non-member states, to the ECS.

******

64
www.iasscore.in POLITY WORKBOOK

MISCELLANEOUS

1. Exp. (b) 3. Exp: (d)


 Statement 1 is correct: The 44th Chess Olympiad  Statement (1) is correct : In 1996 , the Supreme
was an international team chess event organised Court upheld the constitutional validity of
by the Fédération Internationale des Échecs (FIDE) the National Awards–Bharat Ratna, Padma
in Chennai, India from 28 July to 10 August 2022. Vibhushan, Padma Bhushan and Padma Sri. It
This was the first Chess Olympiad to take place ruled that these awards do not amount to ‘titles’
in India. within the meaning of Article 18 that prohibits
 Statement 2 is correct: The Official Mascot of 44th only hereditary titles of nobility. Therefore, they
Chess Olympiad is ‘Thambi’. The word ‘Thambi’ are not violative of Article 18 as the theory of
in Tamil language means - little or younger equality does not mandate that merit should not
brother. be recognised.
 Statement 3 is incorrect: The trophy for  Statement (2) is incorrect: Padma Awards, which
the winning team in the Open section is the were instituted in the year 1954, is announced
Hamilton-Russel Cup. every year on the occasion of Republic Day except
 Statement 4 is incorrect: The trophy for the for brief interruption(s) during the years 1978 and
winning team in the women’s section is the Vera 1979 and 1993 to 1997.
Menchik Cup.
 Statement (3) is incorrect: Padma Awards, which
2. Exp. (b) were instituted in the year 1954, is announced
every year on the occasion of Republic Day except
Option 1 and option 3 is correct
for brief interruption(s) during the years 1978 and
 Major Dhyan Chand khelRatna award is correct 1979 and 1993 to 1997.
 Dronacharya award is correct
4. Exp: (d)
Arjuna Award:
 Statement 1 is incorrect: As per the current
 It was instituted in 1961 by the Government of
regulations, WTC league standings would be
India to recognise outstanding achievement
determined by the percentage of points (PCT)
in national sports events.
earned by teams. PCT is the percentage of points
 It is given for good performance over a period won out of the total number of points contested
of previous four years and showing qualities by each team.
of leadership, sportsmanship and a sense of
 Statement 2 is incorrect: New Zealand was
discipline.
ranked ahead of England in WTC 2019-21 table
 The award carries a cash prize of Rs 15 lakh, a because the percentage of points won by New
bronze statue of Arjuna and a scroll of honour. Zealand was more than England.
Dronacharya Award:
5. Exp: (c)
 It was instituted in 1985 by the Government
 Statement (1) is correct: Laureus World
of India to recognise excellence in sports
coaching. Sportsman of the year 2000 was given to Tiger
Woods.
 It is given to coaches for doing outstanding
 Statement (2) is incorrect: The award was
and meritorious work on a consistent basis
and enabling sportspersons to excel in received mostly by tennis players.
international events.  Statement (3) is correct: Most awards: Roger
Federer (5)
 It carries a cash prize of Rs 15 lakh, a bronze
statue of Dronacharya and a scroll of honour. 6. Exp: (b)
Dhyan Chand Award:  Statement (1) is incorrect: The International
 It was instituted in the year 2002 and comprises Olympic Committee decided upon the change with
a Dhyan Chand statuette, a cash prize of Rs 10 the new motto being, “Faster, Higher, Stronger -
lakh, a certificate and a ceremonial dress. Together.” The specifi c motto for the Tokyo games
remains unchanged, “United by Emotion.”
 It is given to honour sportspersons who have
contributed to sports by their performance  Statement (2) is correct: Surfing, Skateboarding,
and continue to contribute to promotion of Sport Climbing, Karate, Baseball and Softball are
sports events after their retirement. included in this olympics.

65

You might also like